BRS Pathology ALL QUESTIONS

¡Supera tus tareas y exámenes ahora con Quizwiz!

The answer is C. Retcodes for a transmembrane receptor tyrosine kinase that is mutated in the MEN IIa and MEN IIb syndromes, as well as in sporadic cases of medullary carcinoma of the thyroid. Bcr-ablfusion results from the chromosomal translocation of chronic myelogenous leukemia. N-mycis amplified in neuroblastoma. Amylin derived from islet amyloid polypeptide accumulates in the pancreatic islets in type 2 diabetics.

A 15-year-old boy presents to the endocrinologist with multiple mucocutaneous neuromas and a marfanoid habitus (tall with long extremities). His older brother has had a thyroidectomy for medullary carcinoma of the thyroid and later has been diagnosed with bilateral tumors of the adrenal medulla. It is likely that further investigation in both brothers will demonstrate an abnormality in which of the following genes or gene products? (A) Bcr-abl (B) N-myc (C) Ret (D) Amylin (E) Insulin-associated polypeptide

The answer is D. This question illustrates an important point: in young patients, bone malignancies showing prominent cartilaginous differentiation are almost assuredly chondroblastic osteosarcomas, rather than chondrosarcomas. Conventional chondrosarcomas occur almost exclusively in older patients.

A 15-year-old boy presents with a pathologic fracture following a minor injury on the soccer field. The area of fracture is surrounded by a large tumor which shows marked pleomorphism, high mitotic activity, and extensive cartilaginous differentiation on microscopy. The most likely diagnosis is (A) giant cell tumor. (B) osteochondroma. (C) chondrosarcoma. (D) osteosarcoma. (E) Ewing sarcoma.

The answer is D. In spite of the somewhat atypical presentation, the histologic findings are typical of contact dermatitis, which is a classic example of T cell-mediated (type IV) hypersensitivity.

A 16-year-old boy is referred to a dermatologist by an emergency room physician. The patient has had an intractable, severe, itching, burning, pruritic rash of the hands and lower extremities that has lasted several hours. Earlier in the day he had hiked through a wooded area filled with brush but was unaware of any direct contact with plants or other possible irritants. A skin biopsy revealed many infiltrating T cells and macrophages, suggesting an immune hypersensitivity reaction. Which type of reaction is most likely? (A) Type I (immediate or anaphylactic) hypersensitivity (B) Type II (antibody-mediated or cytotoxic) hypersensitivity (C) Type III (immune complex) hypersensitivity (D) Type IV (cell-mediated) hypersensitivity

The answer is D. Sickle cell anemia is the most common hereditary anemia in persons of African lineage. Leg ulcers and recurring painful crises are characteristic. In sickle cell anemia, in contrast to sickle cell trait, sickle cells are often seen on the peripheral blood smear.

A 23-year-old African-American man with a history since early childhood of severe anemia requiring many transfusions has nonhealing leg ulcers and recurrent periods of abdominal and chest pain. These signs and symptoms are most likely to be associated with which of the following laboratory abnormalities? (A) Decreased erythropoietin (B) Increased erythrocyte osmotic fragility (C) Schistocytes (D) Sickle cells on peripheral blood smear (E) Teardrop-shaped cells

The answer is D. Kaposi sarcoma is a malignant vascular tumor that occurs in men infected with the Kaposi sarcoma herpesvirus (KSHV), often as a complication of AIDS. Angiosarcoma is a malignant tumor of blood vessels associated with thorium dioxide (Thorotrast); arsenic; or in the case of angiosarcoma of the liver, polyvinyl chloride. Polyarteritis nodosa is a vasculitis sometimes associated with hepatitis B infection. Serum sickness is due to deposition of antigen-antibody complexes.

A 23-year-old man known to have acquired immunodeficiency syndrome (AIDS) is seen by his primary care physician. The patient is concerned by the development of multiple red-to-purple skin plaques that have become nearly confluent on his arms and legs. The skin disorder is most likely (A) angioedema. (B) angiosarcoma. (C) erythema multiforme. (D) Kaposi sarcoma. (E) multiple telangiectasias of Osler-WeberRendu syndrome.

The answer is B. The vignette describes an instance of traumatic fat necrosis, which must be distinguished from enzymatic fat necrosis. The description of amorphous basophilic material is indicative of calcification, and calcification of previous damaged tissue is termed dystrophic calcification. Dystrophic calcification must be distinguished from metastatic calcification, which occurs in the presence of hypercalcemia and affects nondamaged tissues.

A 25-year-old woman is struck in the left breast in a "steering wheel" injury in what appeared to be a minor auto accident. After several days of pain and tenderness, she noted the persistence of a "lump" at the site of the injury. After excision biopsy, amorphous basophilic material was noted within the mass. The amorphous material is an example of (A) apocrine metaplasia. (B) dystrophic calcification. (C) enzymatic fat necrosis. (D) granulomatous inflammation. (E) mammary dysplasia.

The answer is C. The most common effect of maternal diabetes mellitus and hyperglycemia on the child is increased birth weight. This also increases the likelihood of obstetric complications, including the need for cesarean section and increased likelihood of brachial plexus injuries. Another complication is hyaline membrane disease. Cretinism results from deficiency of thyroid hormone during fetal development and during postnatal life. Ambiguous genitalia can occur in any of the adrenogenital syndromes. Sheehan syndrome occurs in the mother and has no relationship to diabetes. Thyroglossal duct cysts do not usually result in endocrine complications.

A 26-year-old primigravida develops gestational diabetes and remains hyperglycemic during the remainder of her pregnancy. Which of the following abnormalities in the newborn child is likely related to the maternal hyperglycemia? (A) Ambiguous genitalia (B) Cretinism (C) Increased birth weight (D) Sheehan syndrome (E) Thyroglossal duct cyst

The answer is E. Thromboxane A2(TxA2) promotes platelet aggregation, as does ADP. Aspirin irreversibly inhibits the enzymes cyclooxygenase 1 and 2 and thereby the synthesis of TxA2, thus inhibiting platelet aggregation, which is thought to be an important early step in atherogenesis. A negative but apparently unimportant consequence of aspirin prophylaxis is the parallel inhibition of synthesis of the antiaggregant endothelial PGI2, also a product of the cyclooxygenase pathway.

A 56-year-old man is surgically treated by a four-vessel coronary artery bypass graft procedure and placed on prophylactic daily aspirin therapy. Aspirin has been shown to prevent recurrent myocardial infarction through its ability to inhibit the synthesis of (A) adenosine diphosphate (ADP). (B) leukotriene B4(LTB4). (C) nitric oxide (NO). (D) prostaglandin I2(PGI2). (e) thromboxane A2(TxA2).

The answer is D. If infarction is averted by immediate thrombolytic therapy, indicators of necrosis, such as karyorrhexis, pyknosis, and karyolysis, which represent irreversible changes, would not be expected. Swelling of the endoplasmic reticulum from increased cell water, one of the earliest ultrastructural changes observed in injured cells, is reversible and would be expected.

A 56-year-old man recovered from a myocardial infarction after his myocardium was entirely "saved" by immediate thrombolytic therapy. If it had been possible to examine microscopic sections of his heart during his ischemic episode, which of the following would be the most likely cellular change to be found? (A) Karyolysis (B) Karyorrhexis (C) Pyknosis (D) Swelling of the endoplasmic reticulum

The answer is A. A diet low in fiber and high in fat is believed to be a risk factor for the development of colon cancer. Both aflatoxin B1 ingestion and hepatitis B infection are risk factors for hepatocellular carcinoma. Helicobacter pyloriinfection is associated with stomach cancer. Cancers of the mouth, tongue, and esophagus have a marked association with the combined abuse of tobacco and alcohol, as well as HPV infection in a subset of cases

A 56-year-old man was recently diagnosed with early-stage colon cancer. He has no known family history of colon cancer. Which of the following known risk factors is most likely to have contributed to the development of this form of cancer? (A) A diet low in fiber and high in fat (B) Aflatoxin B1 ingestion (C) Helicobacter pyloriinfection (D) Hepatitis B infection (E) Tobacco and alcohol abuse

The answer is C. The combination of generalized edema, massive proteinuria, hypoalbuminemia, and hyperlipidemia constitutes the nephrotic syndrome, the prototype of which is minimal change disease (lipoid nephrosis). This disorder characteristically occurs in young children and demonstrates intracytoplasmic lipid in the proximal convoluted tubules, a paucity of glomerular abnormalities by light microscopy, and "fusing" (absence) of the epithelial foot processes by electron microscopy.

A 3-year-old girl presents with generalized edema shortly after recovery from an upper respiratory infection. Laboratory studies reveal marked albuminuria, as well as hypoalbuminemia and hyperlipidemia. Prior similar episodes responded to adrenal steroid medication. The most likely diagnosis is (A) focal segmental glomerulosclerosis. (B) membranous glomerulonephritis. (C) minimal change disease. (D) poststreptococcal glomerulonephritis. (E) rapidly progressive glomerulonephritis.

The answer is E. The clinical picture strongly suggests a diagnosis of sarcoidosis. The granulomas of sarcoidosis are characteristically noncaseating. Sarcoidosis is a multisystem disorder. Common findings in this highly variable disorder include anergy to tuberculin, hypercalcemia, and broad-based polyclonal hypergammaglobulinemia.

A 30-year-old African-American woman presents with bilateral hilar lymphadenopathy and reticular densities in both lung fields. Which of the following is a defining characteristic of the disorder suggested by these findings? (A) Abnormalities restricted to lung and hilar lymph nodes (B) Hypocalcemia (C) Impaired synthesis of immunoglobulins (D) Marked hyperreactivity to tuberculin (E) Noncaseating granulomas

The answer is B. Adult polycystic kidney disease is the most common inherited disorder of the kidney and is characterized by autosomal dominant inheritance. The disease is characterized by partial replacement of the renal parenchyma with cysts. An important association of adult polycystic kidney disease is berry aneurysm of the circle of Willis.

A 30-year-old man presents with hypertension, hematuria, palpable renal masses, and flank pain. He states that a kidney disease runs in his family, and his mother and maternal grandfather have it. Laboratory abnormalities confirm renal failure. Which of the following is associated with the disorder described here? (A) Autosomal recessive inheritance (B) Berry aneurysm of the circle of Willis (C) Clinical manifestation most commonly at birth (D) Multiple small medullary cysts in both kidneys (E) Presence of uric acid stones

The answer is C. Autophagic granules are intracytoplasmic vacuoles containing debris from degraded organelles, such as mitochondria. They are especially prominent in cells that have become atrophic, such as skeletal muscle cells after prolonged immobilization.

A 30-year-old woman dies following a prolonged period in a vegetative state that resulted from injuries sustained in a headon auto collision 5 years earlier. An expected autopsy finding would be (A) a cherry-red spot on the macula. (B) asbestos bodies in the lung. (C) autophagic granules in skeletal muscles. (D) Lewy bodies within neurons. (E) Mallory bodies within hepatocytes.

The answer is C. Severe diarrhea, fever, and toxicity following broad-spectrum antibiotic therapy is likely due to pseudomembranous colitis. This disorder is caused by overgrowth of Clostridium difficile,a commensal microorganism indigenous to the bowel, and is marked morphologically by superficial mucosal erosions with overlying necrotic, loosely adherent mucosal debris. The clostridia remain intraluminal, but secrete an enterotoxin that is responsible for the clinical and pathologic manifestations of the disorder.

A 56-year-old man who had been receiving intravenous antibiotics for severe cellulitis develops fever, toxicity, and severe diarrhea. This scenario suggests which of the following disorders? (A) Celiac sprue (B) CMV infection (C) Pseudomembranous colitis (D) Ulcerative colitis (E) Whipple disease

The answer is C. The description is that of the syndrome of inappropriate ADH secretion, which is excessive release of ADH, most commonly a manifestation of small cell bronchogenic carcinoma. There are numerous other causes, including a variety of tumors, CNS disorders, trauma, and infections. The cardinal features are marked decreases in both serum sodium and osmolality, normal urine sodium, and urine osmolality considerably exceeding that of the serum.

A 36-year-old man is brought to the emergency department by his wife because of lethargy, weakness, and confusion. Serum sodium and serum osmolality are markedly decreased. Urine osmolality is increased. These findings are most likely related to (A) adenoma of the anterior pituitary. (B) adenoma of the posterior pituitary. (C) bronchogenic carcinoma. (D) diabetes insipidus. (E) Sheehan syndrome.

The answer is B. The figure demonstrates the typical appearance of micronodular cirrhosis, the most common cause of which is alcoholism. Major clinical manifestations include jaundice, ascites, signs of hyperestrinism (palmar erythema, spider telangiectasia, gynecomastia, testicular atrophy), consequences of increased portal venous pressure (esophageal varices, distended abdominal veins [caput medusae], splenomegaly), and consequences of hypoalbuminemia (ascites, peripheral edema).

A 55-year-old alcoholic man died after an illness characterized by increasing jaundice, ascites, and generalized wasting. Laboratory testing revealed hyperbilirubinemia, hypoalbuminemia, and mildly elevated liver enzymes. The appearance of the liver at autopsy is shown in the figure. The most likely diagnosis is (A) alpha-1-antitrypsin deficiency. (B) cirrhosis. (C) hepatitis A. (D) hepatitis C. (E) multiple metastases.

The answer is C. Seminoma is the most common germ cell tumor of the testis. Serum hCG levels, elaborated by syncytiotrophoblasts, are elevated in about 15% of cases, but these elevations are not as high as those seen in choriocarcinoma. The tumor is highly radiosensitive and often curable, even when metastatic. Dysgerminoma represents the ovarian counterpart of seminoma. Embryonal carcinoma is highly pleomorphic and is much less responsive to therapy than seminoma. Yolk sac tumor is the most common tumor of the testis during infancy and early childhood and is usually accompanied by an increase in serum α-fetoprotein.

A 36-year-old man presents to his primary care physician complaining of painless enlargement of the testis. Further laboratory studies reveal an increase in serum hCG. Of the following, which is the most likely diagnosis? (A) Dysgerminoma (B) Embryonal carcinoma (C) Seminoma (D) Teratoma (E) Yolk sac tumor

The answer is E. An increased incidence of berry aneurysm is associated with adult polycystic kidney. Berry aneurysm occurs at sites of discontinuity of the arterial media, most frequently at bifurcations of vessels of the circle of Willis. The most common locations are the junction of the anterior cerebral and anterior communicating arteries, the bifurcation of the middle cerebral artery, the junction of the internal carotid and posterior communicating arteries, and the junction of the basilar and posterior cerebral arteries. Berry aneurysm is the most frequent cause of subarachnoid hemorrhage, and there is no association with atherosclerosis.

A 36-year-old woman experiences a severe headache. Investigation including computed tomographic angiography (CTA) reveals a saccular outpouching of the anterior cerebral artery. Which of the following is a major characteristic of this type of vascular change? (A) Occurs anywhere within branches of internal carotid artery (B) Infrequent site of subarachnoid hemorrhage (C) Middle meningeal artery most frequent location (D) Usually a complication of severe atherosclerosis (E) Often associated with polycystic kidney

The answer is A. Henoch-Schönlein purpura is an IgA immune complex disease characterized by involvement of small vessels (venules, capillaries, arterioles) with multiple lesions, all about the same age, and is a form of hypersensitivity or leukocytoclastic vasculitis. The disorder may involve only the skin, presenting as palpable purpura, or it may involve a variety of other sites, including the glomeruli, gastrointestinal tract, lungs, or brain.

A 4-year-old boy is seen after he suddenly develops a fever, abdominal pain and tenderness, hematuria, and palpable purpuric skin lesions on his buttocks and the extensor surfaces of the arms and legs. The most likely diagnosis is (A) Henoch-Schönlein purpura. (B) idiopathic (immune) thrombocytopenic purpura (ITP). (C) Kawasaki disease. (D) polyarteritis nodosa. (E) thrombotic thrombocytopenic purpura (TTP).

The answer is B. Hypertrophic osteoarthropathy, manifesting as clubbing of the fingers and associated periostitis of the distal radius and ulna, is associated with chronic lung disease, cyanotic heart disease, and other systemic disorders.

A 45-year-old man with a history of chronic obstructive pulmonary disease (COPD) manifests marked clubbing and joint pain of the fingers. Radiographs demonstrate soft tissue swelling at the ends of his fingers and osteolysis of the terminal phalanges. Which of the following is the most likely diagnosis? (A) Ankylosing spondylitis (B) Hypertrophic osteoarthropathy (C) Osteoarthritis (D) Rheumatoid arthritis (E) Felty syndrome

The answer is C. Pancreatic enzymatic fat necrosis represents autodigestion by proteolytic and lipolytic enzymes released from damaged parenchymal cells of the pancreas. Fatty acids liberated by the digestion of fat form calcium soaps, a process referred to as saponification. The precipitated calcium in the soaps can be visualized by radiologic imaging.

A 45-year-old man with a long history of alcoholism presents with severe epigastric pain, nausea, vomiting, fever, and an increase in serum amylase. During a previous hospitalization for a similar episode, computed tomography scanning demonstrated calcifications in the pancreas. A diagnosis of acute pancreatitis superimposed on chronic pancreatitis was made. In this condition, which of the following types of necrosis is most characteristic? (A) Caseous (B) Coagulative (C) Enzymatic (D) Fibrinoid (E) Liquefactive

The answer is B. Alveolar hyaline membrane formation is a characteristic finding in ARDS. The common factor in ARDS is diffuse alveolar damage induced by a number of agents or conditions, one of which is septic shock. Other prominent causes include trauma, uremia, gastric aspiration, inhalation of chemical irritants, oxygen toxicity, Mycoplasmainfection, and the severe acute respiratory syndrome (SARS).

A 45-year-old woman dies several days after a partial small bowel resection for repair of a volvulus. The surgery had apparently gone well, but shortly afterward she developed intractable fever, hypotension, multiorgan failure, and marked respiratory distress. Just prior to death, chest radiographs showed complete "whiteout" of both lungs. At autopsy, both lungs were found to have collapsed or distended alveoli, many of which were lined with fibrin-rich hyaline membranes. The cause of these pulmonary findings is best characterized as (A) aspiration. (B) diffuse alveolar damage. (C) generalized atelectasis. (D) lobar consolidation. (E) pneumothorax.

The answer is B. The decreased size is due to restriction of the blood supply, one of the causes of atrophy. The increase in size of the opposite kidney is referred to as compensatory hypertrophy. Unilateral renal artery stenosis is a well-known cause of secondary hypertension. In this setting, increased renin excretion and stimulation of the renin-angiotensin system results in a form of hypertension that is potentially curable by surgical correction of the underlying vascular abnormality.

A 45-year-old woman is investigated for hypertension and is found to have enlargement of the left kidney. The right kidney is smaller than normal. Contrast studies reveal stenosis of the right renal artery. The size change in the right kidney is an example of which of the following adaptive changes? (A) Aplasia (B) Atrophy (C) Hyperplasia (D) Hypertrophy (E) Metaplasia

The answer is A. Cryptorchidism (undescended testis) predisposes to testicular atrophy and sterility. It is associated with an increased incidence of germ cell tumors of the testis, even if the testis is surgically moved from its ectopic location back to the scrotum. A hydrocele is a serous fluid collection in the scrotum. Orchitis (inflammation of the testis) can result from bacterial or viral infection. A varicocele results from dilation of the veins of the spermatic cord, and the term "bag of worms" aptly describes the abnormality.

A 5-year-old boy is brought to the pediatrician for a physical examination prior to beginning elementary school. On examination, the boy has only one palpable testis in the scrotum. Further examination reveals a palpable mass in the left inguinal region. This condition is referred to as (A) cryptorchidism. (B) hydrocele. (C) orchitis. (D) torsion of the spermatic cord. (E) varicocele.

The answer is B. The patient has secondary hemostatic bleeding, which is characteristic of disorders of the coagulation pathway. Relatively common coagulation pathway disorders include classic hemophilia (factor VIII deficiency) and Christmas disease (factor IX deficiency). Both are disorders of the intrinsic pathway of coagulation, and are clinically indistinguishable from one another except by specific factor assays. DIC is characterized by widespread thrombosis and hemorrhage with both primary and secondary hemostatic bleeding. The other choices listed are characterized by primary hemostatic bleeding, which is manifested by punctate cutaneous hemorrhages and oozing from mucosal surfaces.

A 5-year-old boy is seen because he has recurrent hemarthroses and a large painful hematoma involving the soft tissues of his right thigh. Given the following choices, which is the most likely cause of the bleeding? (A) Ascorbic acid deficiency (B) Christmas disease (C) DIC (D) Rocky Mountain spotted fever (E) Primary thrombocytopenia

The answer is B. Chronic pancreatitis causes fat malabsorption, because pancreatic lipase is required for fat digestion. Fat malabsorption leads to deficiency of the fat-soluble vitamins A, D, E, and K. Vitamin K is required in the synthesis of clotting factors II, VII, IX, and X as a cofactor for the conversion of glutamyl residues to γ-carboxyglutamates.

A 55-year-old woman with chronic pancreatitis undergoes coagulation screening tests before surgery. The PT and APTT are found to be prolonged. Given the following choices, which of the following is the most likely reason for the abnormal coagulation test results? (A) Congenital inherited bleeding disorder (B) Fat malabsorption and vitamin K deficiency (C) Glutamate deficiency due to impaired digestion of dietary protein (D) Nutritional vitamin C deficiency (E) Post-pancreatitic carcinoma of the pancreas

The answer is D. Portal-systemic venous shunting leads to encephalopathy in end-stage cirrhosis. It also leads to esophageal varices, rectal hemorrhoids, and distention of periumbilical venous collaterals. Hypoalbuminemia, increased hepatic lymph formation, increased portal venous pressure, and renal retention of sodium and water all contribute to the development of ascites, but have little to do with encephalopathy.

A 50-year-old chronic alcoholic with jaundice and ascites secondary to known cirrhosis becomes disoriented and confused. Asterixis (flapping tremor) can be demonstrated. Which of the following features of his disease is most closely related to the change in mental status? (A) Hypoalbuminemia (B) Increased hepatic lymph formation (C) Increased portal venous pressure (D) Portal-systemic venous shunting (E) Renal retention of sodium and water

The answer is D. This patient has UIp, which carries a dismal prognosis characterized by refractoriness to steroids and development of honeycomb lung resulting in death, often within 5 years of diagnosis. The key histologic feature is temporal heterogeneity in the fibrotic changes, whereas LIP shows chronologically uniform, diffuse fibrosis. In contrast to DIP, UIP is not related to smoking. It is not attributed to any environmental stimulus or pathogen.

A 50-year-old female presents with restrictive lung disease. She describes an aggressive clinical course with rapidly progressive shortness of breath over the last year. A lung biopsy reveals a patchy process characterized by temporally heterogeneous areas of fibrosis. Which of the following is most likely of her expected clinical course? (A) Symptoms should abate with smoking cessation and steroid treatment (B) Prognosis is relatively good with the majority of patients surviving at 10 years (C) Excellent prognosis following removal of environmental stimulus (D) Poor prognosis with development of honeycomb lung and death within 5 years (E) Symptoms should improve with antibiotic therapy

The answer is A. Congestive or dilated cardiomyopathy is the most common form of cardiomyopathy. It is characterized by four-chamber hypertrophy and dilation as well as right- and left-sided severe heart failure. In some cases, congestive (dilated) cardiomyopathy may be associated with alcoholism, thiamine deficiency, or prior myocarditis.

A 53-year-old woman presents with dyspnea on exertion, orthopnea, paroxysmal nocturnal dyspnea, edema in the legs and feet, and fatigue. She has no history of angina, other signs of coronary artery disease, hypertension, or valvular disease. Echocardiography reveals cardiomegaly, with four-chamber hypertrophy and dilation. Which of the following is the most likely diagnosis? (A) Congestive or dilated cardiomyopathy (B) Hypertrophic cardiomyopathy (C) Myocarditis (D) Restrictive cardiomyopathy

The answer is D. The clinical scenario depicts the classic findings of pellagra, or niacin deficiency, with diarrhea, dementia, and dermatitis. Niacin is synthesized from the essential amino acid tryptophan, which is particularly deficient in corn-based diets. Vitamin C deficiency results in scurvy. Folic acid deficiency often manifests with anemia. Decreased levels of homocysteine, an amino acid, have been associated with cardiovascular disease. Vitamin E deficiency is rare and can result in neurologic abnormalities.

A 54-year-old Native American living on a reservation in southwest Arizona presents to a clinic with impaired memory; diarrhea; and a rash on the face, neck, and dorsum of the hands. It is likely that this patient has a deficiency of which of the following nutrients? (A) Ascorbic acid (B) Folic acid (C) Homocysteine (D) Niacin (E) α-Tocopherol (vitamin E)

The answer is B. The description of clear vacuoles displacing intact nuclei to the periphery is characteristic of fatty change (steatosis) of the liver; however, clear intracytoplasmic vacuolization of hepatocytes may be due to accumulations of water or glycogen, and sometimes special stains are required for confirmation of the nature of the vacuoles. In industrialized countries, such as the United States, the most common cause of fatty change of the liver is alcoholism.

A 54-year-old carpenter was brought to the hospital by ambulance after he was involved in a high-speed automobile chase that terminated in a collision into the trunk of a tree. On admission to the hospital, his skin was cold and clammy, his pulse was rapid and thready, and his blood pressure was 60 systolic and 40 diastolic. His blood alcohol was 0.29 g/dL (presumptive level for drunkenness is 0.08). In spite of blood transfusions, he died during an emergency laparotomy that revealed a ruptured spleen and a slightly enlarged liver. Microscopic examination of the liver at autopsy revealed intracytoplasmic clear vacuoles displacing the intact nuclei of the hepatocytes to the periphery of the cells. Special stains will most likely demonstrate that the vacuolar material is (A) bilirubin. (B) fat. (C) glycogen. (D) hemosiderin. (E) water.

The answer is A. The majority of salivary gland tumors occur in the parotid, and most are pleomorphic adenomas. This tumor most often presents as a painless mass just anterior to the ear. Due to its proximity to the facial nerve, it is often not completely resectable and therefore tends to recur. Mumps infection can lead to parotid swelling that is often bilateral and painful. Horner syndrome is a consequence of some lung tumors, not of salivary gland tumors.

A 54-year-old man presents with a painless unilateral swelling just anterior to the ear. Biopsy of the mass suggests it is a pleomorphic adenoma, and he is scheduled for surgery to remove the mass. Which of the following is true of this tumor? (A) Often recurs (B) Is less prevalent than Warthin tumors of the salivary gland (C) Totally resectable (D) Results from mumps infection (E) Often results in Horner syndrome

The answer is D. The Zollinger-Ellison syndrome, characterized by markedly increased gastric acid production and intractable peptic ulcer, is caused by hypersecretion of a gastrin-producing islet cell tumor (gastrinoma).

A 54-year-old man presents with multiple recurrent peptic ulcers of the duodenum and the jejunum. Gastric acid secretion is refractory to proton pump inhibitors. These findings suggest an underlying (A) adenocarcinoma of the pancreas. (B) adenoma of the adrenal medulla. (C) carcinoid of the jejunum. (D) islet cell tumor of the pancreas. (E) pheochromocytoma.

The answer is D. The clinical presentation of a dissecting aortic aneurysm mimics that of a myocardial infarction; however, electrocardiographic changes and increased concentration of cardiac enzymes are notably absent. The mediastinum is often widened by radiographic examination. Although there is an association with hypertension and disorders of connective tissue (e.g., Marfan syndrome and Ehlers-Danlos syndrome), there is no association with atherosclerosis. The presentation exemplified by this scenario is a true surgical emergency!

A 65-year-old man who has a long history of hypertension presents to the emergency department with tearing chest pain that radiates to the back. An electrocardiogram is normal, as are cardiac enzymes. A "stat" chest radiograph demonstrates widening of the mediastinum. Which of the following is the most likely? (A) Arteriovenous fistula (B) Atherosclerotic aneurysm (C) Berry aneurysm (D) Dissecting aneurysm (E) Syphilitic aneurysm

The answer is C. The fat embolism syndrome occurs 2 to 3 days after severe fracture injury and includes progressive CNS dysfunction and severe respiratory insufficiency. Thrombocytopenia with petechial bleeding is common, and petechial hemorrhages can result from obstruction of the microvasculature by embolic fat droplets. Respiratory insufficiency may be due to injury to pulmonary microvessels with leakage of fluid into the alveoli, resulting in ARDS.

A 65-year-old woman fell and sustained a pelvic fracture. After a short period of rapidly progressive mental confusion and respiratory insufficiency, the woman died. Numerous conjunctival petechiae were noted. These abnormalities were most likely due to which one of the following conditions? (A) Epidural hematoma (B) Aspiration pneumonia (C) Fat embolization (D) Acute tubular necrosis (E) Saddle embolus occluding bifurcation of pulmonary arteries

The answer is D. The location of the mass in the upper outer quadrant and the patient's age of 65 years suggest that the breast lesion is a carcinoma. A breast mass in a postmenopausal patient is most often a carcinoma.

A 65-year-old woman is found to have a 1-cm mass in the upper outer quadrant of the left breast. The most likely cause is (A) fibrocystic disease. (B) acute mastitis. (C) fibroadenoma. (D) carcinoma. (E) Paget disease of the breast.

The answer is A. Renal amyloidosis can occur in both primary and secondary amyloidosis. In the latter, the most frequently occurring underlying illness is rheumatoid arthritis.

A 65-year-old woman with a longstanding history of severe rheumatoid arthritis presents with proteinuria, hypertension, edema, and hypoalbuminemia. Which of the following is the most likely diagnosis? (A) Renal amyloidosis (B) Diabetic nephropathy (C) Membranous glomerulonephritis (D) Minimal change disease (E) Poststreptococcal glomerulonephritis

The answer is D. Small cell carcinoma of the lung is the most aggressive type of bronchogenic carcinoma. The location of this cancer is usually central. This is an undifferentiated tumor with small round blue cells and is least likely to be cured by surgery because it is usually already metastatic at diagnosis. Associated paraneoplastic syndromes include secretion of adrenocorticotropic hormone and antidiuretic hormone.

A 65-year-old woman with a significant smoking history presents with cough and shortness of breath. Computed tomography of the chest reveals a central mass near the left mainstem bronchus. Biopsy of the mass is performed. Histologic examination reveals small round blue cells, and a diagnosis of small cell carcinoma is made. Which of the following is a frequent characteristic of this form of lung cancer? (A) Generally amenable to surgical cure at time of diagnosis (B) More common in women, and a less clear relation to smoking than other forms of lung cancer (C) Secretes a parathyroid-like hormone (D) Secretes either corticotrophin or antidiuretic hormone (E) Usually in a peripheral rather than in a central location

The answer is A. DHT is a major growth factor for prostatic tissue. It is derived from the conversion from testosterone by the action of the enzyme 5α-reductase, type 2. Pharmaceutical inhibition of this enzyme is useful in the medical management of BPH. Estrogens also play an indirect role by stimulating the production of DHT receptors. hCG and α-fetoprotein are serum tumor markers for testicular cancers and bear no relevance to BPH.

A 66-year-old man visits his family physician with complaints of urinary frequency, hesitancy, and dysuria. Digital rectal examination reveals an enlarged prostate, and the consistency is rubbery and nodular. Serum PSA is modestly increased. Which of the following is most closely related to the pathogenesis of the likely disorder described here? (A) DHT (B) Estrogen (C) α-fetoprotein (D) hCG (E) Testosterone

The answer is D. Turbulent blood flow over mechanical heart valves can cause shearing of red blood cells, resulting in fragmented cells termed schistocytes. The hemolysis can result in anemia and hyperbilirubinemia. Hereditary spherocytosis causes hemolytic anemia due to an intrinsic defect in the red blood cells. Dietary deficiencies do not cause fragmented red blood cells. Aplastic anemia would result in severe anemia with marked reticulocytopenia.

A 72-year-old man who has recently had an aortic valve replacement now presents with pallor and fatigue. The red blood cell count is decreased, and schistocytes are reported on examination of a peripheral blood smear. In addition, his indirect (unconjugated) bilirubin is significantly elevated. The cause of the anemia is likely (A) cold agglutinin disease. (B) dietary deficiency. (C) hereditary spherocytosis. (D) mechanical disruption of red cells. (E) paroxysmal nocturnal hemoglobinuria.

The answer is E. The negative predictive value is the probability that a woman with a negative test does nothave the disease in question. In this example, there are 590 women (560 TN [80% of 700] 130 FN [10% of 300]) who had a negative test result. The ratio of TN to all negatives (TN 1FN) is 560/590, or 95%.

A biotechnology company has just released a new blood screening test for ovarian cancer. The test is positive in 90% of all women with ovarian cancer and negative in 80% of women without ovarian cancer. The prevalence of ovarian cancer in the study population of 1,000 women is 30%. What is the negative predictive value of this test? (A) 32.5% (B) 66% (C) 80% (D) 90% (E) 95%

The answer is B. The positive predictive value is the probability that a woman with a positive test actually has the disease in question. In this example, there are 410 women (270 TP [90% of 300] 1140 FP [20% of 700]) who had a positive test result. The ratio of TP to all positives (TP 1FP) is 270/410, or 66%.

A biotechnology company has just released a new blood screening test for ovarian cancer. The test is positive in 90% of all women with ovarian cancer and negative in 80% of women without ovarian cancer. The prevalence of ovarian cancer in the study population of 1,000 women is 30%. What is the positive predictive value of this test? (A) 32.5% (B) 66% (C) 80% (D) 90% (E) 95%

The answer is D. Sensitivity is defined as the probability that a test will be positive in individuals who are already known to have disease (ovarian cancer) and is stated to be 90%.

A biotechnology company has just released a new blood screening test for ovarian cancer. The test is positive in 90% of all women with ovarian cancer and negative in 80% of women without ovarian cancer. The prevalence of ovarian cancer in the study population of 1,000 women is 30%. What is the sensitivity of this test? (A) 32.5% (B) 66% (C) 80% (D) 90% (E) 95%

The answer is C. Specificity is defined as the probability that a test will be negative in individuals who are already known to be healthy (no ovarian cancer) and is stated to be 80%.

A biotechnology company has just released a new blood screening test for ovarian cancer. The test is positive in 90% of all women with ovarian cancer and negative in 80% of women without ovarian cancer. The prevalence of ovarian cancer in the study population of 1,000 women is 30%. What is the specificity of this test? (A) 32.5% (B) 66% (C) 80% (D) 90% (E) 95%

The answer is C. The hallmark of the megaloblastic anemias is the finding of megaloblastic erythroid hyperplasia in the bone marrow; pernicious anemia is a megaloblastic anemia.

A bone marrow aspiration from a 65-year-old man with long-standing profound anemia shows megaloblastic erythroid hyperplasia. Which of the following is the most likely diagnosis? (A) Anemia of chronic disease (B) Pelger-Huet anomaly (C) Pernicious anemia (D) Homozygous hemoglobin E (E) Thalassemia

The answer is A. Calcification within a cystic ovarian tumor in a young woman is most characteristic of mature teratoma of the ovary, a benign lesion and the most frequently occurring ovarian tumor.

A cystic ovarian mass was palpated in a 23-year-old woman. If X-ray films revealed calcifications in the mass, which of the following would be most likely? (A) Teratoma (B) Brenner tumor (C) Mucinous cystadenocarcinoma (D) Krukenberg tumor (E) Choriocarcinoma

The answer is D. Acute hematogenous osteomyelitis occurs with peak incidence in children, most commonly affects the metaphyses of long bones, and is more common in boys. In the acute stage, pyogenic osteomyelitis often resolves with antibiotic therapy. If the disorder is allowed to progress to necrosis and sequestrum formation, surgical intervention is usually required.

A diagnosis of acute hematogenous osteomyelitis is made in a 5-year-old boy who had presented with the sudden onset of a high fever. He had been limping and had had erythema, edema, and pain around his right knee for several days. Which of the following is true of this condition? (A) It occurs with peak incidence in the elderly. (B) It most commonly affects the iliac crests. (C) Surgical incision and drainage is almost always required. (D) It is most commonly caused by Staphylococcus aureus. (E) It is more common in females.

The answer is C. The lesion shown in the figure is a well-differentiated squamous cell carcinoma demonstrating sheets of neoplastic epidermal cells with keratin "pearls," a very common skin tumor. There is a marked predilection for sun-exposed areas, and most lesions occur on the lower part of the face or the back of the hands. Metastasis occurs in fewer than 5% of cases, because most of these lesions are discovered early and are cured by ablative therapy.

A scaling, ulcerated lesion develops on the forearm of a 45-year-old fisherman. Excisional biopsy is performed, and the histologic appearance is similar to that shown in the figure. Which of the following is most applicable to this lesion? (A) Distal metastases almost always occur (B) Indicative of an underlying visceral malignancy (C) Predilection for sun-exposed areas (D) Uncommon skin tumor

The answer is B. Koilocytes are indicative of HPV-infected epithelial cells. Viral infection causes intracytoplasmic vacuolation that is apparent in cytopathologic and histopathologic preparations. Donovan bodies represent macrophages stuffed with numerous C.(Donovania) granulomatisorganisms. Clear cells are characteristic of clear cell adenocarcinoma of the vagina. Paget cells are characteristic large cells surrounded by a clear, halo-like area that occur in Paget disease of the vulva and Paget disease of the breast. Fibroids are the most common benign tumors of the female reproductive tract.

An 18-year-old woman presents for follow-up of an abnormal Pap smear that revealed an abnormality suggestive of HPV infection. Which of the following was the likely cytopathologic finding? (A) Donovan bodies (B) Koilocytes (C) Clear cells (D) Paget cells (E) Fibroids

The answer is C. The clinical description is that of cystitis, which is characterized by pyuria and hematuria but with no white cell casts in the urine. Patients with acute pyelonephritis present with fever, leukocytosis, flank tenderness, urinary white cells, and white cell casts in the urine. Chronic pyelonephritis is almost always the result of chronic urinary tract obstruction and repeated bouts of acute inflammation in the kidneys.

An 18-year-old woman presents with suprapubic pain, urinary frequency, dysuria, and hematuria for the past hour. Urine tests show the presence of pyuria but no white cell casts. Physical examination is remarkable only for suprapubic tenderness on palpation. Which of the following is the most likely diagnosis? (A) Acute pyelonephritis (B) Chronic pyelonephritis (C) Cystitis (D) Fanconi syndrome (E) Nephrocalcinosis

The answer is C. All of the findings listed are characteristic of diabetes mellitus, but only insulitis is specific for type 1 diabetes. Amylin deposition in the pancreatic islets, derived from insulin-associated polypeptide, is found especially in type 2 diabetes mellitus. Armanni-Ebstein lesions (deposition of glycogen in renal tubules) are seen in uncontrolled hyperglycemia, which can occur in either type. Glomerular Kimmelstiel-Wilson nodules are seen in long-standing diabetes, regardless of type. Similarly, proliferative retinopathy is a complication of both forms of diabetes.

An autopsy is performed on an 8-yearold child with diabetes mellitus of recent onset who has died en route to the hospital following an automobile accident. Which of the following autopsy findings would favor the diagnosis of type 1 diabetes as contrasted to type 2 diabetes? (A) Amylin deposition in pancreatic islets (B) Armanni-Ebstein lesion (C) Insulitis (D) Kimmelstiel-Wilson nodules (E) Proliferative retinopathy

The answer is A. The illustration demonstrates diverticulosis of the colon (openings shown by arrows). These lesions are most common in older persons and are found most often in the sigmoid. The incidence of disease is increased in populations that consume low-fiber diets. Although most often asymptomatic, diverticula may become the site of acute inflammation (diverticulitis), sometimes with life-threatening complications, such as perforation and peritonitis.

An elderly woman with chronic constipation dies of a stroke and comes to autopsy. The figure illustrates a portion of her colon. The lesions shown in the figure (A) can be complicated by inflammation, perforation, and peritonitis. (B) are most likely related to a high-fiber diet. (C) most frequently occur high on the right side of the colon. (D) occur most often in teenagers.

The answer is B. Both folate and vitamin B12 deficiency lead to megaloblastic anemia, secondary to impaired DNA replication. In marked contrast to folate deficiency, vitamin B12 deficiency causes neurologic dysfunction associated with damage to the lateral and dorsal spinal columns. The history of gastric resection is consistent with a deficiency of intrinsic factor, which is required for absorption of vitamin B12 in the terminal ileum.

A 64-year-old man undergoes a total gastric resection for adenocarcinoma of the stomach. He has done well for 4 years but now presents with profound anemia, fatigue, and vague neurologic complaints. Position and vibration sensation are markedly diminished, and hyperreflexia is pronounced. Laboratory studies, including examination of the bone marrow, reveal pancytopenia and other findings compatible with a megaloblastic anemia. He is likely suffering a deficiency of which essential vitamin? (A) Folate (B) Vitamin B12 (C) Vitamin C (D) Vitamin D (E) Vitamin K

The answer is B. The term cor pulmonalerefers to right ventricular hypertrophy caused by pulmonary hypertension secondary to disorders of the lungs or pulmonary vessels. Other causes of right ventricular hypertrophy and failure, such as valvular disease, congenital defects, and left-sided heart failure, are precluded by this definition. Therefore, although in general, the most common cause of right-sided heart failure is left-sided heart failure, cor pulmonale with right-sided heart failure is due to an intrinsic disease originating in the lungs. Constrictive pericarditis can clinically mimic right-sided heart failure but is entirely unrelated to cor pulmonale.

A 64-year-old woman presents with dependent peripheral edema in her ankles and feet. She has long-standing chronic obstructive lung disease and a long history of cigarette smoking. Further investigation reveals that she has cor pulmonale with right-sided heart failure. Which of the following is the most likely cause of the right-sided heart failure in this patient? (A) Constrictive pericarditis (B) Disease of the lungs or pulmonary vessels (C) Left-sided heart failure (D) Pulmonary infundibular or valvular stenosis (E) Systemic hypertension

The answer is D. Folate deficiency leads to megaloblastic anemia. The red cells are macrocytic (increased MCV) and normochromic (increased MCHC). Since the thickened red cells appear denser on peripheral blood smears, these cells are often erroneously thought to be hyperchromic; however, the MCHC is normal. An increased MCHC is found in hereditary spherocytosis. Hypochromic erythrocytes are typical of iron deficiency anemia, some cases of the anemia of chronic disease, and the thalassemias. Suppressed β-chain synthesis is characteristic of the β-thalassemias. Decreased susceptibility to malaria is associated with absence of the Duffy blood group antigen and with G6PD deficiency.

A 65-year-old alcoholic is found to have megaloblastic anemia. He has no neurologic abnormalities. Serum and erythrocyte folate are markedly decreased, and serum vitamin B12 is normal. Which of the following is an expected characteristic of this form of anemia? (A) Decreased susceptibility to malaria (B) Hypochromic erythrocytes (C) Increased mean corpuscular hemoglobin concentration (MCHC) (D) Increased mean corpuscular volume (MCV) (E) Suppressed β-chain synthesis

The answer is B. Agnogenic (idiopathic) myeloid metaplasia is characterized by extensive non-neoplastic myelofibrosis and extramedullary hematopoiesis resulting in hepatosplenomegaly. Teardrop-shaped erythrocytes, as well as scattered nucleated red cells and granulocytic precursor cells, can be found in the peripheral blood smear. Although marrow myeloid (granulocytic) and erythroid precursor cells are depleted, megakaryocytes tend to be spared and even increased in number.

A 65-year-old man has anemia, splenomegaly, and extramedullary hematopoiesis. He has experienced easy fatigability, weight loss, and weakness. Bone marrow biopsy reveals marked proliferation of fibrous tissue (myelofibrosis) consistent with agnogenic myeloid metaplasia. Which of the following is a characteristic finding in this disorder? (A) Depletion of bone marrow megakaryocytes (B) Teardrop-shaped erythrocytes (C) Autosplenectomy (D) Neoplastic plasma cells in the bone marrow (E) Blast crisis in the peripheral blood

The answer is C. Carcinoma of the pancreas with common bile duct obstruction is strongly suggested by the clinical findings. Spontaneous migratory venous thrombosis with visceral neoplasms is known as the Trousseau sign or syndrome.

A 65-year-old man is evaluated for abdominal pain radiating through to the back, jaundice, anorexia, and recent weight loss. An additional likely finding is (A) history of thorium dioxide (Thorotrast) exposure. (B) increased AFP. (C) migratory venous thrombosis. (D) pancreatic calcification and pseudocyst formation. (E) urine test negative for bilirubin.

The answer is B. The vignette is suggestive of long-standing, late-stage hereditary hemochromatosis, and the expected findings would include markedly increased serum iron and moderately reduced TIBC. This combination often results in almost 100% saturation of iron-binding capacity.

A 65-year-old man is found to have the combination of increased skin pigmentation, cirrhosis of the liver, and diabetes mellitus. Which pattern of serum iron and total iron-binding capacity (TIBC) is most consistent with the familial illness suggested by these findings? (A) Serum iron increased, TIBC increased (B) Serum iron increased, TIBC decreased (C) Serum iron normal, TIBC increased (D) Serum iron decreased, TIBC increased (E) Serum iron decreased, TIBC decreased

The answer is A. Conjugated hyperbilirubinemia and positive urine tests for bilirubin are indicative of obstructive jaundice. The further finding of the complete absence of urine and stool urobilinogen indicates total common bile duct obstruction. Additionally, the palpable gallbladder (Courvoisier sign) strongly suggests that the etiology is a malignant tumor, such as adenocarcinoma of the head of the pancreas.

A 65-year-old man is seen for the recent onset of jaundice, weight loss, and anorexia. Abdominal examination reveals a distended, palpable gallbladder. Laboratory studies reveal conjugated hyperbilirubinemia, positive urine tests for bilirubin, and total absence of urobilinogen in the urine and stools. The probable diagnosis is (A) adenocarcinoma of the pancreas. (B) amebic abscess of the liver. (C) hepatic vein thrombosis. (D) hepatitis A infection. (E) hereditary spherocytosis.

The answer is A. Squamous cell carcinoma of the esophagus is an aggressive cancer with rapid progression and short survival in all stages of disease. It is most common in subjects with a long-term history of cigarette smoking and alcohol use. The tumor arises most commonly in the upper and middle thirds of the esophagus.

A 65-year-old man presents with dysphagia, weight loss, and anorexia. Physical examination is normal. Esophagogastroduodenoscopy with biopsy of an esophageal lesion is performed, revealing squamous cell carcinoma. Which of the following is true regarding this cancer? (A) Cigarette smoking and chronic alcohol use are associated risk factors. (B) Gastroesophageal reflux disease and Barrett esophagus are associated risk factors. (C) Histologic findings include disordered, back-to-back submucosal glands. (D) It most frequently arises in the lower third of the esophagus. (E) This cancer is characterized by an indolent course, and long survival is common.

The answer is D. Staging is based on clinical evaluation of the distribution and extent of the disease process and is contrasted with grading, which is based on histopathologic evaluation of a malignant neoplasm.

A 65-year-old man presents with recurrent fever and painless cervical and supraclavicular lymphadenopathy. Biopsy and further studies reveal that the patient has Hodgkin disease at a stage that is usually associated with a very poor prognosis. Staging is based on which of the following? (A) Cell of origin (B) Degree of anaplasia (C) Degree of differentiation of tumor cells (D) Distribution and extent of disease (E) Number of mitotic figures

The answer is B. An increase in PSA may occur in both nodular prostatic hyperplasia and in carcinoma of the prostate. Both of these disorders have a peak incidence in elderly men, and nodular prostatic hyperplasia tends to arise in the central zone of prostatic glands.

A 65-year-old man presents with urinary hesitancy, frequency, urgency, sensation of incomplete bladder emptying, and straining to start the urinary stream. Digital rectal examination is performed. Further workup reveals the diagnosis of nodular prostatic hyperplasia. Which of the following is an association of this disorder? (A) Tends to arise in the peripheral zone of the prostatic glands (B) An increase in serum prostate-specific antigen (PSA) may occur (C) Low incidence in older men (D) May frequently progress to bony osteoblastic metastases (E) Often improves over time without intervention

The answer is B. Specificity is defined as the ability of a test to be negative in healthy persons; it is the percentage of known negatives who actually test negative. It is calculated by dividing the number of negative results in healthy persons (TN) by the total number of healthy persons (TN 1FP).

Conversely, the same laboratory has at its disposal 1,000 similarly preserved serum samples from normal subjects who are known not to have had ovarian cancer. The ability of the new procedure to correctly identify these individuals as normal is referred to as (A) sensitivity. (B) specificity. (C) precision. (D) accuracy. (E) predictive value.

The answer is D. Characteristic electron-dense "humps" on the epithelial side of the basement membrane (subepithelial location) are an extremely important diagnostic feature of poststreptococcal glomerulonephritis. The basement membrane is not thickened in this acute, usually self-limited disorder.

Expected findings on electron microscopic examination of the glomerulus from the patient in question 4 demonstrate (A) marked subendothelial immune complex deposition. (B) marked thickening of the glomerular basement membrane with numerous intramembranous and epimembranous (subepithelial) immune complex deposits. (C) no changes except for fused epithelial foot processes. (D) normal-appearing glomerular basement membrane with electron-dense "humps" in subepithelial location. (E) striking increase in thickness of glomerular basement membrane and diffuse increase in mesangial matrix material.

The answer is C. A linear pattern of glomerular immunofluorescence for IgG is found in Goodpasture syndrome, which is caused by antibodies that react with both glomerular and alveolar basement membranes.

In a 44-year-old man with hemoptysis and hematuria, a linear pattern of glomerular immunofluorescence for IgG is observed in a renal biopsy. The most likely associated laboratory finding is a positive test for antibodies directed to (A) streptolysin O. (B) C3 convertase. (C) glomerular basement membranes. (D) hepatitis B virus. (E) Sm (Smith) nuclear antigen.

The answer is D. Mutations in EGFRand PTENare associated with primary GBM, whereas p53mutations are frequently observed in secondary GBMs. MGMTpromoter methylation is not specifically linked to either primary or secondary GBM, but it predicts better response to treatment in GBMs because tumors cannot repair damage caused by alkylating agents. 1p19q translocations are typical of oligodendrogliomas. Dual loss of 1p and 19q can also be seen in astrocytomas and is generally a marker of good prognosis in these tumors. NF2mutations are common in meningiomas.

In the tumor described in question 12, which of the following molecular findings would support a primary, rather than secondary, tumor origin? (A) p53mutation (B) MGMTpromoter methylation (C) 1p19q translocation (D) EGFRmutation (E) NF2mutation

The answer is D. The clinical behavior of neoplasms is the underlying basis for all other indicators that distinguish malignant from benign lesions. For example, some extremely well-differentiated, otherwise benign-appearing lesions are known to metastasize and, thus, are classified as malignant.

$ A 70-year-old man is found to have a papillomatous neoplasm of the bladder. Even though the lesion is extremely well-differentiated, the pathologist makes a diagnosis of transitional cell carcinoma grade I (urothelial neoplasm of low malignant potential). The distinction of "malignant" from "benign" in this instance was most likely based on which of the following known general characteristics of this type of lesion? (A) Appearance of oncofetal antigens (B) Chromosomal aneuploidy (C) Loss of contact inhibition in tissue culture (D) Clinical behavior (E) Ultrastructural alterations

The answer is A. Congenital pyloric stenosis is an obstruction of the gastric outlet caused by hypertrophy of the pyloric muscularis. The hypertrophic muscle is often perceived as a palpable mass. The principal manifestation of this condition, more common in boys, is projectile vomiting, most often occurring in the first 3 to 6 weeks of life.

A 10-day-old boy with projectile vomiting and a palpable midepigastric mass most likely has (A) congenital pyloric stenosis. (B) infantile polycystic kidney. (C) intussusception. (D) tracheoesophageal fistula. (E) Wilms tumor.

The answer is C. The clinical history is strongly suggestive of pyogenic meningitis, the most common form of meningitis in the newborn. The diagnosis is confirmed by abnormalities in the CSF. These findings include increased pressure, cloudy appearance, markedly increased cell count with numerous neutrophils, increased protein, and decreased glucose (a useful rule is that the normal CSF glucose concentration is about two-thirds that of the serum glucose).

A 10-day-old infant has high fever, nuchal rigidity, and photophobia. A lumbar puncture is performed to obtain cerebrospinal fluid (CSF) for analysis. Which of the following is the most likely CSF finding? (A) Decreased protein, decreased glucose (B) Decreased protein, increased glucose (C) Increased protein, decreased glucose (D) Increased protein, increased glucose (E) Normal protein, decreased glucose

The answer is B. Congenital pyloric stenosis is caused by hypertrophy of the circular muscular layer of the pylorus, resulting in a palpable mass in the epigastrium. Hypertrophy of the pyloric musculature leads to obstruction and the characteristic projectile vomiting. This condition most commonly occurs in male infants within the first several days to weeks of life.

A 10-day-old infant presents with projectile vomiting. His mother states that the infant will actively drink his milk, but he forcefully vomits after each feeding. The infant shows signs of failure to thrive, with weight loss, dehydration, and lethargy. Physical examination reveals a firm, nontender, mobile, "olive-shaped" epigastric mass. Which of the following is the most likely diagnosis? (A) Candidaesophagitis (B) Congenital pyloric stenosis (C) Esophageal cancer (D) GERD (E) Tracheoesophageal fistula

The answer is E. In a pediatric patient, the combination of recurrent pulmonary infections and steatorrhea (presumably due to pancreatic insufficiency) is strongly suggestive of cystic fibrosis. This disorder is characterized by a generalized defect in the reabsorption of anions, leading to increased sweat chloride concentration, an important diagnostic indicator.

A 10-month-old girl presents with recurrent pulmonary infections, steatorrhea, and failure to thrive. Measurement of which substance is the most appropriate procedure in this patient? (A) Erythrocyte glucose-6-phosphate dehydrogenase (G6PD) (B) Serum ceruloplasmin (C) Serum β-lipoprotein (D) Serum phenylalanine (E) Sweat chloride

The answer is C. The typical cytogenetic change associated with Burkitt lymphoma is t(8;14) with increased expression of the c-mycgene. This disorder is an aggressive B-cell non-Hodgkin lymphoma most commonly affecting children. The endemic (African) form is characterized by the involvement of the maxilla or mandible, whereas the sporadic (Western) form usually involves the abdominal organs. Burkitt lymphoma is generally a rapidly growing neoplasm, and the endemic form has a frequent association with EBV.

A 10-year-old boy presents with a large abdominal mass. Computed tomography of the abdomen reveals enlarged retroperitoneal and mesenteric lymph nodes. Biopsy of one of the involved lymph nodes shows a "starry-sky" appearance, with prominent debris- containing macrophages. A diagnosis of Burkitt lymphoma is made. Which of the following statements about this disorder is correct? (A) The sporadic (Western) form is most frequently associated with EBV. (B) The disorder is considered to be a derivative of Hodgkin lymphoma, lymphocyte depletion subtype. (C) The most common cytogenetic change is t(8;14), with increased expression of c-myc. (D) The tumor cells are derivatives of T lymphocytes. (E) The tumor most often has an indolent clinical course.

The answer is D. The history is that of a craniopharyngioma, a benign tumor that does not invade or metastasize; however, local effects of this tumor can be quite destructive, and recurrence due to incomplete resection is not uncommon. Local growth and tissue destruction result in both anterior and posterior pituitary dysfunction, and a patient often presents with signs of increased intracranial pressure, sometimes with hydrocephalus and frequently with bilateral hemianopsia (loss of peripheral visual fields) due to impingement on the optic chiasm. Diabetes insipidus is also frequent. Calcification apparent on radiograph is often prominent, facilitating diagnosis.

A 10-year-old boy presents with headache and bilateral hemianopsia, as well as evidence of increased intracranial pressure and diabetes insipidus. Suprasellar calcification is apparent on radiographic examination. Resection of the contents of the sella turcica and parasellar area yields a large tumor with histology closely resembling the enamel organ of the embryonic tooth. The most likely outcome of this lesion is (A) local invasion and intracranial metastasis. (B) hematogenous metastasis to distal sites. (C) lymphatic spread to distal sites. (D) possible local recurrence with continued pressure-related damage to adjacent structures.

The answer is E. This is a case of acute rheumatic fever. Acute rheumatic fever manifests most commonly in patients 5 to 15 years of age with migratory polyarthritis, pancarditis, subcutaneous nodules, erythema marginatum, and Sydenham chorea. Decades later, severe valvular disease, often manifesting as mitral stenosis, may develop as a feature of rheumatic heart disease. In this chronic stage of rheumatic disease, fibrotic valves may become stenotic, insufficient, or both, but much more commonly, progression to cardiac valve complication does not occur.

A 10-year-old boy presents with migratory polyarthritis involving several large joints, fever, and malaise. Physical examination reveals a new heart murmur and friction rub on auscultation, and a painless nodule is detected on the extensor surface of the elbow. He had a severe sore throat approximately 2 weeks ago, apparently recovering without antibiotic therapy. The anti- streptolysin O (ASO) titer is elevated. Which of the following describes the most likely outcome for this patient? (A) Development of mitral valve stenosis over many months to years (B) Development of mitral valve stenosis over the next few months (C) Increasing severity of the current symptoms and findings over the next few decades (D) Persistence of the current symptoms and signs over the patient's lifetime (E) Total recovery after 1 to 2 months with no further complications or sequelae

The answer is D. A Spitz nevus is a pediatric skin neoplasm that can be easily confused with malignant melanoma based on gross and microscopic morphology. It was formerly called "juvenile melanoma" however this name has fallen out of favor because it evokes unnecessary alarm, given the typically benign course of these tumors. Acanthosis nigricans is sometimes an indicator of visceral malignancy. Actinic keratosis is a premalignant epidermal lesion. Dysplastic nevus may transform into malignant melanoma. Xeroderma pigmentosum is associated with a markedly increased incidence of skin cancer caused by failure of DNA repair.

A 10-year-old girl presents with an orange-red, dome-shaped papule on her right leg. Her parents state that the papule has grown from a tiny dot to over a centimeter in length in just a few months. Following biopsy, the dermatologist reassures the parents, stating that although the papule superficially resembles malignant melanoma, it is actually a benign lesion without malignant potential. Which of the following is the likely diagnosis? (A) Acanthosis nigricans (B) Actinic keratosis (C) Dysplastic nevus (D) Spitz nevus (E) Xeroderma pigmentosum

The answer is E. The prototype of the nephritic syndrome is poststreptococcal glomerulonephritis.

A 12-year-old boy presents with smoky brown-colored urine, oliguria, azotemia, and hypertension. After further tests, the patient is diagnosed with a nephritic syndrome. Which of the following is the most likely diagnosis? (A) Renal amyloidosis (B) Diabetic nephropathy (C) Membranous glomerulonephritis (D) Minimal change disease (E) Poststreptococcal glomerulonephritis

The answer is B. The clinical and histologic findings are consistent with lymphomatoid granulomatosis, an EBV-associated B cell neoplasm which is most commonly seen in pulmonary and CNS vessels in the setting of immunosuppression. Abundant reactive T cells are also seen, which led to early confusion about whether this entity primarily represented a B cell or T cell process. CMV is a nearly ubiquitous virus which is latent in over half of healthy adults but which may cause disease in immunosuppressed patients. However, it is not associated with lymphomatoid granulomatosis. HPV infection is associated with carcinogenesis in the cervix, oropharynx, and anus. HHV-8 is the driver behind Kaposi sarcoma. C. pneumoniais thought to contribute to atherogenesis.

A 12-year-old girl undergoes bone marrow transplant as part of therapy for ALL. She subsequently presents with pulmonary complaints and neurologic findings. Lung biopsy reveals a granulomatous vasculitis with associated atypical lymphocytes. Clonality studies reveal a monoclonal B cell population with associated polyclonal T cells. What infectious agent is related to this diagnosis? (A) CMV (B) EBV (C) HPV (D) HHV-8 (E) Chlamydia pneumonia

The answer is A. Wilson disease is a hereditary condition associated with the accumulation of copper in the liver, brain, and eye. Accumulation of copper in the Descemet membrane of the eye results in the pathognomonic lesion known as the Kayser-Fleischer ring. Accumulation in the liver results in cirrhosis. Accumulation in the brain, specifically in the basal ganglia, results in motor symptoms. Councilman bodies are apoptotic hepatocytes that were first identified in yellow fever. Eosinophilic hyaline inclusions, Mallory bodies, are seen in alcoholic liver disease. Glycogen accumulates in the liver in numerous glycogen storage diseases. Iron accumulates in hemochromatosis.

A 13-year-old boy presents to the pediatrician with extrapyramidal signs, including a resting and kinetic tremor. An ophthalmologic examination demonstrates the presence of Kayser-Fleischer rings, and his laboratory studies demonstrate elevated liver enzymes. It is likely that the patient's condition is associated with the accumulation of (A) copper. (B) Councilman bodies. (C) eosinophilic hyaline inclusions. (D) glycogen. (E) iron.

The answer is D. The hemorrhagic phenomena in this patient likely represent recurrent rupture of the convolutions of venules and capillaries in the mucous membranes of the nose, as occurs in the Osler-Weber-Rendu syndrome (hereditary hemorrhagic telangiectasia). This disorder is inherited as an autosomal dominant condition and also affects the vessels of the gastrointestinal tract. Characteristic lesions can often be seen on the lips, conjunctivae, and mucous membranes of the nose or mouth. Cavernous hemangioma is a common lesion. On occasion, it can be seen as a component of the von Hippel-Lindau syndrome. A glomangioma is a benign tumor of blood vessels on the fingers or toes. A hemangioendothelioma is a rare malignant tumor of blood vessels. Varicose veins are a manifestation of incompetency of the valves in the superficial veins of the legs of older patients.

A 14-year-old boy is brought by his family to your clinic in Utah with a long history of recurrent epistaxis. His father also reports such a history. Of the following, which is the most likely condition consistent with these findings? (A) Cavernous hemangioma (B) Glomangioma (C) Hemangioendothelioma (D) Osler-Weber-Rendu syndrome (E) Varicose veins

The answer is D. The clinical findings are suggestive of Addison disease (primary adrenocortical insufficiency). About 70% of cases are now due to autoimmune adrenalitis, but until recently the most frequent cause was tuberculosis. Hyperpigmentation in Addison disease results from compensatory hypothalamic production of proopiomelanocortin, the precursor peptide of both corticotropin and melanocyte-stimulating factor.

A 14-year-old boy is seen because of increasing weakness, easy fatigability, and weight loss over the past 3 months. In addition, he has recently developed nausea, vomiting, and abdominal pain. His blood pressure is markedly decreased, and he has increased pigmentation of his skin creases. These findings are suggestive of (A) Cushing syndrome. (B) secondary hyperaldosteronism. (C) osteitis fibrosa cystica. (D) Addison disease. (E) 1α-Hydroxylase deficiency.

The answer is E. Subacute sclerosing panencephalitis, one of the slow virus infections, is thought to be caused by persistent infection with a defective measles virus. The virus lacks the M component, a protein required for extracellular spread of the virus. This deficiency is thought to explain the slow nature of the infection.

A 14-year-old girl dies after an illness characterized by progressive motor and mental deterioration. At autopsy, there is profound cortical atrophy, loss of white matter, and ventricular enlargement. Special studies indicate participation of a defective measles virus. Which of the following is the diagnosis? (A) Creutzfeldt-Jakob disease (B) Guillain-Barré syndrome (C) Kuru (D) Progressive multifocal leukoencephalopathy (E) Subacute sclerosing panencephalitis

The answer is E. von Willebrand disease, a disorder transmitted by autosomal modes of inheritance (both dominant and recessive) is the most common hereditary bleeding disorder. There are many variants, all marked by either qualitative or quantitative deficiencies of vWF.

A 14-year-old girl presents with prolonged bleeding from wounds and minor trauma and severe menorrhagia. Family history reveals that her father also has prolonged bleeding from wounds and minor trauma, as does her brother. Which of the following is the most likely mechanism of this patient's disorder? (A) Absence of platelet glycoprotein IIb-IIIa (B) Antiplatelet antibodies reacting with platelet surface glycoproteins (C) Deficiency of factor VIII (D) Deficiency of factor IX (E) Deficiency of vWF

The answer is C. Repeated episodes of splenic infarction followed by fibrotic healing lead to a fibrotic, shrunken spleen (autosplenectomy) in adult patients with sickle cell anemia. The spleen is enlarged and congested in children with sickle cell anemia.

A 23-year-old African-American man with a history since early childhood of severe anemia requiring many transfusions has nonhealing leg ulcers and recurrent periods of abdominal and chest pain. The spleen of the patient in question would be expected to be (A) enlarged. (B) normal sized. (C) shrunken.

The answer is E. Pseudohypoparathyroidism is characterized by renal end-organ unresponsiveness to PTH and by shortened fourth and fifth metacarpals and metatarsals, short stature, and other skeletal abnormalities. These abnormalities are due to mutations in GNAS1, a G protein that mediates receptiveness to PTH and other hormones. Selective imprinting results in maternal inheritance of the end-organ unresponsiveness. The clinical findings mimic those of hypoparathyroidism, but PTH is most often normal or elevated.

A 16-year-old girl with short stature, rounded face, and shortening of the fourth and fifth metacarpals and metatarsals has hypocalcemia and hyperphosphatemia. She has multiple calcifications involving the basal ganglia, vasculature, and other sites. PTH varies from normal to increased, and administration of PTH does not result in phosphaturia. These findings are characteristic of which of the following? (A) Primary hyperparathyroidism (B) Secondary hyperparathyroidism (C) Tertiary hyperparathyroidism (D) Hypoparathyroidism (E) Pseudohypoparathyroidism

The answer is D. A keloid is a result of excessive production of collagenous fibrous tissue and is characterized by a tumorlike scar consisting of dense bundles of structurally abnormal collagen. Keloids have a marked tendency to recur after resection. Propensity to keloid formation is markedly increased in persons of African lineage.

A 17-year-old African-American girl recently had her ears pierced for the first time. She now presents to her primary care provider with the complaint of a large "tumor-like" growth in the immediate site of one of the piercings. This lesion is likely a (A) benign fibrous histiocytoma. (B) dermatofibrosarcoma protuberans. (C) fibroepithelial polyp. (D) keloid. (E) xanthoma.

The answer is A. Osteosarcoma is the most common primary malignant tumor of bone. The most common presentation is in adolescents, with pain, swelling, and occasionally pathologic fracture in the proximal tibia or distal femur (about the knee). The characteristic radiologic findings include the Codman triangle (lifting of the periosteum of the bone caused by the expanding tumor) and a spiculated "sunburst" pattern of growth.

A 17-year-old boy presents with pain and swelling about the left knee for the past month. He thought that this condition resulted from an old football injury and that it would resolve without incident. The pain, however, has persisted and is severe enough to cause him to limp. Radiographs of the knee demonstrate a lifting of the periosteum and a spiculated "sunburst" pattern in the distal femur. Which of the following is the most likely diagnosis? (A) Osteosarcoma (B) Chondrosarcoma (C) Ewing sarcoma (D) Giant cell tumor (E) Knee sprain

The answer is A. The scenario is strongly suggestive of infectious mononucleosis—a benign, self-limited lymphoproliferative disorder that is most often caused by EBV infection; however, the heterophil test for sheep red cell agglutinins is negative, and so-called heterophil-negative infectious mononucleosis is most often caused by CMV.

A 17-year-old girl develops an acute syndrome characterized by low-grade fever, lassitude, pharyngitis, generalized lymphadenopathy, and a palpable liver and spleen. A peripheral blood smear reveals the presence of "atypical lymphocytes." A heterophil test for antibodies reacting with sheep erythrocytes is negative. Infection with which one of the following viruses is most likely? (A) Cytomegalovirus (CMV) (B) Epstein-Barr virus (EBV) (C) Measles virus (D) Papillomavirus (E) Parvovirus

The answer is A. Adult polycystic kidney is frequently associated with berry aneurysm of the circle of Willis, often in association with cysts in the liver or pancreas.

A 19-year-old boy is seen because of bilateral enlargement of the kidneys. His father, paternal uncle, and several other family members have similar abnormalities. Which of the following aneurysms is frequently associated with this disorder? (A) Berry aneurysm of the circle of Willis (B) Dissecting aneurysm (C) Fusiform aneurysm of the abdominal aorta (D) Saccular aneurysm of the thoracic aorta

The answer is B. Raynaud disease is cold-induced vasospasm of arterioles and small arteries, most often involving the fingers and sometimes the hands and feet. Young, otherwise healthy women are most often affected.

A 25-year-old woman is seen because of numbness and pain in her fingers and hands on exposure to cold. When either her hands or feet are very cold, they turn white and then blue. These changes are characteristic of which of the following? (A) Buerger disease (thromboangiitis obliterans) (B) Raynaud disease (C) Wegener granulomatosis (D) Kawasaki disease (E) Takayasu arteritis

The answer is D. Inflammation and stenosis of branches of the aortic arch is known as Takayasu arteritis, or "pulseless disease." It most commonly occurs in young Asian females. Buerger disease usually affects young Jewish males and involves the arteries of the extremities. The disease is exacerbated by smoking and can lead to gangrene of the extremities. Kawasaki disease affects the branches of the coronary arteries. Raynaud disease is due to vasospasm of small vessels of the fingers and toes, leading to cyanosis and pallor of the affected tissues. Temporal arteritis is usually encountered in older patients and affects the branches of the carotid artery, most commonly the temporal artery.

A 19-year-old young woman who emigrated from Taiwan 8 years ago presents with fever, malaise, myalgias, and arthritis and "coldness" in her upper extremities. She has a weak radial pulse bilaterally, and a magnetic resonance angiogram demonstrates nearly 75% stenosis of the main arteries originating from the aorta. She likely has which of the following rheumatologic conditions? (A) Buerger disease (B) Kawasaki disease (C) Raynaud disease (D) Takayasu arteritis (E) Temporal arteritis

The answer is A. Osteogenesis imperfecta (or brittle bone disease) is an autosomal dominant disorder characterized by multiple fractures with minimal trauma. It is caused by mutations in either of the genes (COL1A1 or COL1A2) that code for type I collagen and is manifested by connective tissue abnormalities affecting the bones, teeth, skin, and eyes. The sclerae can appear blue due to translucency of the thin connective tissue overlying the choroid. The disorder occurs in several variants defined by the age of onset and the severity of the fractures. In the less obvious cases, the principal differential diagnostic consideration is child abuse.

A 2-year-old boy presents with his third bone fracture within the past several months. There is no history or evidence of trauma. Several close family members have been similarly affected. The child is small for his age, and the sclerae are tinged a bluish color. Radiographs reveal generalized osteopenia and evidence of multiple fractures, both old and new. Which of the following is the usual mode of inheritance of this disorder? (A) Autosomal dominant (B) Autosomal recessive (C) X-linked dominant (D) X-linked recessive (E) Mitochondrial

The answer is A. von Hippel-Lindau disease is an autosomal dominant disorder characterized by multiple vascular tumors and multiple cysts of the liver, kidney, and pancreas. The renal cysts have a high potential for malignant transformation. Retinal and CNS hemangioblastomas are characteristic, as are pheochromocytomas.

A 20-year-old woman with episodic hypertension is found to have a marked increase in urinary VMA. Several close relatives have had vascular tumors of the eye and cysts of the liver, kidney, and pancreas. Which of the following familial syndromes is most likely? (A) von Hippel-Lindau disease (B) von Recklinghausen disease (C) Marfan syndrome (D) Familial hypercholesterolemia (E) Tuberous sclerosis

The answer is B. Associations of Wilms tumor notably include gene deletions localized to the short arm of chromosome 11 (11p13). In some instances, a "two-hit" mechanism of cancer suppressor gene inactivation, similar to that postulated for retinoblastoma, appears to be operative. Other associations of Wilms tumor include congenital aniridia, genitourinary malformations, and mental-motor retardation (the AGR or WAGR complex). Another unusual association is "hemihypertrophy" (gross asymmetry of the body) as part of the Beckwith-Wiedemann syndrome; yet another is the Denys-Drash syndrome, which is characterized by intersexual disorders, nephropathy, and Wilms tumor.

A 2-year-old boy with visible abdominal distention is found to have an enormous left-sided flank mass apparently arising from, but dwarfing, the left kidney. A syndrome that includes the lesion found in the patient in question has which of the following additional characteristics? (A) Berry aneurysm of the circle of Willis (B) Hemihypertrophy (C) Increased serum uric acid (D) Marked amplification of genes on the short arm of chromosome 11 (E) Spontaneous regression

The answer is E. Wilms tumor is the most common renal neoplasm of children. The tumors are often huge and frequently cause abdominal distention. Renal cell carcinoma and adult polycystic kidney can also present as a large flank mass, but not in a child.

A 2-year-old boy with visible abdominal distention is found to have an enormous left-sided flank mass apparently arising from, but dwarfing, the left kidney. The most likely diagnosis is (A) angiomyolipoma. (B) polycystic kidney. (C) renal cell carcinoma. (D) transitional cell carcinoma. (E) Wilms tumor.

The answer is D. Pyogenic meningitis in older infants and young children is most frequently caused by S. pneumoniaeor N. meningitidis.In newborns, the most likely agents are group B streptococci, E. coli,and Listeria; in young adults, the most frequent agent is N. meningitidis.In older adults, especially those with impaired resistance to infection, the most common etiologic agents are S. pneumoniaeand gram-negative rods.

A 2-year-old child presents with fever, headache, prostration, and nuchal rigidity. The CSF is cloudy, and microscopic examination reveals innumerable neutrophils. The CSF protein is increased, and glucose is decreased. The most likely etiologic agent is (A) Escherichia coli. (B) Haemophilus influenzae. (C) group B streptococci. (D) Streptococcus pneumoniae. (E) Staphylococcus aureus.

The answer is E. Repeated pulmonary infections and a positive sweat test are characteristic of cystic fibrosis. In this condition, viscid secretions cause defective exocrine gland function. The lungs and pancreas are the most significant sites of involvement, and the disorder is marked by repeated bouts of pneumonia and by pancreatic failure with wasting and steatorrhea.

A 2-year-old girl with a history of repeated pulmonary infections is found to have elevated chloride in a sweat test. An additional expected finding is (A) hypercalcemia. (B) hypotension. (C) increased metabolism. (D) renal failure. (E) steatorrhea.

The answer is B. The jaundice of hemolytic anemia is due to unconjugated hyperbilirubinemia. Because unconjugated bilirubin is not excreted into the urine, the type of jaundice is acholuric, jaundice without bilirubin pigment in the urine. In hemolytic anemia, haptoglobin is markedly decreased. Peripheral red cell destruction is mirrored by marrow erythroid hyperplasia with release of newly formed red cells into the peripheral blood, manifest as reticulocytosis.

A 20-year-old man is found to have hemolytic anemia with jaundice and splenomegaly. A younger brother is found to be similarly affected, and his mother had had a history of splenectomy. Which of the following abnormalities is an expected finding in this patient? (A) Increased haptoglobin (B) Unconjugated hyperbilirubinemia (C) Increased urine bilirubin (D) Decreased reticulocytes (E) Marrow erythroid hypoplasia

The answer is A. The findings are characteristic of meningococcemia with meningococcal meningitis. A well-recognized complication of meningococcemia is the WaterhouseFriderichsen syndrome, which is catastrophic adrenal insufficiency and vascular collapse caused by hemorrhagic necrosis of the adrenal cortex, often with associated DIC.

A 20-year-old man is hospitalized with fever, shaking chills, and widespread cutaneous hemorrhages. He complains of severe headache, and nuchal rigidity is noted on physical examination. Examination of the peripheral blood and CSF reveals gramnegative diplococci within neutrophils. A well-known complication of this disorder is hemorrhage into the (A) adrenal cortex. (B) anterior pituitary. (C) brain stem. (D) pancreas. (E) subarachnoid space.

The answer is A. This patient has synovial sarcoma, a poorly named soft tissue malignancy which does not appear to be related to the synovium. It is commonly biphasic with both spindled and epithelial contributions demonstrable on both light microscopy and immunohistochemistry. Although monophasic cases do not show obvious epithelial differentiation on H&E staining, scattered cytokeratin-positive cells may be seen on immunohistochemistry, suggesting that there is divergent epithelial differentiation in these cases too. The characteristic translocation involves X;18. While demonstration of t(X;18) is required to confirm a diagnosis of synovial sarcoma, it is not entirely specific for this entity.

A 20-year-old man presents with a soft tissue tumor of the lower leg. The tumor closely approximates the knee joint but does not appear to arise within or communicate with the synovium. Microscopy reveals a spindled-cell proliferation with admixed epithelial components, and molecular testing shows an X;18 translocation. The diagnosis is (A) synovial sarcoma. (B) undifferentiated pleomorphic sarcoma. (C) liposarcoma. (D) rhabdomyosarcoma. (E) fibrous histiocytoma.

The answer is C. The inflamed appendix in acute appendicitis should be surgically removed because of possible devastating complications of perforation or abscess.

A 20-year-old man presents with severe right lower quadrant abdominal pain, nausea, and anorexia. He states that the abdominal pain started around his umbilicus and has now migrated to the right lower quadrant of his abdomen. Physical examination reveals exquisite tenderness at McBurney point (the point one-third of the distance along the line from the right anterior superior iliac spine to the umbilicus). This patient is diagnosed with acute appendicitis. Which of the following is the treatment for this condition? (A) Antibiotics only, because the appendix is crucial for survival (B) Surgical resection of the appendix, because appendicitis can lead to appendiceal cancer (C) Surgical resection of the appendix, because appendicitis can lead to perforation or abscess (D) "Watch-and-wait" approach over days to see if inflammation subsides

The answer is B. Psoriasis is a chronic inflammatory skin disease characterized by erythematous plaques covered with a silvery scale. Histologic findings include epidermal proliferation with acanthosis, parakeratosis, and Munro abscesses (minute neutrophilic abscesses). Psoriasis is sometimes associated with a rheumatoid arthritis-like condition termed psoriatic arthritis.

A 20-year-old woman presents with a skin rash. The rash is localized to the extensor surfaces of her elbows and knees, and is composed of multiple well-demarcated circular-to-oval-shaped plaques and papules covered by a silvery scale overlying reddened erythematous skin. The silvery scale can be scraped off, revealing inflamed skin underneath. She states that several family members have a similar rash. Which of the following is the most likely diagnosis? (A) Neurodermatitis (lichen simplex chronicus) (B) Psoriasis (C) Pemphigus vulgaris (D) Bullous pemphigoid (E) Dermatitis herpetiformis

The answer is B. Fibroadenoma is a benign tumor most often presenting as a single discrete, freely movable lesion, often demonstrating a pattern of compressed glands and young fibrous stroma similar to that shown in the illustration. Fibroadenoma is the most frequent cause of a palpable mass in the breast in women younger than 25 years of age.

A 20-year-old woman presents with a solitary discrete, freely movable, firm, rubbery, nontender, well-circumscribed breast lesion. On resection biopsy, the lesion appears similar to that shown in the figure. The most likely diagnosis is (A) fibrocystic disease. (B) fibroadenoma. (C) medullary carcinoma. (D) colloid carcinoma. (E) intraductal carcinoma.

The answer is C. Vitiligo is an acquired loss of melanocytes in discrete areas of skin that appear as depigmented white patches. Vitiligo has no relationship to albinism. It is sometimes associated with autoimmune disorders, such as Graves disease, Addison disease, and antimelanocyte antibodies.

A 20-year-old woman presents with depigmented white patches of skin on the face, neck, and hands. She has a past history of Graves disease. Which of the following is the most likely diagnosis? (A) Ocular albinism (B) Oculocutaneous albinism (C) Vitiligo (D) Freckle (E) Verruca vulgaris (common wart)

The answer is B. The diffuse thickening of the walls of the capillary loops seen in the illustration is characteristic of membranous glomerulonephritis, the most frequent cause of the nephrotic syndrome in young adults. Expected associated findings in this immune complex disease include granular immunofluorescence and a characteristic "spike and dome" appearance that is best visualized with special stains. Intramembranous and epimembranous (subepithelial) electron-dense immune complex deposits are seen by electron microscopy.

A 20-year-old woman with the nephrotic syndrome and slowly progressive impairment of renal function marked by azotemia undergoes a renal biopsy. The patient's response to corticosteroid medication has been unimpressive. The appearance of the biopsy is similar to that shown in the figure. The most likely diagnosis is (A) focal segmental glomerulosclerosis. (B) membranous glomerulonephritis. (C) minimal change disease. (D) poststreptococcal glomerulonephritis. (E) rapidly progressive glomerulonephritis.

The answer is E. Hypertrophic cardiomyopathy, a condition that is usually inherited as an autosomal dominant disorder, is often associated with sudden death in young athletes. The ventricular septum is especially involved, with protrusion into the left ventricular cavity (asymmetric septal hypertrophy), sometimes leading to left ventricular outflow tract obstruction.

A 21-year-old basketball player died suddenly during a game. Autopsy revealed hypertrophy of the left ventricular wall, especially of the ventricular septum. Histologically, the myocardial fibers were arranged in a disorganized pattern. Which of the following best characterizes this disorder? (A) Can be a manifestation of primary amyloidosis (B) Can be a result of myocarditis (C) Is often associated with alcohol abuse (D) Is often associated with coronary artery disease (E) Often demonstrates autosomal dominant inheritance

The answer is B. The overall prognosis in SLE is most closely related to the glomerular lesions in affected patients. These renal lesions are highly variable. In the diffuse proliferative form, almost all of the glomeruli are involved in a marked inflammatory reaction to widespread subendothelial and mesangial immune complex deposition.

A 22-year-old woman presents with fever, malaise, generalized arthralgias, and a skin rash over the nose and malar eminences. Which one of the following possible findings has the greatest relative significance in the overall prognosis for the patient? (A) Atypical verrucous vegetations of the mitral valve (B) Glomerular subendothelial immune complex deposition (C) Immune complexes at the dermalepidermal junction in skin (D) Perivascular fibrosis in the spleen (E) Pleuritis

The answer is E. Priapism is an intractable, often painful erection associated with conditions such as sickle cell anemia, hypercoagulable states, spinal injuries, and some drugs. Balanitis is associated with poor hygiene and results from inflammation of the glans penis. Hypospadias is an anatomical anomaly wherein the urethral meatus opens on the ventral side of the penis. Peyronie disease results from subcutaneous fibrosis of the dorsum of the penis. Phimosis is an abnormally tight foreskin that is difficult or impossible to retract over the glans penis.

A 23-year-old African-American man who is known to have sickle cell anemia presents to the emergency department with a painful erection. The patient explains that the erection had started 3 hours ago. This condition is referred to as (A) balanitis. (B) hypospadias. (C) Peyronie disease. (D) phimosis. (E) priapism.

The answer is D. Spherocytes are present on the peripheral blood smear and, along with the history, strongly suggest a diagnosis of hereditary spherocytosis. Similar cells are also observed in warm antibody autoimmune hemolytic anemia; therefore, these two conditions must be distinguished by means of the direct Coombs test, which is negative in hereditary spherocytosis and positive in warm antibody autoimmune hemolytic anemia. An expected finding would be an increase in indirect (unconjugated) serum bilirubin, not direct (conjugated). The jaundice is acholuric (no bilirubin in the urine, so bilirubinuria would not be expected). The MCHC is often increased. Because hereditary spherocytosis is a normocytic anemia, the mean corpuscular volume is not decreased. Polychromatophilic erythrocytes are an expected finding, as in any hemolytic anemia.

A 23-year-old man of northern European lineage presents with anemia. His father and paternal aunt had a similar illness that was treated successfully by splenectomy. His peripheral blood smear is similar to that shown in the illustration. Which of the following additional abnormalities is expected? (A) Bilirubinuria (B) Decreased mean corpuscular volume (C) Increased direct (conjugated) serum bilirubin (D) Polychromatophilic erythrocytes on peripheral blood smear (E) Positive DAT (Coombs) test

The answer is A. Acute cold agglutinin disease is a form of hemolytic anemia due to autoantibodies to blood group antigens and is sometimes a complication of Mycoplasma pneumoniaeinfection. Aplastic anemia is associated with a variety of toxic exposures, including, among others, the antibiotic chloramphenicol, not azithromycin. Aplastic crises can occur as acute events in the course of hemolytic anemias, such as hereditary spherocytosis or sickle cell anemia. Paroxysmal nocturnal hemoglobinuria is an acquired defect that renders red blood cells sensitive to complement-induced lysis. Warm antibody autoimmune hemolytic anemia can be idiopathic or secondary to autoimmune disorders or lymphoid neoplasms.

A 23-year-old man presented to his primary care physician 2 weeks ago with a nonproductive cough and malaise. He was treated with the antibiotic azithromycin for "walking pneumonia" due to Mycoplasma pneumoniae. He now returns with fatigue and pallor. Laboratory studies demonstrate a decreased red blood cell count with polychromatophilia and an increase in indirect (unconjugated) bilirubin. The laboratory findings are most likely due to (A) acute cold agglutinin disease. (B) aplastic anemia. (C) aplastic crisis. (D) paroxysmal nocturnal hemoglobinuria. (E) warm antibody autoimmune hemolytic anemia.

The answer is C. Although the significance remains unclear, 30% of patients with polyarteritis nodosa have serum antibodies to the hepatitis B virus. Arsenic is associated with the development of angiosarcoma. Chlamydia pneumonia has been implicated in the development of atherosclerosis. KSHV, HHV-8, causes Kaposi sarcoma in individuals infected with human immunodeficiency virus.

A 23-year-old man presents with fever, weight loss, malaise, abdominal pain, and myalgias. Workup reveals that the patient has polyarteritis nodosa. Which of the following is associated with this form of vasculitis? (A) Arsenic (B) Chlamydia pneumonia (C) Hepatitis B virus (D) Human herpesvirus 8 (E) Human immunodeficiency virus

The answer is C. The illustration shows noncaseating granulomas and giant cells, which, in the clinical setting described, are diagnostic of sarcoidosis. A frequent abnormal laboratory finding is polyclonal hypergammaglobulinemia along with hypercalcemia. Anergy to tuberculin is often demonstrable. The disorder is much more common in persons of African lineage. Patients most often present with lung findings and hilar lymphadenopathy, but any organ system can be involved.

A 23-year-old man presents with radiographic evidence of bilateral hilar lymphadenopathy and interstitial lung disease. A lung biopsy gives findings similar to those shown in the figure. A major characteristic of this disorder is (A) a positive test for tuberculin. (B) highest incidence in persons of Asian lineage. (C) hypercalcemia. (D) hypogammaglobulinemia. (E) involvement restricted to the lung.

The answer is E. Hydatidiform mole should be suspected when the uterus is enlarged beyond the expected size for the time of the pregnancy. HCG is markedly elevated in this gestational trophoblastic disease. Preeclampsia and eclampsia are forms of toxemia of pregnancy marked by severe hypertension, albuminuria, and edema, with the addition of convulsions and DIC in the latter. Placenta accreta occurs when the placenta adheres directly to the myometrium, leading to severe bleeding at the time of delivery. Ectopic pregnancy is usually discovered early in a suspected pregnancy when ultrasound examination reveals the uterus to be empty.

A 23-year-old woman consults an obstetrician because of the onset of vaginal bleeding in what she considers to be the fifth month of pregnancy; however, examination reveals the uterus to be enlarged to the size of a 7-month pregnancy. Intravaginal ultrasound fails to detect a fetal heartbeat and instead shows a "snowstorm pattern." HCG is markedly elevated. These findings are strongly suggestive of (A) preeclampsia. (B) eclampsia. (C) placenta accreta. (D) ectopic pregnancy. (E) hydatidiform mole.

The answer is D. Patent ductus arteriosus and septal defects are the most frequent congenital cardiac abnormalities associated with congenital rubella infection; however, defects are not limited to the cardiovascular system, and congenital infection can also lead to deafness and mental retardation. In addition, congenital rubella infection, along with other congenital intrauterine infections, accounts for only a small proportion of cases of congenital heart disease, the majority being of unknown cause. The most severe consequences occur as a result of infection during the first trimester of pregnancy. As with all infections, an IgM antibody response indicates recent primary infection.

A 23-year-old woman consults an obstetrician because she is hoping to become pregnant but is concerned about possible consequences of rubella infection. She received all of her childhood immunizations, but now she has been found to be negative for antibodies to rubella. The obstetrician administers a rubella vaccination and advises her to return for an anti-rubella titer prior to becoming pregnant. Which of the following is true regarding congenital rubella infection? (A) Associated fetal defects are limited to the cardiovascular system. (B) The fetus is most vulnerable during the third trimester of pregnancy. (C) The majority of cases of congenital heart disease are caused by rubella or other intrauterine infections. (D) Patent ductus arteriosus and septal defects are the most frequent congenital cardiac abnormalities associated with rubella infection. (E) A predominant IgG antibody response indicates recent primary infection.

The answer is C. Liver adenomas are benign liver tumors commonly associated with oral contraceptive use in young women. They are of concern because they resemble hepatocellular carcinoma. In addition, if they are subcapsular, they can rupture, causing intraabdominal hemorrhage. Hepatitis B and hepatitis C can lead to hepatocellular carcinoma. Polycythemia vera is associated with thrombosis of the hepatic veins. Polyvinyl chloride predisposes to angiosarcoma of the liver.

A 23-year-old woman is involved in a minor motor vehicle accident, prompting an abdominal computed tomography scan, which was read by the emergency department radiologist as normal with the exception of a questionable mass in the right lobe of the liver. A subsequent fine-needle biopsy confirms the presence of a liver adenoma. Which of the following is associated with the development of this lesion? (A) Hepatitis B (B) Hepatitis C (C) Oral contraceptives (D) Polycythemia vera (E) Polyvinyl chloride

The answer is E. The diagnosis is Hodgkin lymphoma, NS subtype. This form of Hodgkin lymphoma differs from other forms of classical Hodgkin lymphoma in being the most common in young women, having a relatively favorable clinical course, and having little association with EBV infection. Lacunar cells are considered a Reed-Sternberg cell variant, and the diagnosis of NS can be based on the finding of fibrous bands and lacunar cells.

A 23-year-old woman presents with cervical and mediastinal lymphadenopathy. Biopsy of a cervical lymph node reveals a nodular appearance with fibrous bands, effacement of the lymph node architecture, and numerous lacunar cells. Which of the following is characteristic of this disorder? (A) Benign neoplasm (B) Frequent association with EBV infection (C) Most often a complication of human immunodeficiency virus infection (D) Peak incidence in early childhood (E) Relatively favorable clinical course

The answer is A. The patient presents with signs and symptoms of Graves disease, an autoimmune disorder in which patients develop autoantibodies that stimulate thyroid hormone production. As with many other autoimmune disorders, there is an association with certain human leukocyte antigens (HLAs); in the case of Graves disease, it is HLADR3 and HLA-B8.

A 23-year-old woman presents with tremor, restlessness, heat intolerance, palpitation, and unexplained weight loss. The thyroid is symmetrically enlarged, the pulse is rapid, the skin is moist and warm, and exophthalmos is apparent. This condition is considered to be (A) autoimmune. (B) congenital. (C) iatrogenic. (D) infectious. (E) nutritional.

The answer is D. Phenytoin is a commonly used antiseizure medication that can cause impaired folate absorption, with resultant folate deficiency and megaloblastic anemia. The drug is contraindicated for use during pregnancy, because folate is required during embryogenesis to prevent neural tube defects. Aplastic anemia results in a major failure of erythropoiesis. Myelophthisic anemia occurs in some cancer patients with metastatic disease. Warm antibody hemolytic anemia is manifested by anemia and reticulocytosis, often with spherocytosis. Hemolytic disease of the newborn most commonly occurs with Rh blood group incompatibility between mother and fetus.

A 23-year-old woman with epilepsy who is desirous of becoming pregnant is found to be anemic. Examination of a peripheral blood smear reveals oval macrocytes and hypersegmented neutrophils. Neurologic examination is entirely normal. Which of the following is the most likely cause of the anemia? (A) Alloantibodies directed against fetal red cell antigens (B) Bone marrow aplasia (C) Coating of red cells by IgG "warm" autoantibodies (D) Folate deficiency (E) Neoplastic replacement of the bone marrow

The answer is B. Phospholipase A2 catalyzes the release of arachidonic acid from membrane phospholipids. Arachidonic acid metabolism then proceeds through two major pathways, the lipoxygenase and cyclooxygenase pathways. The lipoxygenase pathway yields HETE and leukotrienes. The cyclooxygenase pathway yields thromboxanes and prostaglandins. Prostacyclin (PGI2) is synthesized in endothelial cells, and thromboxane A2(TxA2) is synthesized in platelets. It should be noted that prednisone inhibits both the cyclooxygenase and lipoxygenase pathways by inhibiting the formation of precursors to each pathway.

A 23-year-old woman with systemic lupus erythematosus (SLE) is placed on high doses of the steroid prednisone. Prednisone is a potent inhibitor of the enzyme phospholipase A2, which normally functions in which of the following events? (A) Cyclooxygenase pathway (B) Liberation of arachidonic acid from membrane phospholipids (C) Lipoxygenase pathway (D) Synthesis of prostacyclin (PGI2) (E) Synthesis of thromboxane A2(TxA2)

The answer is D. Condyloma acuminatum is a nonmalignant neoplastic-like papillomatous condition that affects epithelium of the perineal, vulval, perianal, and vaginal regions. It is caused by HPV (most often HPV 6 and 11). Genital herpes is usually caused by HSV type 2. T. pallidumis a spirochete and the etiologic agent of syphilis. H. ducreyi causes chancroid, a painful, erosive, chancre-like lesion. Candidais not a sexually transmitted organism and is often considered normal vaginal flora, although it is an important cause of vulvovaginitis.

A 25-year-old woman has cauliflowershaped perineal lesions that are diagnosed as condyloma acuminatum. The etiologic agent is (A) HSV. (B) Treponema pallidum. (C) Haemophilus ducreyi. (D) HPV. (E) Candida albicans.

The answer is A. The combination of emphysema and hepatic cirrhosis in a young person with a family history of similarly affected family members is strongly suggestive of homozygous α1-antitrypsin deficiency. In these instances the emphysema is usually panacinar in type.

A 24-year-old nonsmoker who does not consume alcohol is found to have both pulmonary emphysema and cirrhosis of the liver. A sister and several close relatives also have had similar findings. This history suggests that this patient's illness may be caused by deficiency of (A) α1-antitrypsin. (B) galactokinase. (C) glucose-6-phosphatase. (D) glucocerebrosidase. (E) phenylalanine hydroxylase.

The answer is D. The association of a severe febrile illness with signs of gastrointestinal dysfunction and a diffuse macular rash is strongly suggestive of toxic shock syndrome. This diagnosis is especially likely if clinical abnormalities are associated with concomitant menstruation and the use of highly absorbent tampons. The clinical abnormalities are due to staphylococcal infection of the tampon, demonstrable in vaginal secretions, and the elaboration of staphylococcal exotoxins. The differential diagnosis includes streptococcal infection, usually pharyngeal, with a scarlatiniform eruption; meningococcemia; endotoxemia associated with various gram-negative organisms; rickettsial infections, and so forth. Except for streptococcal infection, all of these conditions are less likely than toxic shock syndrome.

A 24-year-old woman is seen because of high fever, prostration, vomiting, and diarrhea. Her pulse is rapid and thready, and her blood pressure is 60/40 mm Hg. A diffuse generalized macular rash is noted. Culture of which of the following specimens most likely leads to the correct diagnosis? (A) Sputum (B) Urine (C) Stool (D) Cervical secretions (E) Cerebrospinal fluid

The answer is E. Sarcoidosis most often presents as a restrictive pulmonary disease that is characterized morphologically by noncaseating granulomas and can involve any organ system. Diagnostic features of note include highest incidence in persons of African lineage, somewhat higher incidence in women, bilateral interstitial pulmonary involvement, prominent hilar lymphadenopathy, polyclonal hypergammaglobulinemia, and hypercalcemia. Increased serum angiotensin-converting enzyme activity is a nonspecific indicator of granulomatous inflammation.

A 25-year-old African-American woman presents with fatigue, dyspnea, nonproductive cough, and chest pain. She does not smoke. A chest radiograph reveals prominent bilateral hilar lymphadenopathy ("potato nodules") and diffuse reticular densities in the interstitium of the lung. Laboratory studies reveal polyclonal hypergammaglobulinemia, hypercalcemia, and increased serum angiotensin-converting enzyme. Which of the following is the most likely diagnosis? (A) Acute respiratory distress syndrome (B) Adenocarcinoma of the lung (C) Eosinophilic granuloma (D) Idiopathic pulmonary fibrosis (E) Sarcoidosis

The answer is B. Nodular melanoma tends to expand vertically rather than horizontally, a phenomenon associated with a more aggressive course and a greater likelihood of metastasis. Among the malignant melanomas, nodular melanoma has the poorest prognosis.

A 25-year-old woman has recently been diagnosed with a suspicious pigmented skin lesion and is waiting for the results of a biopsy. She is concerned because she had heard that such lesions vary, with some being more serious than others. Which of the following pigmented lesions is most likely to metastasize early? (A) Dysplastic nevus (B) Nodular melanoma (C) Pigmented nevus (D) Juvenile melanoma (E) Superficial spreading melanoma

The answer is C. The bleeding disorder is most likely factor VIII deficiency. The patient has a disorder of the intrinsic pathway of coagulation (prolonged APTT). The abnormality is localized proximal to factor X →Xa activation because the PT is normal. Significant platelet-related problems, such as von Willebrand disease, are ruled out by the normal bleeding time. The two most common intrinsic pathway factor deficiencies are factor VIII and factor IX. Of these, factor VIII deficiency occurs 5 to 10 times more frequently than factor IX deficiency and, therefore, is the most likely cause of the bleeding disorder.

A 25-year-old man has a lifelong hemorrhagic diathesis. The PT and bleeding time are normal, but the APTT is prolonged. The most likely cause of the bleeding disorder is (A) a platelet functional disorder. (B) factor VII deficiency. (C) factor VIII deficiency. (D) factor IX deficiency. (E) von Willebrand disease.

The answer is E. The clinical description is most consistent with infection with Neisseria gonorrhoeae,which most often manifests in men as acute purulent urethritis. Without treatment, gonorrheal infection can extend to the prostate and seminal vesicles and sometimes to the epididymis. The testes are rarely involved.

A 25-year-old man is seen because he has a purulent urethral discharge. Microscopic examination reveals the presence of gram-negative diplococci within neutrophilic phagocytes. Which of the following structures is most resistant to infection with the likely organism responsible for this man's findings? (A) Urethra (B) Prostate (C) Seminal vesicles (D) Epididymis (E) Testes

The answer is C. Interstitial (primary atypical) pneumonia is most commonly caused by M. pneumoniaeor viruses. Interstitial pneumonia is characterized by diffuse, patchy inflammation localized to the interstitial areas of alveolar walls, with no exudate in alveolar spaces, and intra-alveolar hyaline membranes. M. pneumoniaeinfection is associated with the presence of cold agglutinins, which are IgM antibodies that react nonspecifically with all human red blood cells. P. jiroveci (carinii)pneumonia is the most common opportunistic infection in patients with acquired immunodeficiency syndrome or other immunodeficiency disorders. Viral pneumonias are the most common type of pneumonia in childhood, caused most commonly by the influenza virus.

A 25-year-old man presents with a progressive illness of several days' duration characterized by nonproductive cough, fever, and malaise. A lateral view chest radiograph reveals platelike atelectasis. Elevated titers of cold agglutinins are detected. Which of the following is the most likely type of pneumonia in this patient? (A) Bacterial pneumonia, most likely caused by S. pneumoniae (B) Hospital-acquired pneumonia, most likely caused by P. aeruginosa (C) Interstitial pneumonia, most likely caused by M. pneumoniae (D) P. jiroveci(carinii) pneumonia, most likely related to an immunocompromised state (E) Viral pneumonia, most likely caused by influenza virus

The answer is D. Erythema multiforme manifests as a variegated group of lesions: macules, papules, and vesicles. The most characteristic of these is the "target" lesion. The disorder is thought to be due to hypersensitivity, often to coexistent stimuli, such as infectious agents or drugs, or to a concomitant connective tissue disorder or an associated malignancy.

A 25-year-old man presents with a skin rash of "target" lesions on the palms of the hands, the soles of the feet, and the arms and legs. The rash is composed of macules, papules, and vesicles. The dermatologist makes a diagnosis of erythema multiforme. Which of the following is the probable etiology of this condition? (A) Physical scratching of the skin (B) IgG autoantibodies directed against the epidermal intercellular cement substance (C) IgA antibody deposits localized to the tips of dermal papillae (D) Hypersensitivity to multiple concomitant infectious agents or drugs (E) Chemical exposure

The answer is D. Focal segmental glomerulosclerosis has clinical manifestations similar to minimal change disease, but it tends to occur in older patients and does not respond well to steroid treatment. Glomerular hyaline sclerosis occurs in a focal (some, but not all, glomeruli) and segmental (only part of an affected glomerulus) distribution.

A 25-year-old man presents with generalized edema. Blood tests reveal severe proteinuria, hypoalbuminemia, and hyperlipidemia. The patient does not respond well to a course of corticosteroids. A renal biopsy demonstrates findings indicative of focal segmental glomerulosclerosis. Which of the following best describes the histologic findings in this disorder? (A) Crescentic formation in glomeruli on light microscopy (B) Intramembranous and epimembranous immune complex deposits in the glomerular basement membrane on electron microscopy (C) Nodular accumulations of mesangial matrix material on light microscopy (D) Sclerosis within capillary tufts that involves only some glomeruli and only parts of affected glomeruli on light microscopy (E) Wire-loop abnormalities from immune complex deposits and thickening of the glomerular basement membrane on light microscopy

The answer is A. The clinical description is that of Goodpasture syndrome (antiglomerular basement membrane disease), caused by antibodies directed against antigens in the glomerular and pulmonary alveolar basement membranes. Because antigens are an intrinsic component of the basement membrane, labeled antibodies "paint" the surface of the basement membrane, resulting in the characteristic linear immunofluorescent pattern characteristic of this disorder. "Lumpy-bumpy" immunofluorescence is very coarse, granular immunofluorescence found in poststreptococcal immune complex deposit disease. The tram-track appearance is seen in membranoproliferative glomerulonephritis. The "spike and dome" appearance is seen in membranous glomerulonephritis. Subendothelial immune complex deposition is seen in lupus nephropathy.

A 25-year-old man presents with hematuria, periorbital edema, hypertension, and hemoptysis. He has also experienced nausea, vomiting, fever, and chills. Serologic testing is positive for antiglomerular basement membrane antibodies. Which of the following is the classic histologic finding in this renal disease? (A) Linear immunofluorescence (B) "Lumpy-bumpy" immunofluorescence (C) "Spike and dome" appearance of the glomerular basement membrane (D) Subendothelial immune complex deposition (E) Tram-track appearance of the glomerular basement membrane on electron microscopy

The answer is D. Crohn disease and ulcerative colitis are the two classic inflammatory bowel diseases. Crohn disease is a chronic inflammatory condition that can affect any part of the gastrointestinal tract from mouth to anus, but most commonly involves the distal ileocecum, small intestine, or colon. Morphologically, Crohn disease manifests as transmural inflammation (involving all layers of the intestinal wall), thickening of involved intestine, linear ulceration, a cobblestone appearance, skip lesions (normal intestine between affected regions), and granulomas. Strictures and fistulae may develop, leading to intestinal obstruction. Crohn disease may lead to carcinoma of the small intes-tine or colon, but much less commonly than ulcerative colitis.

A 25-year-old man presents with lowgrade fever, weight loss, fatigue, crampy abdominal pain, episodic diarrhea, and postprandial bloating. Right lower quadrant tenderness is elicited on palpation of the abdomen. A capsule endoscopy reveals thickening of the terminal ileum, edema, marked luminal narrowing, and a cobblestone appearance of the mucosa. Which of the following is a characteristic of this condition? (A) Additional typical findings include crypt abscesses and pseudopolyps. (B) Inflammation and ulceration limited to mucosa and submucosa with sparing of deeper layers. (C) It can affect any portion of the gastrointestinal tract, but proximal jejunum is most common site of involvement. (D) It can cause fistula formation between loops of affected bowel. (E) It is a benign, self-limited disorder with no complicating sequelae.

The answer is C. The leading cause of pancreatitis, particularly in nonalcoholic patients, is cholelithiasis, or gallstones. Gallstones obstruct the pancreatic ducts, leading to autodigestion of the pancreas by the enzymes it normally secretes into the duodenum (i.e., lipases). These lipids then form soaps (saponify) with calcium, leading to hypocal-cemia. Alcohol is another leading cause of pancreatitis. Cystic fibrosis, abetalipoproteinemia, and mumps infection are all less common causes of pancreatitis.

A 25-year-old obese woman who denies any history of alcohol abuse presents with severe abdominal pain radiating to the back. Laboratory results indicate an increase in serum amylase and lipase, with a marked decrease in calcium. Which of the following likely has caused this condition? (A) Abetalipoproteinemia (B) Alcohol (C) Cholelithiasis (D) Cystic fibrosis (E) Mumps

The answer is E. This is a classic case of acute pyelonephritis, an acute infection of the renal parenchyma. White cell casts in the urine are pathognomonic of acute pyelonephritis. Although microscopic hematuria is a frequent finding in acute pyelonephritis and other urinary tract infections, red cell casts are not seen since the glomeruli tend to be spared in renal infection. Red cell casts are a specific indicator of glomerular inflammation.

A 25-year-old woman experiences the sudden onset of fever, chills, right flank pain, and right-sided costovertebral angle tenderness. Her urinary sediment contains numerous gram-negative bacilli. Which of the following additional urinary findings would help establish the likely diagnosis? (A) Broad waxy casts (B) Decreased protein (C) Decreased volume (D) Red cell casts (E) White cell casts

The answer is C. Multiple sclerosis is the most common demyelinating disease and is characterized by destruction of myelin with relative preservation of axons. The classic CSF electrophoretic finding is that of multiple oligoclonal bands of immunoglobulin. Also, the incidence of multiple sclerosis is increased in association with HLA haplotypes A3, B7, DR2, and DW2, suggesting that immune factors may play a role. The disease is associated with plaques scattered irregularly throughout the central nervous system. Common sites are the optic nerve, brain stem, and paraventricular areas. Although characterized by exacerbations and remissions, the course is progressive, leading to increasing disability. Albumino-cytologic dissociation in the CSF is characteristic of Guillain-Barré syndrome. Increased neutrophils, decreased glucose, and increased protein are characteristics of bacterial meningitis.

A 25-year-old woman presents with brief episodes of loss of vision in her left eye, paresthesias (sensory loss), and clumsiness in her hands. She states that these episodes come and go. Magnetic resonance imaging (MRI) of the head reveals paraventricular plaques of demyelination in the central nervous system white matter. A CSF tap is performed. The patient is diagnosed with multiple sclerosis. Which of the following is the most likely finding in the CSF? (A) Acellular, with normal glucose and normal protein (B) Albumino-cytologic dissociation, with markedly increased protein but only modestly increased cell count (C) Increased immunoglobulin, manifesting as multiple oligoclonal bands on electrophoresis (D) Increased neutrophils, with decreased glucose and increased protein

The answer is A. A melanocytic nevus (common mole) is a benign tumor. Nevus cells are derived from melanocytes and ordinarily occur in clusters or nests. A dysplastic nevus is an atypical, irregularly pigmented lesion with disorderly proliferation of melanocytes, dermal fibrosis, and lymphocytic infiltration. Dysplastic nevi may transform into malignant melanoma. Lentigo maligna is characterized by atypical melanocytes at the epidermal-dermal junction and is a precursor to lentigo maligna melanoma.

A 25-year-old woman presents with concern about a mole on her abdomen. The mole is small, round, symmetric, and darkcolored, and has sharply defined borders. It is diagnosed as a common mole. This lesion is properly termed a (A) nevocellular nevus. (B) blue nevus. (C) Spitz nevus. (D) dysplastic nevus. (E) lentigo maligna.

The answer is A. Association with certain human leukocyte antigen (HLA) types, autoantibodies, and increased incidence in persons with other autoimmune disorders are frequent occurrences in autoimmune disorders. In addition, Hashimoto thyroiditis is characterized by dense lymphocytic infiltrates with germinal center formation, striking morphologic evidence of immune cell (B lymphocyte) participation.

A 43-year-old woman has a recent diagnosis of Hashimoto thyroiditis. This disease's autoimmune nature is suggested by which of the following features? (A) Association with human leukocyte antigen (HLA)-DR5 and HLA-B5 (B) Atrophy of thyroid follicles (C) Palpable thyroid (D) Presence of Hürthle cells (E) Signs and symptoms of hypothyroidism

The answer is E. The radiographic calcifications are highly suggestive of a mature teratoma (dermoid cyst). This tumor is composed of all three germ layers and often contains skin (including adnexal structures, such as sebaceous glands and hair follicles), bone, teeth, cartilage, and gastrointestinal, neurologic, respiratory, and thyroid tissues. As the tumor enlarges, it is at risk of torsion. Granulosa cell tumor presents with signs and symptoms of excess estrogen production. Brenner tumor is rare and histologically resembles transitional epithelium of the bladder. Serous cystadenoma is commonly bilateral. Struma ovarii is a monodermal teratoma composed of functional ectopic thyroid tissue.

A 26-year-old woman experiences the sudden onset of left-sided lower abdominal pain with radiation to the back, pelvis, and thigh. Following a negative test for pregnancy, abdominal radiography shows an enlarged left ovary with multiple calcifications. Laparoscopy reveals adnexal torsion and an ovarian tumor. Ovariectomy is performed. The tumor is most likely a (A) granulosa cell tumor. (B) Brenner tumor. (C) serous cystadenoma. (D) struma ovarii. (E) dermoid cyst.

The answer is E. Urinary vanillylmandelic acid, a norepinephrine metabolite, is markedly elevated in pheochromocytoma. Serum C-peptide is elevated in insulinoma. Serum calcitonin is sometimes used to screen for medullary carcinoma of the thyroid. Serum hemoglobin A1c is an indicator of long-term blood glucose control in diabetes mellitus. Urine aldosterone is elevated in aldosteronism, both primary and secondary.

A 26-year-old woman has episodic hypertension with headache, diaphoresis, and palpitation. Which of the following diagnostic procedures would be most useful in evaluating the possibility that a pheochromocytoma might be the cause of these findings? (A) Serum C-peptide (B) Serum calcitonin (C) Serum hemoglobin A1c (glycosylated hemoglobin) (D) Urinary aldosterone (E) Urinary vanillylmandelic acid

The answer is E. Bacterial vaginosis is the most common cause of vaginal discharge. Characteristically, the discharge is thin and homogeneous. Addition of 10% potassium hydroxide elicits a fishy amine odor. Usually, increased numbers of G. vaginalisorganisms are present and large numbers of "clue cells" are apparent on the Pap smear. The condition is caused by loss of the normal vaginal lactobacilli, with consequent overgrowth of anaerobes and other bacteria, including G. vaginalis. N. gonorrhoeaeis a common cause of pelvic inflammatory disease. Candidacauses vulvovaginitis. T. vaginalisis yet another cause of vaginosis, but a wet mount displays flagellated motile organisms.

A 26-year-old woman presents for routine gynecologic examination and Pap smear. A thin, homogenous vaginal discharge is noted, and a sample is taken. When potassium hydroxide is added to a wet mount of the sample, a fishy odor is noted. In addition, the Pap smear reveals the presence of "clue cells." Which of the following organisms is likely to be present in increased numbers? (A) Staphylococcus aureus (B) Neisseria gonorrhoeae (C) Candida albicans (D) Trichomonas vaginalis (E) Gardnerella vaginalis

The answer is B. The patient has a case of hepatitis A (HAV), most likely associated with the consumption of uncooked shellfish. Complete resolution occurs in an overwhelming majority of cases. Although possible, fulminant hepatic failure is unlikely with HAV infection. There is no association of HAV with either cirrhosis or hepatocellular carcinoma, as there is for hepatitis B and C.

A 26-year-old woman presents to her primary care physician with fever, malaise, and "yellow eyes." She denies alcohol abuse, but admits to indulging in a dozen raw oysters at happy hour 3 weeks ago. In addition to scleral icterus, physical examination reveals a mildly enlarged liver with tenderness to palpation. Laboratory studies demonstrate a markedly increased aspartate aminotransferase and alanine aminotransferase and increased IgM and antihepatitis A titers. Which of the following is the most likely result of this infection? (A) Cirrhosis (B) Complete resolution (C) Establishment of a chronic carrier state (D) Fulminant hepatitis (E) Hepatocellular carcinoma

The answer is A. Hypochromic microcytic anemia is most often associated with iron deficiency secondary to chronic blood loss. Hookworm infestation causes chronic blood loss and should not be confused with fish tapeworm infestation, which causes megaloblastic anemia. Folate deficiency with megaloblastic anemia can occur in severely malnourished persons (often alcoholics) or in association with increased demand for folate in pregnancy. Cobalamin (vitamin B12) deficiency megaloblastic anemia can occur in pernicious anemia, in strict vegetarians (vitamin B12 is only found in foods of animal origin), and in association with surgically induced intestinal blind loops overgrown with microorganisms with high avidity for cobalamin. The anemia of chronic disease is most often normochromic and normocytic, but can be hypochromic and microcytic. Usually, signs and symptoms of the underlying chronic disease are evident.

A 27-year-old man who recently arrived in the United States from Central America is found to have hypochromic microcytic anemia. Which of the following is the most likely cause of this finding? (A) Hookworm infestation (B) Fish tapeworm infestation (C) A strict vegetarian diet (D) Prolonged use of the antiseizure medication phenytoin (E) Anemia of chronic disease

The answer is B. Splenomegaly occurs in children with sickle cell anemia, but repeated bouts of splenic infarction and fibrosis reduce the spleen to a fibrous remnant in adults (autosplenectomy). The other listed conditions are all well-known causes of splenomegaly.

A 45-year-old African-American man has marked splenomegaly. This finding is unusual and unexpected in which of the following disorders? (A) Infectious mononucleosis (B) Sickle cell anemia (C) Chronic myelogenous leukemia (D) Hereditary spherocytosis (E) Agnogenic myeloid metaplasia (idiopathic myelofibrosis)

The answer is D. Concern over familial breast cancer is well warranted in this patient, with two first-degree relatives dying of breast cancer at an early age. Both BRCA-1 and BRCA-2 mutations are major risk factors for the development of breast cancer. Typically, breasts in a younger woman are too dense to be viewed well on normal screening mammograms, although in this case the mammogram and genetic studies are warranted to determine if she too carries the deleterious mutations. There is evidence that a first pregnancy after 30 years of age, early menarche, and late menopause are also contributing factors. A high-fat diet, not a high-fiber diet, is another apparent risk factor. There is no evidence to support that either oral contraceptive use or fibroadenoma is a risk factor for breast cancer.

A 27-year-old woman requests a mammogram because both her mother and sister died of metastatic breast cancer before 40 years of age. Which of the following would add to this patient's risk factors for breast cancer? (A) Multiparity (B) High-fiber diet (C) Oral contraceptive use (D) BRCA-1 mutation (E) Bilateral fibroadenomas

The answer is D. Recurrent intractable peptic ulcer disease is characteristic of the Zollinger-Ellison syndrome with excess gastrin production, most often from a gastrinoma. Cushing syndrome causes an excess of cortisol. Glucagonoma is a rare neuroendocrine tumor of pancreatic alpha cells that can cause hyperglycemia. The Whipple triad (episodic hyperinsulinemia and hypoglycemia causing CNS dysfunction reversible by glucose administration) is seen with insulinoma. Acromegaly results from growth hormone excess.

A 28-year-old man is evaluated for recurrent peptic ulcer disease, apparently refractory to pharmacologic intervention. Serum gastrin is markedly elevated. These findings are most characteristic of which of the following? (A) Cushing syndrome (B) Glucagonoma (C) Whipple triad (D) Zollinger-Ellison syndrome (E) Acromegaly

The answer is D. Primary adrenocortical deficiency (Addison disease), as distinguished from adrenal cortical insufficiency secondary to pituitary hypofunction, is indicated by the presence of pigmentation. Also, decreased plasma cortisol in Addison disease is unresponsive to ACTH administration. Progressive weakness and hypotension are strongly suggestive of adrenocortical deficiency regardless of cause, and other expected findings include decreased serum sodium, increased serum potassium, decreased serum glucose, and decreased urinary 17-ketosteroids.

A 28-year-old man was evaluated for progressive weakness, weight loss, and anorexia. He was found to be hypotensive, and had generalized hyperpigmentation involving exposed surfaces of the skin, lips, and buccal mucosa. Which of the following laboratory findings is expected in this patient? (A) Increased serum sodium (B) Decreased serum potassium (C) Increased serum glucose (D) Decreased plasma cortisol not corrected by administration of adrenocorticotropic hormone (ACTH) (E) Increased urinary 17-ketosteroids

The answer is C. Paroxysmal nocturnal hemoglobinuria results in an acquired intracorpuscular defect in the ability to synthesize GPI anchors required for appropriate placement of complement regulatory proteins on the surface of red blood cells. Functional deficiency of such proteins as CD55 and CD59 renders the cells sensitive to complementmediated lysis. Anti-intrinsic factor antibodies are seen in pernicious anemia. Spectrin is deficient in hereditary spherocytosis. Ineffective erythropoiesis is seen in megaloblastic anemia due to folate or vitamin B12 deficiencies. Substitution of valine for glutamic acid in the β-globin gene underlies the defect in sickle cell anemia.

A 28-year-old woman complains of fatigue, dyspnea, and malaise. She also notes that her urine has been reddishbrown, particularly with the first void of the morning. Subsequent studies confirm that she has paroxysmal nocturnal hemoglobinuria. Which of the following best describes the defect leading to this condition? (A) Anti-intrinsic factor antibodies (B) Deficiency of the intracellular structural protein spectrin (C) Impaired synthesis of the cell-surface GPI anchor (D) Ineffective erythropoiesis (E) Substitution of a valine for a glutamate residue in the β-globin gene

The answer is A. The most common cause of a breast mass in women younger than age 25 is fibroadenoma. Characteristically, this benign tumor presents as a firm, rubbery, painless, well-circumscribed lesion.

A rubbery, well-encapsulated, freely movable mass is found in the breast of a 20-year-old woman. Which of the following is most likely? (A) Fibroadenoma (B) Fibrocystic disease (C) Intraductal papilloma (D) Paget disease (E) Phyllodes tumor

The answer is E. The findings are those of the polycystic ovary (Stein-Leventhal) syndrome, an important cause of infertility in young women that is characterized by amenorrhea, infertility, obesity, and hirsutism. Increased levels of LH and of androgens are often found, and it has been thought that LH may stimulate follicular theca-lutein cells, with consequent hyperproduction of androgens. Alternatively, there has been recent attention to the link of this disorder to the metabolic syndrome and insulin resistance. Hyperinsulinemia may lead to increased ovarian androgen production, which then may lead to increased LH. The syndrome is characterized by multiple small follicular cysts, a markedly thickened ovarian capsule, and cortical stromal fibrosis. A mass resembling a bunch of grapes projecting into the vagina is characteristic of sarcoma botryoides. Bloodfilled cysts on the ovary are termed "chocolate cysts" and are associated with endometriosis, another cause of infertility. Fluid accumulation in the fallopian tubes is termed hydrosalpinx. Islands of endometrium in the myometrium are seen in adenomyosis.

A 28-year-old woman is evaluated for infertility and amenorrhea. She is obese and has significant facial hair in a male distribution. Laboratory studies reveal an increase in serum LH, as well as in total serum testosterone and in the testosterone metabolite dehydroepiandrosterone sulfate. A pelvic ultrasound examination would probably reveal which of the following findings? (A) A mass resembling a bunch of grapes projecting into the vagina (B) Blood-filled cysts in the ovary (C) Fluid accumulation in the fallopian tubes (D) Islands of endometrium within the myometrium (E) Multiple small follicular cysts on the ovary

The answer is A. Organisms involved in urinary tract infections are most often normal flora of the colon, and the most frequent of these is E. coli.The other pathogen that would be likely in this patient is S. saprophyticus, however that was not an answer choice.

A 28-year-old woman presents with fever, dysuria, urinary frequency, and flank tenderness. The urine contained numerous neutrophils and many white cell casts. Urine protein was moderately increased. A quantitative urine culture revealed more than 105 bacteria per milliliter. The most likely causative organism is (A) Escherichia coli. (B) Haemophilus influenzae. (C) Neisseria gonorrhoeae. (D) Proteus vulgaris. (E) Pseudomonas aeruginosa.

The answer is C. True hermaphroditism requires the presence of both ovarian and testicular tissue, as in this case. The karyotype is either XX (with translocation of at least part of the Y chromosome to an X chromosome or to an autosome) or a mosaicism, such as XX/XXY.

A 3-month-old, apparently female infant is evaluated for ambiguous genitalia. The clitoris appears large, and there are palpable masses in the inguinal region. Further investigation reveals the presence of both ovarian and testicular tissue. Which of the following terms is used to describe these findings? (A) Male pseudohermaphroditism (B) Female pseudohermaphroditism (C) True hermaphroditism (D) Turner syndrome (E) Klinefelter syndrome

The answer is B. The patient presents with mucocutaneous lymph node syndrome, or Kawasaki disease. It is often a self-limiting condition, although as many as 20% of patients develop an aneurysm of the coronary vessels. Dissection of the thoracic aorta is associated with hypertension. Rupture of a berry aneurysm can result in a subarachnoid hemorrhage. Aneurysm of the abdominal aorta is associated with atherosclerosis. Ischemia and gangrene can be a result of Buerger disease.

A 3-year-old boy is brought by his parents to the emergency department because they are concerned that he has had a high fever for several days. On examination, the boy has conjunctival and oral erythema. He has palpable cervical lymphadenopathy and erythema of his palms and soles. What is a potential life-threatening complication of this disorder? (A) Aneurysm of the abdominal aorta (B) Aneurysm of the coronary arteries (C) Dissection of the thoracic aorta (D) Ischemia and gangrene of the extremities (E) Rupture of a berry aneurysm

The answer is D. During the first several hours of an inflammatory process, the predominant inflammatory cells are neutrophils. After 1 or 2 days, neutrophils are largely replaced by longer-lived monocytes-macrophages.

A 3-year-old boy is brought to the clinic because of fever and "fussiness," and he is diagnosed as having acute otitis media. In this acute inflammatory reaction, which of the following cells would have reached the site of inflammation first? (A) Basophils (B) Lymphocytes (C) Monocytes-macrophages (D) Neutrophils (E) Plasma cells

The answer is B. Endodermal sinus (yolk sac) tumor is the most common malignant germ cell tumor of the testis in infancy and early childhood. It is characterized by an increase in serum α-fetoprotein, as well as histologically stainable α1-antitrypsin. Choriocarcinoma is less common in this age group and often results in an increase in hCG. Although hepatocellular carcinomas also elaborate α-fetoprotein, this is very unlikely in this patient's clinical setting. Leydig cell tumors are derived from testicular stroma and produce androgens and estrogens, often presenting with precocious puberty. Teratomas contain multiple germ layers and numerous tissues, including hair, teeth, and sebaceous tissue. Unlike the teratomas encountered in women, most teratomas in men are malignant.

A 3-year-old boy is brought to the pediatrician because his mother noticed an abnormal mass in his scrotum while changing his diapers. Further workup demonstrates elevated levels of serum α-fetoprotein. Which of the following is the most likely diagnosis? (A) Choriocarcinoma (B) Endodermal sinus (yolk sac) tumor (C) Hepatocellular carcinoma (D) Leydig cell (interstitial) tumor (E) Teratoma

The answer is D. In the tetralogy of Fallot, the characteristic lesions include ventricular septal defect, overriding aorta, pulmonary valve stenosis, and right ventricular hypertrophy. The pulmonary stenosis and overriding aorta cause increased right ventricular pressure and lead to right-to-left shunting. Cyanosis, which occurs when the arterial concentration of reduced hemoglobin exceeds 5 mg/mL, is seen with a right-to-left shunt, in which venous blood gains direct access to the arterial circulation. In contrast, patent ductus arteriosus, atrial septal defect, and ventricular septal defect are associated with left-to-right blood flow.

A 3-year-old boy presents with cyanosis and shortness of breath that develops when he plays with friends. According to his mother, the boy was born cyanotic. The boy is very small and short for his age, and he squats on the floor next to his mother. Chest radiography reveals a boot-shaped heart, normal heart size, and a right aortic arch. Echocardiography reveals a large ventricular septal defect with an overriding aorta, pulmonary stenosis, and right ventricular hypertrophy. Which of the following is the most likely diagnosis? (A) Coarctation of the aorta (B) Patent ductus arteriosus (C) Rheumatic heart disease (D) Tetralogy of Fallot (E) Transposition of the great vessels

The answer is A. ALL is the most common malignancy in children and is the form of acute leukemia that is most responsive to therapy. ALL is characterized by a predominance of lymphoblasts in the circulating blood and in the bone marrow. Other progenitor cells do not mature normally, resulting in neutropenia and thrombocytopenia. CD10-positive ALL is the most frequently occurring form of ALL and is the most amenable to therapy. Thus CD10 is a favorable prognostic marker of this disease.

A 3-year-old boy presents with epistaxis and fever. Multiple cutaneous petechiae are evident, and there is generalized enlargement of lymph nodes, as well as palpable splenomegaly. The hemoglobin and platelet count are markedly decreased, and the white blood cell count is elevated to 40,000 cells/µL, with a preponderance of lymphoblasts. Which of the following statements best characterizes this disorder? (A) It is the form of acute leukemia that is most responsive to therapy. (B) It occurs most often in adults but can occur in children. (C) Lymphoblastic cells cause damage to normal blood cells, resulting in low cell counts. (D) The presence of the CD10 marker is indicative of a poorer prognosis.

The answer is B. The child has rickets, which is caused by failure of action of calcitriol (1,25-dihydroxycholecalciferol), the active form of vitamin D. The bony abnormalities in rickets are caused by failure of osteoid matrix to calcify, leading to excess accumulation of osteoid, increased thickness of the epiphyseal growth plates, and other skeletal deformities. Many of the effects of calcitriol deficiency are mediated by increased release of parathyroid hormone (PTH). PTH stimulates bony osteoblastic activity, which is mirrored by an increase in serum alkaline phosphatase.

A 3-year-old boy, an inner city resident, has multiple bony abnormalities, including bowlegs and knock-knees, thickening of the skull with frontal bossing, knobby deformities of the costochondral junctions and, at the ends of the long bones, distortion of the rib cage with flaring over the diaphragm, and pigeon-breast deformity. A decrease in which of the following is characteristic of this condition? (A) Bony osteoblastic activity (B) Calcification of osteoid (C) Release of parathyroid hormone (D) Serum alkaline phosphatase activity (E) Synthesis of osteoid

The answer is E. This is a classic case of acute laryngotracheobronchitis (croup), an acute inflammation of the larynx, trachea, and epiglottis. The most common cause of croup is a viral (parainfluenza virus type I) infection.

A 3-year-old girl presents to the emergency department with fever, hoarseness, a "seal bark-like" cough, and inspiratory stridor. Her father states that she has had a cold for the past few days, with runny nose, nasal congestion, sore throat, and cough. He is now concerned because her cough has become loud, harsh, and brassy. Which of the following is the most likely cause of her ailment? (A) Fungus (B) Gram-negative bacteria (C) Gram-positive bacteria (D) Parasite (E) Virus

The answer is A. Classic features of lead poisoning include dark precipitates forming a gingival lead line (composed of precipitated lead sulfide), radiopaque deposits in epiphyses, basophilic stippling of erythrocytes, increased urinary delta-ALA, and peripheral neuropathy and other CNS changes.

A 3-year-old girl presents with dark precipitates along gingival margins, radiopaque deposits in the epiphyses of her bones, and urinary excretion of delta-aminolevulinic acid (delta-ALA). Her father states that they live in an old house that has chipped paint. The patient is diagnosed with lead toxicity. The child's blood would likely also have which of the following? (A) Basophilic stippling of erythrocytes (B) Schistocytes and helmet cells (C) Increased osmotic fragility of erythrocytes (D) Clumping of erythrocytes at temperatures below 30°C (E) Macrocytes

The answer is B. The clinical history is characteristic of multiple sclerosis, the most frequently occurring of the demyelinating diseases. Multiple sclerosis is characterized by destruction of myelin, with preservation of axons. The optic nerve, brain stem, and paraventricular areas are favored sites of demyelination. Other characteristics of note include multiple oligoclonal immunoglobulin bands on CSF electrophoresis, association with certain HLA haplotypes, and geographic distribution, with incidence increasing with distance away from the equator.

A 30-year-old woman has sudden blurring of vision in her right eye, paresthesias, and spasticity. CSF findings include oligoclonal bands on electrophoresis. Magnetic resonance imaging of the brain reveals T2 hyperintensities characteristic of demyelination in the paraventricular regions. Which of the following is a prominent characteristic of this patient's disorder? (A) Axonal degeneration (B) Optic nerve, brain stem, and paraventricular areas are favored sites (C) Confined to the peripheral nervous system (D) Associated with leukemia or lymphoma (E) Caused by prions

The answer is D. Myasthenia gravis is an autoimmune disorder caused by autoantibodies to acetylcholine receptors of the neuromuscular junction. Dramatic improvement in muscle strength with an anticholinesterase agent, such as edrophonium, is a useful diagnostic sign.

A 30-year-old woman presents with ptosis and severe generalized muscle weakness. Administration of edrophonium (an anticholinesterase) results in a rapid and dramatic recovery of muscle strength. Which of the following is the most likely diagnosis? (A) Alzheimer disease (B) Huntington disease (C) Idiopathic Parkinson disease (D) Myasthenia gravis (E) Wernicke-Korsakoff syndrome

The answer is C. Meningioma is a benign tumor of the meninges that is external to the brain and is therefore usually surgically resectable. Meningioma is the second most common primary intracranial neoplasm. This tumor causes a mass effect (a space-occupying defect) that physically compresses, but does not invade, brain parenchyma. Symptoms correlate with the location of the meningioma. For example, this patient's right-sided parasagittal meningioma caused slow and progressive left-sided lower extremity weakness.

A 30-year-old woman presents with weakness and hyperreflexia of the left lower extremity. These symptoms had begun as mild weakness, but had slowly and progressively become more severe. A CT scan of the head reveals a well-defined right-sided parasagittal mass compressing (but not invading) the brain parenchyma. Which of the following is the most likely diagnosis? (A) Glioblastoma multiforme (B) Medulloblastoma (C) Meningioma (D) Neurilemmoma (schwannoma) (E) Oligodendroglioma

The answer is C. Endometriosis, ectopic non-neoplastic endometrial tissue, is an important cause of infertility. The ectopic tissue replicates the cyclic endometrial changes during the menstrual cycle, including sloughing and bleeding. Such ectopic hemorrhage can result in pain, scarring, tubal obstruction, and infertility. Favored sites of endometriosis include pelvic ligaments, ovaries, and rectum. Adenomyosis is a term denoting the presence of endometrial tissue within the myometrium. Endometrial hyperplasia can be a precursor to endometrial carcinoma and manifests with postmenopausal bleeding. Endometritis results from infection of the endometrium. Leiomyomas are benign tumors of the uterus that can also result in painful menses and can contribute to infertility.

A 32-year-old woman has been attempting to become pregnant for the past 2 years without success. She also has had extremely painful menstrual cramping of many years' duration. An exploratory laparoscopy demonstrated multiple red-blue nodules covering the surface of her ovaries and uterine ligaments. These findings are most likely indicative of which of the following conditions? (A) Adenomyosis (B) Endometrial hyperplasia (C) Endometriosis (D) Endometritis (E) Leiomyoma

The answer is A. The findings described are associated with prolactinoma, the most common hormone-secreting tumor of the pituitary. Hyperprolactinemia results in amenorrhea and galactorrhea. The tumor is generally small; however, about 10% are large enough to impinge on adjacent structures, and bitemporal hemianopsia from pressure on the optic chiasm is common. Somatotropic adenoma (an older name is acidophilic adenoma) secretes growth hormone, causing acromegaly or gigantism, whereas corticotropic adenoma secretes ACTH, causing Cushing syndrome of pituitary origin (Cushing disease). Craniopharyngioma is a nonendocrine tumor of Rathke pouch origin that can be locally destructive, indirectly causing panhypopituitarism.

A 32-year-old woman presents with amenorrhea, galactorrhea, and visual field defects, all of several months' duration. Magnetic resonance imaging reveals a hypophyseal mass impinging on the optic chiasm. This is most likely a(n) (A) prolactinoma. (B) somatotropic adenoma. (C) corticotropic adenoma. (D) craniopharyngioma. (E) acidophilic adenoma.

The answer is C. The great majority of cases of hypertension are classified as essential hypertension. Essential hypertension results from the interactions of predisposing determinants and exogenous factors, including family history, ethnicity, stress, obesity, sodium intake, smoking, and physical activity. A small minority of cases of hypertension are due to secondary causes. Unilateral renal artery stenosis is a secondary cause that is typically correctable by surgery.

A 32-year-old woman returns to her physician for follow-up of hypertension that has been poorly controlled in spite of numerous antihypertensive medications. It is decided to evaluate the patient for possible "secondary" hypertension. Which of the following is a well-known cause of secondary hypertension? (A) Ethnicity (B) Obesity (C) Renal artery stenosis (D) Smoking (E) Stress

The answer is B. The patient is only positive for antibody to the hepatitis B antigen. This suggests that the patient has been vaccinated for hepatitis B virus (HBV). The vaccine consists of recombinantly produced HBV surface antigen (HBsAg) alone; antibodies to this protein convey immunity. HBsAg would be seen in the serum within the first 3 to 4 months after initial infection. Antibodies to the core protein (anti-HBcAg) appear during acute illness and between the disappearance of HBsAg and the appearance of anti-HBsAg, the "window period." Anti-HBeAg appears during the window period as well. HBeAg is closely correlated with viral infectivity. HBV viral DNA is also an index of infectivity.

A 32-year-old woman seeking to become pregnant visits her physician for a prepregnancy examination. Routine prenatal laboratory testing demonstrates the following profile: HBsAg (−), anti-HBsAg (+), anti-HBcAg (−), anti-HBeAg (−), and HBV DNA (−). Which of the following likely represents the status of the patient? (A) Hepatitis B carrier (B) Immunized against hepatitis B (C) Infected and within the "window period" (D) Infected with hepatitis B and highly transmissible (E) Recently infected with hepatitis B

The answer is E. The VIPoma is an islet cell tumor of the pancreas that is associated with Watery Diarrhea, Hypokalemia, and Achlorhydria (WDHA syndrome or Verner-Morrison syndrome), all caused by the secretion of vasoactive intestinal peptide (VIP) by the tumor.

A 32-year-old woman was evaluated for severe watery diarrhea. Diagnostic testing revealed achlorhydria and reduced concentrations of serum potassium. Imaging studies revealed the presence of a pancreatic tumor. Special stains will most likely reveal the pancreatic tumor to be which of the following? (A) Alpha-cell tumor (glucagonoma) (B) Beta-cell tumor (insulinoma) (C) Gastrinoma (D) Somatostatinoma (E) VIPoma

The answer is C. Pheochromocytoma of the adrenal medulla (and its extra-adrenal counterpart paraganglioma) secretes the catecholamines epinephrine and norepinephrine. Increased urinary excretion of catecholamines or their metabolites metanephrine and VMA is a clinical indicator of this tumor.

A 33-year-old woman presents with episodic palpitations, sweating, tremor, and a sense of apprehension. During these episodes, her blood pressure is markedly elevated. Which of the following laboratory tests is most likely to be of diagnostic significance? (A) Serum serotonin (B) Serum gastrin (C) Urine VMA (D) Urine cortisol (E) Serum PTH

The answer is E. The combination of hypertension, persistent hypokalemia, and slightly elevated serum sodium is highly suggestive of Conn syndrome (primary aldosteronism, hyperaldosteronism). The diagnosis can be confirmed by demonstration of increased aldosterone, lack of response of aldosterone to sodium loading, and decreased serum renin.

A 34-year-old man is referred for evaluation of hypertension and persistent hypokalemia in spite of taking oral potassium supplements. Blood pressure is 180/110 mm Hg. Serum sodium is 149 mEq/L (normal 140 to 148 mEq/L); potassium, 3.3 mEq/L (normal 3.6 to 5.2 mEq/L); bicarbonate, 29 mEq/L (normal 22 to 29 mEq/L); chloride, 103 mEq/L (normal 98 to 107 mEq/L); and urea nitrogen, 23 mg/dL (normal 7 to 18 mg/dL). Computed tomography demonstrates a 3-cm mass in the right adrenal gland. The most likely diagnosis is (A) Addison disease. (B) Cushing syndrome. (C) Sipple syndrome. (D) DiGeorge syndrome. (E) Conn syndrome.

The answer is D. Disseminated histoplasmosis is characterized by widespread dissemination of macrophages filled with fungal yeast forms.

A 34-year-old man, who is positive for human immunodeficiency virus (HIV) and a resident of the Ohio-Mississippi River valleys region, developed a systemic illness characterized by hepatosplenomegaly and generalized lymphadenopathy. Which of the following microscopic descriptions of organisms from a resected lesion is most consistent with a presumptive diagnosis of disseminated histoplasmosis? (A) Budding yeast forms surrounded by empty haloes (B) Cup-shaped forms demonstrable by silver stain within a foamy amorphous intra-alveolar exudate (C) Intracavitary mycelial forms (D) Minute fungal yeast forms within phagocytes (E) Thick-walled spherules filled with endospores

The answer is C. The illustration demonstrates a well-circumscribed adrenal cortical adenoma. Cushing syndrome is a manifestation of hyperproduction of adrenal glucocorticoids, and when of adrenal origin, it is most often caused by adrenal cortical adenoma. Pituitary and hypothalamic causes of Cushing syndrome result in bilateral adrenal cortical hyperplasia. In contrast, Cushing syndrome caused by exogenous steroid medication results in adrenal atrophy.

A 34-year-old woman is seen because of unexplained weight gain, selectively over the trunk, upper back, and back of the neck; irregular menstrual periods; and increasing obesity. She is especially concerned about the changing contour of her face, which has become rounder, creating a "moon-faced" appearance. She has also developed purple-colored streaking resembling stretch marks over the abdomen and flanks, as well as increased hair growth in a male distribution pattern. Blood pressure is elevated to 190/100 mm Hg. Blood sugar is elevated. Computed tomography reveals a smooth, homogeneous lesion in the left adrenal gland. Surgery is performed, and the resected adrenal resembled that shown in the figure. The clinical findings and the change in the adrenal gland are most likely related to which of the following? (A) Adrenal (glucocorticoid) steroid therapy (B) Ectopic production of adrenocorticotropin (C) Hyperproduction of adrenal glucocorticoids (D) Hyperproduction of hypothalamic corticotropin-releasing factor (E) Hyperproduction of pituitary corticotropin

The answer is C. This is a typical presentation for a patient with malignant hypertension. The kidneys appear "flea-bitten" because of multiple petechial hemorrhages on the surface. Microscopically, the glomeruli display fibrinoid necrosis and hyperplastic arteriolosclerosis. Immune complex vasculitis of the glomeruli is typical of polyarteritis nodosa. Longitudinal intraluminal tears of the ascending aorta occur with dissecting aneurysm. A "tree-bark" appearance of the ascending aorta results from tertiary syphilis. Unilateral renal artery stenosis can result from fibromuscular dysplasia.

A 35-year-old African-American man is transported to the emergency department because of an unrelenting, severe, excruciating headache. His blood pressure is 245/150 mm Hg, and bilateral papilledema is evident on ophthalmoscopic examination of the retina. Despite all interventions, including administration of nitroprusside, the patient dies. At autopsy, which of the following is a likely finding? (A) Immune complex vasculitis of the glomeruli (B) Longitudinal intraluminal tears of the ascending aorta (C) Multiple punctuate hemorrhages on the surface of both kidneys ("flea-bitten kidneys") (D) "Tree-bark" appearance of the ascending aorta (E) Unilateral renal artery stenosis

The answer is E. Seminoma, a germ cell tumor of the testis with peak incidence in the mid-30s age group, is analogous to (and histologically closely resembles) dysgerminoma of the ovary.

A 35-year-old man presents with a painless lump in his right testicle. Scrotal ultrasound reveals a homogeneous intratesticular mass. Histologic examination of the testicular mass tissue demonstrates morphology that closely resembles that of dysgerminoma of the ovary in women. Which of the following is the most likely diagnosis? (A) Androblastoma (Sertoli cell tumor) (B) Endodermal sinus tumor (C) Mature teratoma (D) Mixed germ cell tumor (E) Seminoma

The answer is B. This patient most likely has a mutation in CDH1, the gene encoding the cellular adhesion protein E-cadherin. E-cadherin mutations account for a significant proportion of familial gastric cancers and are also implicated in lobular carcinoma of the breast. CDH1-mutated gastric adenocarcinomas typically show signet-ring morphology with diffuse infiltration resulting in a "linitis plastic" ("leather bottle") gross appearance. APCis mutated in the majority of colorectal adenocarcinomas. MSH2and PMS2are both mismatch repair genes that may be mutated in Lynch syndrome. Although Lynch syndrome patients are at increased risk for gastric adenocarcinoma, their tumors are typically conventional, rather than signet-ring, in morphology. p53is mutated in a wide variety of sporadic cancers and shows germline mutations in Li Fraumeni syndrome.

A 35-year-old man undergoes gastrectomy for gastric carcinoma. Gross examination of the resected stomach reveals diffuse thickening without a discrete mass lesion. Microscopic exam shows an infiltration of signet-ring cells dispersed singly. Family history reveals that his father had a similar cancer at a young age. What gene is most likely to be mutated in this patient and his father? (A) APC (B) CDH1 (C) MSH2 (D) PMS2 (E) p53

The answer is E. Huntington disease results from an expansion of the CAG trinucleotide repeat within the huntingtingene, which can be detected by restriction length polymorphism (RFLP) analysis. Unlike many autosomal dominant disorders, it manifests later in life, in the fourth to fifth decade of life. Deposition of copper in the lenticular nuclei occurs in Wilson disease. A deficiency of sphingomyelinase results in some forms of Niemann-Pick disease. Loss of dopamine-producing neurons in the substantia nigra occurs in Parkinson disease.

A 35-year-old man whose father had died of Huntington disease has the onset of neurologic abnormalities that had been predicted earlier by genetic analysis. Changes in the distance between the heads of the two caudate nuclei by magnetic resonance imaging are consistent with atrophy of the caudate nucleus and putamen. Which of the following is an important characteristic of this disorder? (A) Copper deposition in lenticular nuclei (B) Early childhood onset most common (C) Sphingomyelinase deficiency (D) Substantia nigra depigmentation (E) Trinucleotide repeat expansion

The answer is B. The clinical picture is that of alcoholic hepatitis, which is characterized by fatty change, focal liver cell necrosis, infiltrates of neutrophils, and intracytoplasmic hyaline inclusions referred to as Mallory bodies.

A 35-year-old man with a known history of severe chronic alcohol abuse presents with low-grade fever, jaundice, hepatomegaly, leukocytosis, and markedly abnormal liver function tests. Aspartate aminotransferase (AST) and alanine aminotransferase (ALT) are both elevated. An expected histologic finding in this condition is (A) effacement of the normal liver architecture by diffuse fibrosis and abnormal regenerating nodules. (B) Mallory hyaline inclusions, macrovesicular steatosis, and neutrophilic infiltration. (C) multiple giant cells. (D) nests or cords of well-differentiated cells separated by dense collagen lamellae. (E) parenchymal deposition of hemosiderin.

The answer is D. The patient presents with a lesion of secondary syphilis known as condyloma lata. The original lesion, the painless ulcer or chancre, is characteristic of primary syphilis. Chancroid is a similar lesion to the primary lesion in syphilis, but instead is painful. Condyloma acuminatum is a similarly named lesion often referred to as a venereal wart and is due to HPV. Papillary hidradenoma is a common benign tumor of the vulva, presenting as a nodule that may ulcerate and bleed.

A 35-year-old prostitute is seen in a community health care clinic. About 4 months earlier, she had a painless labial sore and swelling of a right inguinal lymph node, both of which had subsided uneventfully over a period of several weeks. About 3 weeks later she developed fever and a generalized maculopapular skin rash that involved the palms of the hands and the soles of the feet. She has developed a flattened, wart-like labial lesion that is most likely a (A) chancre. (B) chancroid. (C) condyloma acuminatum. (D) condyloma lata. (E) papillary hidradenoma.

The answer is A. Carcinoma of the cervix is associated with infection with certain serotypes of HPV. Other characteristics include a history of early sexual activity, squamous cell morphology, and frequent origin at the squamocolumnar junction. Leiomyomas (fibroids) may increase in size during pregnancy and decrease in size following menopause. Endometrial carcinoma is the most common gynecologic malignancy and is associated with hyperestrinism from estrogen therapy. Endodermal sinus (yolk sac) tumors produce α-fetoprotein.

A 35-year-old woman consults a gynecologist because she has postcoital vaginal bleeding. The Papanicolaou (Pap) smear is abnormal. Colposcopy and cervical biopsy lead to a diagnosis of carcinoma of the cervix. Which of the following is most characteristic of this disorder? (A) Association with human papillomavirus (HPV) infection (B) History of exogenous estrogen therapy (C) Most common gynecologic malignancy (D) Secretion of AFP (E) Spontaneous regression following menopause

The answer is A. The history is strongly suggestive of panhypopituitarism due to ischemic necrosis of the pituitary, occurring as a sequela to childbirth complicated by hemorrhagic shock (Sheehan syndrome). This syndrome is clinically dominated by overt evidence of gonadotropin and corticotropin deficiencies, along with laboratory evidence of these deficiencies and thyrotropin deficiency. Overt secondary hypothyroidism sometimes occurs.

A 35-year-old woman is seen 6 months after giving birth to a normal infant. She suffered severe cervical lacerations during delivery, resulting in hemorrhagic shock. Following blood transfusion and surgical repair, postpartum recovery has so far been uneventful. She now complains of continued amenorrhea and loss of weight and muscle strength. Further investigation might be expected to demonstrate which of the following findings? (A) Decreased serum cortisol (B) Hyperestrinism (C) Hyperglycemia (D) Increased hair growth in a male distribution pattern (E) Increased serum free thyroxine

The answer is E. Graves disease is characteristically associated with decreased TSH activity. Thyroid-follicle TSH receptors are stimulated by TSI, an IgG autoantibody, not by TSH. Laboratory abnormalities in hyperthyroidism include increases in serum T4, serum T3, T3 resin uptake, and radioactive iodine uptake. Total T4 and T3 resin uptake are falling into disuse. The best screening tests for Graves disease are free T4 (elevated in Graves disease) and TSH (greatly decreased in Graves disease).

A 35-year-old woman presents with amenorrhea and weight loss despite increased appetite. The history and physical examination reveal exophthalmos, fine resting tremor, tachycardia, and warm, moist skin. Laboratory tests for thyroid function would be expected to yield a decreased value for which of the following? (A) Free T4 (B) Radioactive iodine uptake (C) T3 resin uptake (D) T3 (E) Thyroid stimulating hormone

The answer is D. The classic pentad of TTP includes fever, microangiopathic hemolytic anemia, thrombocytopenia, renal insufficiency, and neurologic abnormalities. Hyaline microaggregates of platelets in small vessels can be observed on histologic examination. The disorder is caused by deficiency of the enzyme vWF metalloprotease (ADAMTS 13). The enzyme promotes degradation of very-high-molecular-weight multimers of vWF, and the enzyme deficiency results in multimer accumulation in the plasma and consequent platelet microaggregate formation. The enzyme deficiency can be caused by a mutation in the gene that codes for the enzyme, or it can be caused by an antibody inhibiting the enzyme. Treatment is by plasma exchange, and the disorder can be fatal if diagnosis and therapy are delayed.

A 35-year-old woman presents with fever, fatigue, mucocutaneous bleeding, and changing neurologic signs. Laboratory examination reveals thrombocytopenia, anemia, and reticulocytosis, as well as increased concentrations of creatinine and urea nitrogen. Examination of a peripheral blood smear reveals many fragmented circulating red cells (helmet cells and schistocytes). The most likely diagnosis is (A) Bernard-Soulier disease. (B) DIC. (C) ITP. (D) TTP. (E) von Willebrand disease.

The answer is D. Both epithelioid cells and multinucleated giant cells are modified macrophages. A macrophage is a mononuclear phagocyte that has migrated into tissue.

A 35-year-old, HIV-positive man presents with productive cough, hemoptysis, fever, night sweats, weight loss, and anorexia. Chest radiograph demonstrates a cavitary lesion in the apex of the left lung. Sputum examination reveals acidfast bacilli. The patient is diagnosed with tuberculosis, in which the classic histologic feature is granulomatous inflammation. The epithelioid cells and multinucleated giant cells of this form of chronic inflammation are derived from which of the following? (A) Basophils (B) CD4T lymphocytes (C) Eosinophils (D) Monocytes and macrophages (E) Plasma cells

The answer is A. Aflatoxin B1 is a fungal metabolite found on moldy nuts and grain commonly found in southern Africa. It is a cocarcinogen with hepatitis B, which is nearly endemic to this region of the world. Together they greatly increase the incidence of hepatocellular carcinoma, the most prevalent cancer worldwide. C. sinensisis a parasite associated with the development of cholangiocarcinoma. Hepatitis A has no association with malignancy. Polyvinyl chloride is associated with angiosarcoma of the liver. Tetracyclines can cause microvesicular fatty change in the liver.

A 36-year-old man from sub-Saharan Africa presents to the clinic with jaundice and right upper quadrant pain. On examination, the liver is palpably enlarged. Laboratory studies demonstrate an increase in liver enzymes. Computed tomography demonstrates a single large mass in the right lobe of the liver, and serum -fetoprotein is markedly elevated. Which of the following is likely to have contributed to the patient's condition? (A) Aflatoxin B1 (B) C. sinensis (C) Hepatitis A (D) Polyvinyl chloride (E) Tetracycline

The answer is B. The association of episodic headache, palpitation, and diaphoresis, along with severe hypertension and hyperglycemia, is most suggestive of a catecholamine-secreting pheochromocytoma. Other nondiabetic endocrine disorders associated with hyperglycemia include Cushing syndrome, either pituitary or adrenal, with hypersecretion of corticotropin or cortisol; acromegaly, with hypersecretion of growth hormone; and hyperthyroidism, with hypersecretion of thyroxine.

A 38-year-old woman is found to have episodic headache, palpitation, and diaphoresis, along with severe hypertension. She is also found to have hyperglycemia, but diabetes mellitus has been ruled out. These findings suggest an endocrine tumor secreting which of the following hormones? (A) Antidiuretic hormone (ADH) (B) Catecholamines (C) Insulin (D) PTH (E) Prolactin

The answer is D. The child exhibits signs of congenital hypothyroidism (cretinism). This disorder can result from many causes, including iodine deficiency, maldevelopment of the fetal thyroid, or transplacental transfer of antithyroid antibodies from a mother with autoimmune thyroid disease (a high possibility in this clinical case). The diagnosis of congenital hypothyroidism is confirmed in infants, as in adults, by decreased serum free T4 and increased TSH. If maternal antibody transfer is suspected, then tests for antithyroid antibody testing can be performed in both the mother and child.

A 4-month-old child is brought to the pediatrician for evaluation. The mother received no prenatal care and states that she has "a thyroid condition." The child appears markedly developmentally delayed, with coarse features, macroglossia, and an umbilical hernia. The child likely has which of the following conditions? (A) Cushing disease (B) Acromegaly (C) Diabetes insipidus (D) Cretinism (E) Thyroglossal duct cyst

The answer is E. The clinical picture is that of Duchenne muscular dystrophy, the most common and most severe of the muscular dystrophies. This X-linked disorder is characterized by failure of synthesis of dystrophin, most often because of deletion of one or many exons in the DMDgene. Patients manifest with proximal muscle weakness, progressing to muscle necrosis. Serum CK is markedly increased. Compensatory hypertrophy is followed by pseudohypertrophy, in which necrotic muscle is replaced by fat and connective tissue. Most patients become wheelchair-bound and die of respiratory or cardiac failure in their late teenage years or in their early twenties.

A 4-year-old boy develops weakness of proximal lower back and extremity muscles, manifested by lordosis, a waddling gait, and the need to push on his knees in order to stand (Gower sign). Examination reveals proximal muscle weakness and bilateral enlargement of the calves. His younger brother has begun to display similar findings, as has his older half-brother, who has the same mother. Serum CK is markedly elevated. Which of the following is characteristic of this disorder? (A) Aberrant protein coded by a very small gene sequence on the Y chromosome (B) Autosomal dominant mode of inheritance (C) Mitochondrial inheritance (D) Regression of findings in late adolescence and adult life (E) Total absence or marked decrease of an important gene product

The answer is C. Classic hemophilia (factor VIII deficiency) is an abnormality of the intrinsic pathway of coagulation proximal to the final common pathway, which begins at factor X →Xa activation. This defect leads to a prolonged APTT. The other laboratory tests listed remain normal, because the bleeding time is a measure of platelet plug formation, the PT a measure of the extrinsic pathway of coagulation, and the thrombin time an assay of the conversion of fibrinogen to fibrin. The presumptive diagnosis is confirmed by specific factor VIII assay.

A 4-year-old boy presents with recurrent joint pain involving the knees and hips. He had always bruised easily, and recently the parents had seen blood in his urine. A presumptive diagnosis of classic hemophilia (hemophilia A) is made, and coagulation blood tests are performed. Which of the following is the most likely set of findings of coagulation screening tests? (A) Normal bleeding time, platelet count, and thrombin time; prolonged PT and APTT (B) Normal bleeding time, platelet count, thrombin time, and APTT; prolonged PT (C) Normal bleeding time, platelet count, thrombin time, and PT; prolonged APTT (D) Normal platelet count and thrombin time; prolonged bleeding time, PT, and APTT (E) Prolonged bleeding time, PT, APTT, and thrombin time; decreased platelet count

The answer is E. Minimal change disease (lipoid nephrosis) is the prototype of the nephrotic syndrome in children, who usually respond well to steroid therapy. Minimal change disease is characterized grossly by lipid-laden renal cortices. Light microscopy reveals normal glomeruli, but electron microscopy demonstrates the fusion of epithelial foot processes.

A 4-year-old boy presents with severe proteinuria, hypoalbuminemia, generalized edema, and hyperlipidemia. The patient improves on an empiric trial of corticosteroids, with complete resolution of proteinuria. Which of the following is the most likely diagnosis? (A) Diabetic nephropathy (B) Focal segmental glomerulosclerosis (C) Lupus nephropathy (D) Membranous glomerulonephritis (E) Minimal change disease

The answer is C. This patient has cystic fibrosis. Cystic fibrosis is an autosomal recessive disease caused by mutations in the CFTR gene, which encodes a protein that functions as a chloride channel. Pseudomonas aeruginosais the most likely pathogen causing chronic pulmonary infection and pulmonary failure, and is the leading cause of death in patients with cystic fibrosis. P. aeruginosais also (after S. aureusand E. coli) the leading cause of nosocomial (hospital-acquired) infections and a frequent cause of death from burns.

A 4-year-old girl has had multiple hospitalizations for pneumonia. Additionally, she is small for her age and has had symptoms of fat malabsorption. Her father refers to her as his "little potato chip" because she tastes salty when he kisses her. Chest radiograph demonstrates pulmonary infiltrates indicative of pneumonia. Which of the following is the most likely pathogen causing pulmonary infection in this patient? (A) Legionella pneumophila (B) Haemophilus influenzae (C) Pseudomonas aeruginosa (D) Staphylococcus aureus (E) Streptococcus pneumoniae

The answer is B. The vignette is consistent with a hematologic diagnosis of acute lymphoblastic leukemia, a condition that occurs with markedly increased incidence in association with Down syndrome.

A 4-year-old girl with a known chromosomal defect is seen because of fever and a skin rash. The rash consists of numerous small petechial hemorrhages with predominant involvement of the lower extremities. Her platelet count is markedly reduced, and the total white count is markedly increased. A blood smear and bone marrow aspirate reveal large numbers of undifferentiated blast cells, which by flow cytometry are found to be positive for the CD10 antigen. This complication occurs most frequently in association with which one of the following chromosomal disorders? (A) Cri du chat syndrome (B) Down syndrome (C) Fragile X syndrome (D) Klinefelter syndrome (E) Turner syndrome

The answer is A. In DIC, widespread thrombosis activates the fibrinolytic system, with degradation of both fibrin and fibrinogen; therefore, fibrin and fibrinogen degradation products are markedly increased. Since platelets are consumed, thrombocytopenia, not thrombocytosis, is an expected finding. In addition, the consumption of coagulation factors results in prolongation of the PT, a measure of the extrinsic pathway of coagulation; the APTT, a measure of the intrinsic pathway of coagulation; and the thrombin time, a measure of fibrinogen concentration. The clinical history strongly suggests premature separation of the placenta (abruptio placentae), a well-known cause of DIC.

A 40-year-old woman in the 30th week of gestation presents to the emergency room because she has vaginal bleeding and lower abdominal pain. The uterus is tender to palpation and there are signs of fetal distress. Because of hematuria and rectal bleeding, disseminated intravascular coagulation (DIC) is suspected. Which of the following findings would be most supportive of the diagnosis of DIC? (A) Increased fibrin degradation products (B) Decreased activated partial thromboplastin time (APTT) (C) Decreased prothrombin time (PT) (D) Normal thrombin time (E) Thrombocytosis

The answer is A. ITP (immune) is a chronic disease in adults, presumably caused by antibodies that bind to the cell surface of platelets.

A 40-year-old woman presents with a "skin rash." Questioning reveals easy bruising on minimal trauma, menorrhagia, and frequent bouts of epistaxis. She is not taking any medications, and there is no history of toxic exposures. Physical examination reveals multiple petechial hemorrhages, most prominently on the dependent portions of the lower extremities. Splenomegaly is not detected. Laboratory studies reveal marked thrombocytopenia, and a bone marrow aspiration reveals increased numbers of megakaryocytes. Which of the following is the most likely mechanism of this disorder? (A) Antibody-mediated platelet destruction (B) DIC, with consumption of platelets and coagulation factors (C) Intravascular spontaneous lysis of platelets due to increased osmotic fragility (D) Myeloid stem cell suppression in the bone marrow, with inability to produce platelets (E) Physical destruction of platelets while negotiating through partially blocked microvasculature

The answer is D. Approximately 80% to 90% of salivary gland tumors originate in the parotid gland and, of these, approximately 70% are pleomorphic adenomas. The term "mixed tumor" properly applies to this benign tumor, which often demonstrates myxoid and cartilage-like elements in addition to stellate or fusiform epithelial cells. Complete surgical resection is difficult because of the tumor's proximity to the facial nerve, and, thus, recurrence is frequent.

A 40-year-old woman presents with a painless mass anterior to her left ear. The mass had been slowly enlarging over the past year. The mass is firm and nontender. Computed tomography and magnetic resonance imaging reveal a well-circumscribed, homogeneous mass within the left parotid gland. Biopsy reveals anastomosing strands of stellate and fusiform epithelial cells embedded in a myxoid stroma. Which of the following is a characteristic of the lesion? (A) It is also called papillary cystadenoma lymphomatosum. (B) It is most often localized to the submandibular gland. (C) It is the most common malignant salivary gland tumor. (D) Recurrence often takes place after surgical resection. (E) Surgical resection should not be performed, because this condition is usually already metastatic on diagnosis.

The answer is B. The clinical presentation is illustrative of acanthosis nigricans, a cutaneous lesion that may be associated not only with diabetes and other endocrinopathies, but also with visceral malignancy, such as carcinoma of the stomach, lung, breast, or uterus. Other associations of malignancy include migratory venous thrombosis, which is also associated with visceral malignancies; clubbing of the fingers, which may be associated with a number of disorders, including carcinoma of the lung; and marantic endocarditis, which is associated with wasting diseases, such as widespread cancer.

A 40-year-old woman presents with grayish pigmentation of the skin in a large region covering her entire posterior neck and axillae. The hyperpigmented areas started out as smaller macules but have now progressed to form palpable plaques. At times the hyperpigmented areas are pruritic. Which of the following is an important association of this skin lesion? (A) Hypercholesterolemia (B) Marker of visceral malignancy (C) Tends to recur after resection (D) Convulsions, mental retardation, and retinal detachment (E) Viral infection

The answer is B. Myasthenia gravis is an autoimmune disorder caused by autoantibodies to postsynaptic acetylcholine receptors of the neuromuscular junction. The disease commonly presents as ptosis, diplopia, and difficulty chewing, speaking, or swallowing. Respiratory failure from diaphragmatic weakness can occur. Marked improvement with cholinesterase inhibitors is characteristic. For unexplained reasons, myasthenia gravis is associated with thymic hyperplasia or thymoma.

A 40-year-old woman presents with ptosis, diplopia, and dysarthria that fluctuate in intensity and tend to worsen as the day progresses. Edrophonium (an acetylcholinesterase inhibitor) is administered, and, after a minute, there is a striking improvement in muscle strength. Which of the following is characteristic of this disorder? (A) Tangles of small rod-shaped granules in type I muscle fibers (B) Autoantibodies to acetylcholine receptors (C) Multiple CTG trinucleotide repeats (D) Hypothalamic dopamine-mediated inhibition (E) Paraneoplastic manifestation of small cell lung cancer

The answer is A. This is a case of rheumatoid arthritis. Rheumatoid arthritis is a chronic inflammatory autoimmune disorder that primarily affects the synovium, especially in the proximal interphalangeal and metacarpophalangeal joints of the hands. Ulnar deviation of the fingers results from synovitis of the ligaments. In the synovium, an acute inflammatory reaction is followed by hyperplasia and hypertrophy of the synovial lining cells, granulation tissue (pannus) development over the articular cartilage, and scarring, contracture, and deformity from chronic inflammation.

A 40-year-old woman presents with symmetric stiffness and swelling of the proximal interphalangeal joints of the hands and of the metacarpophalangeal joints, as well as fatigue, malaise, and myalgia. Symptoms are worse in the morning or after extended inactivity and improve with movement and use throughout the day. Ulnar deviation of the fingers symmetrically on both hands is noted. Which of the following is the most likely etiology of this condition? (A) Autoimmune disease (B) Mechanical injury ("wear and tear") (C) Neisseria gonorrhoeaeinfection (D) Secondary manifestation of chronic lung disease or cyanotic cardiac disease (E) Urate crystal deposition

The answer is D. Meningioma is the second most common primary intracranial neoplasm of adults, second only to glioblastoma multiforme. Because this slowly growing benign tumor arises from the arachnoidal cells of the meninges, it is actually external to the brain and is frequently amenable to surgical resection and complete cure.

A 40-year-old woman who has had progressive localizing signs of CNS compression fully recovers following resection of an intracranial neoplasm. These clinical findings are highly suggestive of a specific diagnosis. Assuming that this diagnosis is correct, which of the following is most characteristic? (A) Extracranial metastases (B) "Fried egg" appearance of tumor cells (C) Multiple areas of necrosis and hemorrhage within the tumor (D) Origin in arachnoidal cells of the meninges (E) Tumor cells arranged in a rosette pattern

The answer is B. Cardiomyopathies are noninflammatory myocardial disorders that are not associated with coronary artery obstruction, hypertension, valvular disease, congenital heart disease, or infectious disease. They are most often characterized by otherwise unexplained ventricular dysfunction, such as cardiac failure, ventricular enlargement, or ventricular arrhythmias.

A 42-year-old man is seen because of a long history of slowly developing congestive heart failure. His blood pressure is normal. Coronary artery angiography reveals no vascular disease. No heart murmurs are heard. The white blood cell count, differential, and erythrocyte sedimentation rate are normal. The most likely diagnosis is (A) carcinoid heart disease. (B) cardiomyopathy. (C) coarctation of the aorta. (D) constrictive pericarditis. (E) myocardial infarction.

The answer is B. C. trachomatisis one of the leading causes of urethritis and should be suspected whenever a gonorrhea-like discharge fails to show gram-negative diplococci within neutrophilic phagocytes. It is a sexually transmitted disease that can also cause epididymitis. Bowen disease presents as a single erythematous plaque on the penis or scrotum and may evolve into invasive carcinoma. Herpes simplex virus can cause a vesicular rash on the penis. T. pallidumcauses syphilis, which may present with a painless chancre on the penis.

A 43-year-old businessman visits his primary care physician complaining of a purulent penile discharge. He had unprotected sex with a woman he met at a conference 1 week ago. Gram stain of the discharge fails to reveal any organisms. Which of the following is the most likely cause of the discharge? (A) Bowen disease (B) Chlamydia trachomatis (C) Herpes simplex virus (D) Neisseria gonorrhoeae (E) Treponema pallidum

The answer is A. This patient presents with the classic signs of cholecystitis, or inflammation of the gallbladder wall, which is usually due to obstruction of the cystic duct by gallstones. Carcinoma of the ampulla of Vater is closely related to carcinoma of the extrahepatic biliary duct, and presents with jaundice and a palpably enlarged gallbladder, as opposed to stones, which typically do not cause an enlarged gallbladder. Cholangiocarcinoma is a malignancy of the bile ducts. Cholesterolosis, also known as strawberry gallbladder, is noninflammatory in nature. Sclerosing cholangitis is an inflammatory condition of the extrahepatic bile ducts that is often associated with chronic ulcerative colitis.

A 43-year-old multigravida presents with nausea, vomiting, fever, and right upper quadrant pain. On examination, she displays arrested inspiration on palpation of the right upper quadrant (Murphy sign). Her laboratory results reveal neutrophilia with a "left shift." Which of the following is the most likely diagnosis? (A) Acute cholecystitis (B) Carcinoma of the ampulla of Vater (C) Cholangiocarcinoma (D) Cholesterolosis (E) Sclerosing cholangitis

The answer is B. Urolithiasis (calculi or stones in the urinary tract) is a very common problem, especially in men. Calcium stones (composed of calcium oxalate or calcium phosphate, or both) account for 80% to 85% of urinary stones. Such stones are associated with hypercalciuria, which in turn is caused by increased intestinal absorption of calcium, increased primary renal excretion of calcium, or hypercalcemia. Ammonium magnesium phosphate stones are associated with ammonia-producing (urease-positive) bacteria, such as Proteus vulgarisor Staphylococcus. Uric acid stones are associated with hyperuricemia in 50% of patients and can occur secondary to gout or increased cell turnover (leukemias or myeloproliferative syndromes). Cystine stones are almost always associated with cystinuria, a genetically determined aminoaciduria.

A 45-year-old man comes to the emergency department in obvious severe pain. He states that he has severe pain in his right flank that extends down to his right groin. The pain is sharp and severe, and it started several minutes earlier. An abdominal radiograph demonstrates the presence of right ureteral urolithiasis. Which of the following is the most likely composition of this patient's stone? (A) Ammonium magnesium phosphate (B) Calcium (C) Cystine (D) Uric acid

The answer is E. This is a classic case of GERD, which is caused by reflux of gastric acid contents into the lower esophagus. GERD manifests as burning epigastric pain on eating spicy foods or on lying recumbent. The pain is usually relieved by antacids. GERD is most commonly associated with hiatal hernia and an incompetent lower esophageal sphincter, as well as with excessive use of alcohol or tobacco, increased gastric volume, pregnancy, and scleroderma. Barrett esophagus, or columnar intestinal metaplasia of the epithelium of the distal esophagus, is a complication of long-standing GERD.

A 45-year-old man complains of "heartburn" and burning epigastric pain, relieved by antacids and triggered by eating spicy or acidic foods or by assuming a recumbent position. The patient smokes two packs of cigarettes a day and consumes several alcoholic drinks each evening. Which of the following is the usual cause of this patient's condition? (A) Columnar intestinal metaplasia of esophageal squamous epithelium (B) Excessive acid production in the stomach (C) Excessive NSAID use (D) H. pylori infection (E) Hiatal hernia and incompetent lower esophageal sphincter

The answer is B. C3b is an important opsonin. C3a and C5a (anaphylatoxins) mediate degranulation of mast cells and basophils. In addition, C5a is a potent chemotactic agent for neutrophils. C5b-9 is the membrane attack complex that mediates complementinduced cell lysis.

A 45-year-old man presents with a form of bacterial infection in which the invading microorganisms are opsonized prior to their engulfment by phagocytic cells. Of the following complement components or groups of components, which is most likely involved? (A) C3a and C5a (B) C3b (C) C4a (D) C5a alone (E) C5b-9

The answer is A. Androblastoma (Sertoli cell tumor), a nongerm cell tumor derived from the sex cord, is most often benign. These tumors, along with Leydig (interstitial) cell tumors, may be hormone-producing, sometimes elaborating estrogens and presenting in males with gynecomastia and other feminizing effects.

A 45-year-old man presents with breast enlargement, erectile dysfunction, and decreased libido. On physical examination, an intratesticular mass is palpated. Which of the following is the most likely diagnosis? (A) Androblastoma (Sertoli cell tumor) (B) Endodermal sinus tumor (C) Mature teratoma (D) Mixed germ cell tumor (E) Seminoma

The answer is C. This is a case of Huntington disease, which is an autosomal dominant, fatal, progressive degeneration and atrophy of the striatum (caudate nucleus and putamen). The disorder is characterized by an increased number of trinucleotide (CAG) repeats in the HD (huntingtin) gene on the short arm of chromosome 4. Degeneration of the upper and lower motor neurons is characteristic of ALS. Dopamine depletion and depigmentation of the substantia nigra is characteristic of Parkinson disease. Neurofibrillary tangles and amyloid plaques are found in Alzheimer disease. Pick bodies can be found in Pick disease, which clinically resembles Alzheimer disease.

A 45-year-old man presents with involuntary facial grimaces and movements of the fingers. His mother had had similar symptoms beginning at about the same age. Her disorder had progressed to dancing movements, writhing of the arms and legs, and eventually coma and death. His maternal grandfather had had a similar disorder but at an age older than the mother. Which of the following is most characteristic of this disease? (A) Degeneration of upper and lower motor neurons (B) Dopamine depletion and depigmentation of the substantia nigra (C) Increased number of trinucleotide repeats in a gene on chromosome 4 (D) Neurofibrillary tangles and amyloid plaques in the cerebral cortex (E) Pick bodies, characterized by round intracytoplasmic inclusions consisting of neurofilaments

The answer is C. Primary biliary cirrhosis is an autoimmune condition that typically presents in middle-aged women. The itching and hypercholesterolemia are secondary to severe obstructive jaundice. Leptospirosis is a condition caused by a treponemal bacterium that results in jaundice, renal failure, and hemorrhagic phenomena. Macronodular cirrhosis is usually a result of hepatitis B or hepatitis C infection. Primary sclerosing cholangitis is associated with ulcerative colitis and with an increased incidence of cholangiocarcinoma. Secondary biliary cirrhosis is caused by extrahepatic biliary obstruction.

A 45-year-old woman presents to her primary care physician with jaundice, pruritus, and periocular and intradigital xanthomas. Her laboratory results indicate a significantly increased alkaline phosphatase as well as a positive test for antimitochondrial antibodies. The most likely cause of her symptoms is (A) leptospirosis. (B) macronodular cirrhosis. (C) primary biliary cirrhosis. (D) primary sclerosing cholangitis. (E) secondary biliary cirrhosis.

The answer is B. Symmetric polyarthritis with involvement of the proximal interphalangeal and metacarpophalangeal joints in a female patient are characteristics of rheumatoid arthritis. Rheumatoid factor, an IgM antibody directed against the Fc portion of IgG, is found in about 80% of affected individuals.

A 45-year-old woman presents with an insidious and progressive syndrome characterized by pain and tenderness in multiple joints, with joint stiffness on rising in the morning, and early afternoon fatigue and malaise. Joint involvement is symmetric, with the proximal interphalangeal and metacarpophalangeal joints especially involved. Physical examination reveals tenderness in nearly all inflamed joints. Which of the following laboratory abnormalities is most likely? (A) Antibodies to double-stranded DNA (B) IgM anti-IgG antibodies (C) Urate crystals and neutrophils in synovial fluid (D) Anti-DNAase B (E) HLA-B27 antigen

The answer is A. CML is almost invariably marked by the finding of the Philadelphia chromosome, a small residual chromosome 22 with the addition of a small segment of chromosome 9, resulting from a 9;22 translocation.

A 45-year-old woman presents with marked splenomegaly. Her leukocyte count is increased to 300,000/µL. The differential count reveals the presence of small numbers of myeloblasts and promyelocytes, with a predominance of myelocytes, metamyelocytes, bands, and segmented neutrophils. Basophils are also increased in number, as are platelets. The patient is not anemic. Leukocyte alkaline phosphatase is decreased. Which of the following describes a major characteristic of this disorder? (A) 9;22 translocation (B) Expansion of mature B lymphocytes within multiple lymph nodes (C) Hypogammaglobulinemia (D) Neoplastic cells exhibiting hair-like filamentous projections (E) Peak incidence at 65 years of age

The answer is A. Sjögren syndrome is an autoimmune disorder characterized by keratoconjunctivitis sicca and xerostomia, due to lymphocytic infiltration and parenchymal destruction of the parotid and lacrimal glands in association with a connective tissue disorder, such as rheumatoid arthritis. Malignant lymphoma is a frequent complication.

A 45-year-old woman with long-standing rheumatoid arthritis complains of dry eyes and dry mouth. Bilateral enlargement of the parotids is noted on physical examination. The syndrome described here is best described as (A) autoimmune. (B) infectious. (C) metabolic. (D) metastatic. (E) primary neoplastic.

The answer is E. This woman has an adenocarcinoma. These are the most likely lung cancers to arise in never-smokers and are more common in women. They have been associated with EGFRmutations, and EGFRmutation testing is rapidly becoming the standard-of-care for lung adenocarcinomas in order to direct chemotherapy (EGFRmutated cases may respond to targeted tyrosine kinase inhibitors). Adenocarcinomas may be preceded by or associated with atypical adenomatous hyperplasia, not squamous dysplasia. Unlike small cell carcinomas, they do not show neuroendocrine features such as synaptophysin staining and metastasis at the time of presentation is not the rule.

A 45-year-old woman with no smoking history presents with new onset of cough, shortness of breath, and weight loss. Imaging reveals a peripherally located lung mass with no evidence of primary tumor elsewhere in the body. On light microscopy, the tumor is comprised of poorly formed glands. Which of the following is most likely to be true of this tumor? (A) Positive for synaptophysin immunostaining (B) Associated squamous dysplasia at the periphery of the tumor (C) Most likely already metastatic, therefore surgery is not recommended (D) Driven by tobacco-associated carcinogens (E) Positive for mutation in EGFR

The answer is C. The findings are those of acromegaly, which is caused by a pituitary somatotropic adenoma. Growth hormone excess causes elevation in concentration of IGF-I (somatomedin C), measurement of which is a reliable indicator of disease activity. The tumor can also produce local effects, the most common of which is bitemporal hemianopsia from pressure on the optic chiasm.

A 46-year-old man is referred to an endocrinologist because of the recent onset of diabetes mellitus. His overall appearance is striking, however, and on questioning, he describes marked changes that have been occurring slowly over many years. Comparison with old photographs reveals that he has developed generalized coarseness of his facial features, including frontal bossing, thickening of the nose, prognathism (enlargement and increased prominence of the jaw), and macroglossia (enlargement of the tongue). In addition, he has enlarged extremities, with sausage-like fingers, and he says that he is no longer able to wear his wedding ring and that his shoe size has increased. These findings are characteristic of increased activity of which of the following? (A) Corticotropin (B) Dopamine (C) Insulinlike growth factor-I (D) Prolactin (E) Thyroid stimulating hormone

The answer is E. Tuberculosis, at one time a frequent hazard in the United States, is now relatively uncommon except in immunocompromised individuals and persons returning from parts of the world where the disease remains a common problem. Primary tubercu-losis is the initial infection by M. tuberculosis, and is restricted to the primary, or Ghon, complex, a combination of a peripheral subpleural parenchymal lesion and involved hilar lymph nodes. Cavitation and selective localization to the pulmonary apices are characteristics of secondary tuberculosis. Secondary tuberculosis may spread through the lymphatics and blood to other organs, resulting in miliary tuberculosis.

A 46-year-old woman presents with fever, hemoptysis, weight loss, and night sweats. She has never smoked. She recently returned from a month-long trip to Asia. A chest radiograph reveals apical lesions with cavitation in the left lung. A purified protein derivative (PPD) test is placed, and 48 hours later an 18-mm wheal develops. Sputum cultures reveal numerous acid-fast organisms. This patient is put on contact precautions, and a regimen for tuberculosis is started. Which of the following disorders does this patient most likely have? (A) Acquired immunodeficiency syndrome (B) Congenital immunodeficiency (C) Miliary tuberculosis, with seeding of distal organs with innumerable small millet seed-like lesions (D) Primary tuberculosis, characterized by the Ghon complex (E) Secondary tuberculosis, resulting from activation of a prior Ghon complex, with spread to a new pulmonary site

The answer is A. GTPase activity, which is required for inactivation, is decreased in mutant ras (p21) proteins. This change is mediated by reduced responsiveness to GTPase activating protein.

A 48-year-old man with early-stage colon cancer undergoes a partial resection of the colon. Molecular analysis reveals that the tumor tissue harbors a mutation in codon 12 of the rasoncogene. The mutant gene codes for a ras protein product that has (A) decreased GTPase activity. (B) decreased reverse transcriptase activity. (C) increased protein phosphatase activity. (D) increased responsiveness to growth f actors. (E) increased tyrosine kinase activity.

The answer is E. Small cell carcinoma of the lung is almost always metastatic at the time of initial diagnosis and is thus poorly amenable to surgery. Despite morphologic differences, it is thought that all lung carcinomas, including small cell carcinoma, share a common endodermal origin. The location is most often central rather than peripheral, and there is a marked association with cigarette smoking. Paraneoplastic syndromes include inappropriate secretion of ACTH and ADH. Secretion of a protein with PTH-like activity is an association of squamous cell lung carcinoma.

A 49-year-old man has a recent diagnosis of small cell carcinoma of the lung. Which of the following is an important characteristic of this form of lung cancer? (A) Ectodermal origin (B) Frequent peripheral location (C) Less association with cigarette smoking than other forms of lung cancer (D) Paraneoplastic hyperparathyroidism (E) Poorly amenable to surgery

The answer is D. The clinical description is that of RPGN, which is defined as the nephritic syndrome that progresses rapidly to fulminant renal failure within weeks or months. RPGN is classified into three categories: antiglomerular basement membrane (anti-GBM) antibody disease, immune complex deposit disease, and pauciimmune disease (ANCApositive). Characteristic histologic findings in RPGN include crescents between the Bowman capsule and the glomerular tuft. In approximately 50% of cases of RPGN, the disease is of poststreptococcal etiology. It should be noted, however, that the most common outcome of poststreptococcal glomerulonephritis is complete recovery, and only a small minority of patients progress to RPGN.

A 5-year-old boy diagnosed with poststreptococcal glomerulonephritis was admitted to the hospital several weeks ago. Over the last several weeks, his clinical state has not improved. Severe oliguria has developed, his serum creatinine has continued to rise, and his glomerular filtration rate has decreased by 50% since his admission to the hospital. Which of the following is the most likely diagnosis? (A) Alport syndrome (B) Membranoproliferative glomerulonephritis (C) Membranous glomerulonephritis (D) Rapidly progressive glomerulonephritis (E) Renal papillary necrosis

The answer is E. The most frequent cause of cardiac rupture is MI. This complication, which often results in hemopericardium and cardiac tamponade, occurs with peak incidence within 4 to 10 days after infarction.

A 55-year-old woman who died suddenly at home was found at autopsy to have suffered a rupture of the left ventricle. Which one of the following myocardial changes is the most frequent cause of this catastrophic event? (A) Abscess formation and tissue destruction due to infective endocarditis (B) Fatty change due to interaction of diphtheria exotoxin and carnitine (C) Inflammation associated with Aschoff bodies (D) Inflammation due to coxsackie B infection (E) Necrosis due to coronary artery obstruction

The answer is E. The clinical description is that of poststreptococcal glomerulonephritis, the prototype of the nephritic syndrome. An antecedent infection, usually of the pharynx or skin, with a group A β-hemolytic streptococcus occurs approximately 1 to 2 weeks before the onset of the renal manifestations. Poststreptococcal glomerulonephritis is an immune complex disease, with antigen-antibody-complement complexes localizing to the outside (subepithelial side) of the glomerular basement membrane. An intense inflammatory response is elicited, including chemotactic attraction of neutrophils, release of lysosomal enzymes, partial destruction of basement membrane, and bleeding into the Bowman space.

A 5-year-old boy presents with "teacolored urine," oliguria, and periorbital edema. He had a sore throat 2 weeks ago that had resolved before his parents sought medical treatment. The patient is found to be hypertensive. Urea nitrogen and creatinine are elevated. Antistreptolysin O titer and anti-DNAase B titer are also elevated. Urine tests are positive for blood and red cell casts. Which of the following is the most likely mechanism for this patient's condition? (A) Acute bacterial infection of the kidneys (B) Amyloid deposits (C) ANCA-positive vasculitis (D) Antibody against glomerular basement membrane antigens (E) Immune complex deposits

The answer is C. IgA nephropathy is the most common glomerular disease and is defined by mesangial deposition of IgA. In its most typical form, the clinical manifestations are limited to a few days of hematuria following an infection; however, there are many etiologic factors, and the condition can vary from one of little import to a severe form of renal disease.

A 5-year-old boy presents with hematuria. His mother states that he has had a sore throat for the past 2 days and that he has had hematuria a few times in the past, also concomitantly with a sore throat. She states that his urine usually returns to a normal clear yellow color after a few days. Which of the following is the most likely diagnosis? (A) Alport syndrome (B) Goodpasture syndrome (C) IgA nephropathy (D) Membranoproliferative glomerulonephritis (E) Poststreptococcal glomerulonephritis

The answer is A. Kwashiorkor is a form of protein-calorie malnutrition attributed to a relative lack of protein despite a diet relatively high in carbohydrates. Marasmus, in contrast, is more severe, and results from a major lack of calories from any source. Carbohydrate metabolites are converted to lipid, which is processed and stored by the liver; however, protein sources to serve as precursor amino acids for apolipoprotein synthesis are lacking. The result is hepatic fatty change, not cirrhosis.

A 5-year-old child in a refugee camp in sub-Saharan Africa is seen by a volunteer doctor in the camp and is diagnosed as having severe protein-calorie malnutrition consistent with kwashiorkor. Which of the following is a characteristic of this disorder? (A) Impaired apolipoprotein synthesis (B) Increased fat storage in adipocytes (C) More severe than marasmus (D) Myocardial fatty change exceeds that in liver (E) Precursor to cirrhosis

The answer is B. A young patient presenting with precocious puberty and an ovarian mass is likely to have a granulosa cell tumor. Call-Exner bodies, small follicles filled with eosinophilic secretion, are an important diagnostic feature. Signet-ring cells are seen in Krukenberg tumors (bilateral metastatic involvement of the ovaries by a signet-ring carcinoma arising in the stomach). Schiller-Duval bodies are seen in dysgerminoma, an ovarian tumor analogous to the male seminoma. Hyperfunctional thyroid tissue is found in struma ovarii, and the Brenner tumor resembles transitional cell epithelium.

A 5-year-old girl begins developing secondary sexual characteristics, including pubic hair growth and breast development. Her concerned parents bring her to the pediatrician for evaluation. An abdominal ultrasound examination reveals an ovarian mass. The mass is resected, and pathologic examination reveals it to be a granulosa cell tumor. Which of the following histologic findings is a characteristic of this type of ovarian tumor? (A) Signet-ring cells (B) Call-Exner bodies (C) Schiller-Duval bodies (D) Hyperfunctioning thyroid tissue (E) Transitional cell epithelium

The answer is A. Bronchial asthma, or hyperreactive airway disease, is a type of COPD caused by narrowing of airways. Asthma manifests morphologically by bronchial smooth muscle hypertrophy, hyperplasia of bronchial submucosal glands and goblet cells, and airways plugged by mucus-containing Curschmann spirals (whorl-like accumulations of epithelial cells), eosinophils, and Charcot-Leyden crystals (crystalloids of eosinophil-derived proteins).

A 50-year-old man dies of a respiratory illness that had been characterized by dyspnea, cough, and wheezing expiration of many years' duration. Initially episodic, his "attacks" had increased in frequency and at the time of death had become continuous and intractable. At autopsy, which of the following is the most likely histologic finding in the lungs? (A) Bronchial smooth muscle hypertrophy with proliferation of eosinophils (B) Diffuse alveolar damage with leakage of protein-rich fluid into alveolar spaces (C) Dilation of air spaces with destruction of alveolar walls (D) Hyperplasia of bronchial mucussecreting submucosal glands (E) Permanent bronchial dilation caused by chronic infection, with bronchi filled with mucus and neutrophils

The answer is B. DIC is characterized by widespread clotting with resultant consumption of platelets, coagulation factors, and fibrinogen, and secondary activation of the fibrinolytic system. Laboratory studies reveal thrombocytopenia; prolonged bleeding time, PT, APTT, and thrombin time (reflecting decreased fibrinogen); and increased fibrin and fibrinogen split products. In addition, DIC is often marked by microangiopathic hemolytic anemia with circulating fragmented red cells.

A 50-year-old man has been in the medical intensive care unit for septic shock for the past few days. He has now developed rectal bleeding, epistaxis, and gingival bleeding. DIC is suspected. Which of the following sets of results for a panel of screening tests is most consistent with this diagnosis? (A) Normal bleeding time, PT, APTT, thrombin time, and platelet count (B) Prolonged bleeding time, PT, APTT, and thrombin time; reduced platelet count (C) Prolonged PT and APTT; normal bleeding time, platelet count, and thrombin time (D) Prolonged PT and APTT; reduced platelet count; normal bleeding time and thrombin time (E) Prolonged bleeding time, PT, and APTT; normal platelet count and thrombin time

The answer is C. Epidural hematoma is most often caused by skull fracture with laceration of the branches of the middle meningeal artery. The classic CT scan finding is formation of a "lens-shaped" density contained within the suture lines of the skull.

A 50-year-old man is seen in the emergency room. He sustained a head injury after falling from a ladder. He had a brief period of loss of consciousness after the fall, which then resolved. He now has headache, vomiting, and borderline state of consciousness. Emergency CT demonstrates a fracture in the temporoparietal area of the skull, as well as a lens-shaped homogeneous density contained within the suture lines. Which of the following is the most likely diagnosis? (A) Bruising of the brain substance of the cerebral hemisphere (B) Enlargement of the cerebral ventricles (C) Epidural hematoma (D) Subdural hematoma (E) Subarachnoid hemorrhage

The answer is A. Mycosis fungoides is a T-cell lymphoma characterized by a rash that may be sited at any cutaneous location. Atypical CD4+T cells with cerebriform nuclei are found on biopsy. The disorder may remain localized to the skin for many years, but the neoplastic cells eventually disseminate to lymph nodes and other organs. Sézary syndrome, the leukemic form of this cutaneous T-cell lymphoma, is characterized by the combination of skin lesions and circulating neoplastic cells.

A 50-year-old man presents because of a pruritic rash of several years' duration. The rash is characterized by erythematous, eczematoid patches, and raised plaques and is distributed asymmetrically over the chest and abdomen. Biopsy of the plaques reveals atypical CD41 T cells with cerebriform nuclei. Further marker studies lead to a diagnosis of mycosis fungoides. Which of the following is true of this disease? (A) The disease eventually disseminates to lymph nodes and internal organs. (B) The neoplastic cells most commonly display cell markers of CD19 and CD20. (C) The skin rash most commonly disappears over time. (D) This disease is caused by a chronic fungal infection in the skin. (E) This is a benign condition and no further workup is necessary.

The answer is B. Huntington disease, a disorder caused by an increased number of trinucleotide repeats, is manifested anatomically by progressive degeneration and atrophy of the caudate nucleus, putamen, and frontal cortex. The caudate nucleus normally bulges convexly into the lateral ventricles; thus atrophy of the caudate nucleus results in the appearance of "bat-wing-shaped" or enlarged lateral ventricles.

A 50-year-old man presents with a neurologic disorder that began with choreoathetoid movements and has progressed to dementia. Other family members have been similarly affected. A CT scan reveals atrophy of the caudate nucleus, resulting in the appearance of enlarged lateral ventricles. Which of the following is the most likely diagnosis? (A) Alzheimer disease (B) Huntington disease (C) Idiopathic Parkinson disease (D) Myasthenia gravis (E) Wernicke-Korsakoff syndrome

The answer is E. This is a case of gout. Gout is characterized by deposition of monosodium urate crystals in joints and other tissues as a result of hyperuricemia. Gouty arthritis most commonly affects the metacarpophalangeal joint of the great toe (podagra). Primary gout, the most common type of gout, is characterized by hyperuricemia without evident cause. Secondary gout, much less common, is characterized by hyperuricemia due to evident cause, such as leukemia, multiple myeloma, myeloproliferative syndromes, or Lesch-Nyhan syndrome (HGPRT deficiency).

A 50-year-old man presents with recurrent episodes of acute excruciating pain, swelling, and redness of his right great toe, all following a large meal that was accompanied by copious consumption of alcohol. In the past, others in his family have had similar symptoms. The metatarsophalangeal joint of the right great toe is erythematous, edematous, and tender. Which of the following is the most likely etiology of this condition? (A) Autoimmune disease (B) Mechanical injury ("wear and tear") (C) N. gonorrhoeaeinfection (D) Secondary manifestation of chronic lung disease or cyanotic cardiac disease (E) Urate crystal deposition

The answer is A. This is a case of paradoxical embolism, which denotes the passage of an embolus of venous origin into the arterial circulation, by way of a right-to-left shunt (e.g., atrial septal defect or patent foramen ovale). Ordinarily, atrial septal defects result in a left-to-right shunt across the atrial septum, but over time may develop into a rightto-left shunt. The likelihood of right-to-left passage of an embolus is often enhanced by pulmonary hypertension, sometimes secondary to pulmonary thromboembolism.

A 50-year-old man presents with sudden weakness in his left leg. He has felt well lately and has no past medical history of coronary artery disease, hyperlipidemia, or hypertension, and no family history of myocardial infarction or stroke. Physical examination reveals motor weakness in the left leg, with no other neurologic deficits, and no cardiac murmur. Magnetic resonance imaging of the brain demonstrates a small ischemic infarct in the arterial distribution of the brain correlating with motor control of the left leg. Angiography and echocardiography reveal normal coronary arteries, normal valves with no vegetations, and a small right-to-left shunt. Which of the following is most likely associated with this scenario? (A) Atrial septal defect (B) Bacterial endocarditis (C) Nonbacterial thrombotic (marantic) endocarditis (D) Tetralogy of Fallot (E) Ventricular septal defect

The answer is C. The illustration shows nodular glomerulosclerosis (Kimmelstiel-Wilson nodules), the most characteristic glomerular finding in diabetes mellitus. The nodules are accumulations of mesangial matrix-like material.

A 50-year-old man with hypertension and the nephrotic syndrome undergoes a renal biopsy. The appearance of the biopsy is similar to that shown in the figure. Of the following possible additional laboratory findings, which one is most characteristically associated with this lesion? (A) Decreased creatinine clearance (B) Fixed specific gravity of urine (C) Hyperglycemia (D) Hypocalcemia (E) Increased blood urea nitrogen

The answer is D. Although too nonspecific for initial diagnosis or screening, CEA is useful for follow-up of cancer of the colon. Increased serum hCG may be observed in normal pregnancy, hydatidiform mole, choriocarcinoma, and many testicular mixed germ cell tumors. Fetal neural tube defects are associated with increased levels of AFP in the mother. In addition, elevated levels of AFP (unrelated to pregnancy) may be associated with hepatocellular carcinoma, yolk sac (endodermal sinus) tumors, and some nonseminomatous germ cell tumors of the testes. VMA is a marker for neuroblastoma and pheochromocytoma. Granulosa cell tumors and thecomas of the ovary characteristically produce estrogen.

A 50-year-old woman had a partial colectomy performed for adenocarcinoma of the sigmoid colon, with apparent complete and uneventful recovery. At follow-up visits, her physician is particularly interested in changes that may occur in which of the following laboratory measures? (A) Human chorionic gonadotropin (hCG) (B) α-fetoprotein (AFP) (C) Vanillylmandelic acid (VMA) (D) Carcinoembryonic antigen (CEA) (E) Estriol

The answer is B. A history of breast cancer in first-degree female relatives and a diet high in animal fats are important associations of breast cancer. Associations related to excess estrogen stimulation include high-fat diet, obesity, early menarche, and nulliparity. Additional associations are delayed first pregnancy and exogenous estrogen therapy.

A 50-year-old woman has a lumpectomy following mammographic discovery of a carcinoma of the breast. Which of the following is a well-known characteristic or association of breast cancer? (A) Low-fat diet (B) Positive family history (C) Excessive thinness (D) Late menarche (E) Multiparity

The answer is C. Pulmonary embolism most often originates from venous thrombosis in the lower extremities or pelvis. An embolus migrates through the venous circulation to the right heart and gets trapped in branches of the pulmonary artery. Pulmonary embolism occurs in clinical settings of venous stasis, such as primary venous disease, congestive heart failure, prolonged bed rest or immobilization, or prolonged sitting while traveling.

A 50-year-old woman has been immobilized in bed for several days after a motor vehicle accident. She had been improving, but this morning she suffered the sudden onset of pleuritic chest pain, hemoptysis, tachypnea, tachycardia, and dyspnea. What is the likely basis of this set of findings? (A) Arterial thrombus originating in pulmonary blood vessels (B) Arterial thrombus originating in the lower extremities with migration to pulmonary veins (C) Deep venous thrombus of the lower extremities with embolization to branches of the pulmonary artery (D) Mural thrombus originating in the left heart with migration to pulmonary blood vessels (E) Venous thrombus originating in pulmonary blood vessels

The answer is C. Diabetic nephropathy manifests clinically as the nephrotic syndrome; however, this syndrome is compounded by renal failure and hypertension. Ultrastructural changes include a marked increase in the thickness of the glomerular basement membrane and mesangial accumulation of glycosylated basement membranelike material. Light microscopy findings include diffuse glomerulosclerosis (a diffuse increase in mesangial matrix) and nodular glomerulosclerosis (nodular accumulations of mesangial matrix).

A 50-year-old woman with a 20-year history of type 2 diabetes mellitus presents with proteinuria, hypoalbuminemia, edema, and hyperlipidemia. She has not monitored her serum glucose levels over the past several years. Diabetic nephropathy is diagnosed. What is the classic morphologic finding in diabetic nephropathy? (A) Crescentic formation in glomeruli on light microscopy (B) Intramembranous and epimembranous immune complex deposits in the glomerular basement membrane on electron microscopy (C) Nodular accumulations of mesangial matrix on light microscopy (D) Sclerosis within capillary tufts that involves only some glomeruli and only parts of affected glomeruli on light microscopy (E) Wire-loop abnormalities from immune complex deposits and thickening of the glomerular basement membrane on light microscopy

The answer is C. The description is that of lymphocyte depletion Hodgkin lymphoma, the least frequently occurring form of Hodgkin lymphoma. It is marked by few lymphocytes, numerous Reed-Sternberg cells, and extensive necrosis and fibrosis. It often presents in an advanced stage and has the poorest prognosis of the Hodgkin lymphoma variants. This variant is associated with EBV infection in the great majority of cases and also is more common in persons infected with HIV.

A 50-year-old, HIV-positive man has Hodgkin disease. Lymph nodes on both sides of the diaphragm are involved, as are the liver and bone marrow. Histologic examination of an involved node reveals a diffuse infiltrate with large numbers of Reed-Sternberg cells with many bizarre sarcomatous variants. Tests for Epstein-Barr proteins are positive. From this description, which Hodgkin lymphoma variant is most likely? (A) Nodular lymphocyte predominant (B) Mixed cellularity (C) Lymphocyte depletion (D) Nodular sclerosis

The answer is E. The recommended screening test for hemochromatosis is serum transferrin saturation, which is increased. Further molecular testing can be performed to look for mutations in the Hfegene on chromosome 6. Early detection and occasional phlebotomy can prevent multiorgan failure attributed to iron accumulation in tissues. Antimitochondrial antibodies are used in the diagnosis of primary biliary cirrhosis. Decreased alpha-1-antitrypsin levels are associated with liver disease, as well as panacinar emphysema. Ceruloplasmin levels are decreased in Wilson disease. Heterophil antibodies occur in infectious mononucleosis due to EBV.

A 53-year-old man presents to his physician's office for an insurance physical. During the history, you learn that his father had diabetes and died of congestive heart failure. The patient was told it was due to "bronze diabetes." The patient complains only of vague fatigue and arthralgias. You are concerned that the patient may be at risk for hereditary hemochromatosis. Which test would you order to confirm your suspicion? (A) Antimitochondrial antibodies (B) alpha-1-antitrypsin (C) Ceruloplasmin (D) Heterophil antibodies (E) Transferrin saturation

The answer is A. The patient has pancreatic adenocarcinoma, a common malignancy of the pancreas. It is more common in smokers. Although chronic pancreatitis is a risk factor, there is not a straightforward link with alcohol use and indeed, the chronic pancreatitis most often associated with pancreatic cancer is not the conventional type observed in alcoholics. Pancreatic adenocarcinomas are most common in the head of the pancreas and are thought to arise in pancreatic dysplasia known as PanIN. PanIN usually shows mutations in KRASwhile subsequent mutations, often involving p53, p16,and DPC4/SMAD4are required for full-blown invasive cancer. Mucinous cystic neoplasms of the pancreas are relatively indolent tumors which are most common in the pancreatic tail and which usually show ovarian-type stroma, which is not a feature of pancreatic adenocarcinoma.

A 55-year-old alcoholic man presents with jaundice and recent weight loss. Imaging reveals a large mass in the head of his pancreas. Biopsy reveals an infiltrate of small malignant glands set in a background of prominent desmoplasia. Which of the following is true of this cancer? (A) PanIN represents a precursor lesion. (B) It most commonly arises in the tail of the pancreas. (C) It is not associated with smoking. (D) It is invariably associated with heavy drinking. (E) Ovarian-type stroma is common.

The answer is C. Malignant melanoma arises from melanocytes or nevus cells, is most often associated with excessive sun exposure, and is most common in fair-skinned persons. Of the clinical variants of malignant melanoma, nodular melanoma has the worst prognosis. Malignant melanomas have a better prognosis when characterized by a long period of radial (superficial) growth, as opposed to early vertical growth (as in nodular melanoma).

A 55-year-old man presents with a large, black-colored, asymmetric skin lesion with ill-defined borders on his back. He reports a family history of malignant melanoma. Which of the following clinical variants of malignant melanoma has the poorest prognosis? (A) Lentigo maligna melanoma (B) Superficial spreading melanoma (C) Nodular melanoma (D) Acral-lentiginous melanoma

The answer is D. The diagnosis is hairy cell leukemia, as evidenced by the presentation with splenomegaly, typical cellular morphology, and a positive stain for TRAP. Hairy cell leukemia is a B-cell disease, and the neoplastic cells are positive for the B cell markers CD19, CD20, and CD22. The most common presentation is in middle-aged men who present with anemia, leukopenia, and thrombocytopenia. The most common physical finding is massive splenomegaly. Hairy cell leukemia is of special interest because of the striking therapeutic efficacy of agents such as α-interferon, 2-chlorodeoxyadenosine, and deoxycoformycin.

A 55-year-old man presents with abdominal discomfort and fullness. Physical examination is remarkable for a massively enlarged spleen. Attempts at bone marrow aspiration are unsuccessful. A bone marrow core biopsy reveals numerous cells that have a single round nucleus surrounded by a cytoplasm with fine fibrillary projections. A stain for TRAP confirms the likely diagnosis. Which of the following statements about this disorder is correct? (A) The cell surface marker CD3 is almost always demonstrable. (B) The neoplastic cells stain positive for nonspecific esterase, a marker of monocytic maturation. (C) The typical patient with this disorder presents with a markedly elevated total leukocyte count. (D) This is an example of a well-known B-cell disorder. (E) There is currently no effective therapy for this condition.

The answer is B. The majority of urinary tract tumors are transitional cell carcinomas. Following resection, there is a marked tendency to recur. These tumors tend to spread contiguously rather than to distal locations. An early sign is hematuria.

A 55-year-old man presents with painless hematuria. On cystoscopy, a papillary mass is found in the bladder. Which of the following is a characteristic of this lesion? (A) Hematuria as a late manifestation (B) Marked tendency to recur after resection (C) Much more likely to be benign than malignant (D) Occurrence only in the bladder and nowhere else in the urinary tract (E) Usual presence of distant metastases at the time of diagnosis

The answer is C. The most frequent cause of transfusion-related hepatitis is hepatitis C virus infection. Hepatitis C virus is the most frequent cause of what was formerly termed non-A, non-B hepatitis.

A 55-year-old woman has cirrhosis. Twenty years ago she received a blood transfusion for profuse bleeding associated with a complication of childbirth. Shortly thereafter, she had an acute disease diagnosed as non-A, non-B hepatitis. Throughout her lifetime, her alcohol consumption has been minimal. Which of the following viruses is most likely responsible for her past and current liver disease? (A) Hepatitis A (B) Hepatitis B (C) Hepatitis C (D) Hepatitis D (E) Hepatitis E

The answer is C. Subarachnoid hemorrhage is most often caused by rupture of a berry aneurysm of the circle of Willis. Frank blood may be found in the CSF, and diffuse hemorrhage over the surface of the brain is due to subarachnoid bleeding.

A 55-year-old woman presents with "the worst headache of my life" and severe nausea. She states that the headache and nausea started suddenly several hours ago. A CSF tap reveals frank blood in the CSF. A CT scan of the head reveals diffuse hemorrhage over the surface of the brain. Which of the following is the most likely mechanism causing these findings? (A) Laceration of the middle meningeal artery (B) Laceration of the bridging veins (C) Rupture of a berry aneurysm in the circle of Willis (D) Rupture of a Charcot-Bouchard aneurysm (E) Hypertensive intraparenchymal hemorrhage

The answer is D. This is a classic case of stable angina, which is chest pain that is precipitated by exertion but relieved by rest. Stable angina is due to atherosclerosis of the coronary arteries. This patient has risk factors for ischemic heart disease (IHD) (e.g., cigarette smoking, hypertension, hyperlipidemia, diabetes, family history of IHD/coronary artery disease). Prinzmetal angina is intermittent chest pain at rest, and unstable angina is prolonged chest pain at rest.

A 55-year-old woman presents with complaints of chest pain. She states that the chest pain predictably occurs when she climbs four flights of stairs to reach her apartment or when she has been jogging for more than 10 minutes. She is particularly concerned because her mother died of a myocardial infarction at 50 years of age. Which of the following best describes this patient's state? (A) Arrhythmia (B) Myocardial infarction (C) Prinzmetal angina (D) Stable angina pectoris (E) Unstable angina pectoris

The answer is B. Diabetic nephropathy is marked by diffuse or nodular mesangial accumulations of glycosylated basement membrane-like material.

A 60-year-old man presents with proteinuria, hypertension, edema, and hypoalbuminemia. Histologic findings in the glomeruli of his kidneys include mesangial accumulation of basement membrane-like material. Which of the following is the most likely diagnosis? (A) Renal amyloidosis (B) Diabetic nephropathy (C) Membranous glomerulonephritis (D) Minimal change disease (E) Poststreptococcal glomerulonephritis

The answer is A. Asterixis is a flapping tremor of the hands associated with hepatic encephalopathy. Failure of the liver to detoxify metabolites absorbed from the gastrointestinal tract leads to accumulation of nitrogenous wastes that are neurotoxic. Caput medusae results from dilation of the periumbilical venous collaterals as a result of portal hypertension and activation of portal-caval anastomoses. Palmar erythema, capillary tel-angiectasias, and gynecomastia result from the hyperestrinism seen in liver disease with failure of the liver to metabolize estrogen.

A 56-year-old alcoholic man presents to the emergency department with confusion and lethargy. On physical examination, he is visibly jaundiced with ascites. Laboratory studies reveal increased prothrombin time and prolonged activated partial thromboplastin time, as well as significantly increased serum ammonia levels. Given a significantly increased serum ammonia, which of the following physical findings might you expect to see? (A) Asterixis (B) Capillary telangiectasias (C) Caput medusae (D) Gynecomastia (E) Palmar erythema

The answer is B. C. albicansis a major cause of vulvovaginitis. Many women are carriers of the yeast, and it therefore constitutes normal vaginal flora, although there are many conditions that alter the vaginal microenvironment, including pregnancy, oral contraceptives, and systemic antibiotics, making overgrowth possible. C. granulomatiscauses granuloma inguinale. Chancroid results in painful genital ulcers and is caused by the organism H. ducreyi. HSV type 2 can cause genital vesicular lesions. N. meningitidis can cause meningitis.

A 56-year-old diabetic woman has recently been treated with a 2-week course of antibiotics for a skin infection. She returns to the clinic for follow-up with a new complaint of a "cottage cheese-like" vaginal discharge with significant vaginal itching. The most likely cause of these symptoms is (A) C. granulomatis. (B) C. albicans. (C) chancroid. (D) herpes simplex. (E) Neisseria meningitidis.

The answer is C. In the first several hours after MI, the most common cause of death is arrhythmia. Although evidence of acute coronary artery obstruction may be found, morphologic myocardial changes and serum myocardial marker protein elevations are most often delayed for several hours. A myocardial fibrotic scar is evidence of an old prior MI.

A 56-year-old man collapsed at work and died 20 minutes later in the emergency room while blood was being drawn. The patient's history revealed an episode of prolonged chest discomfort 3 months earlier. Which of the following is least likely? (A) Death from arrhythmia (B) Fibrotic scar in the ventricular septum (C) Loss of myocardial striations and beginning infiltration with neutrophils (D) Normal values for serum creatine kinase, troponin I, and myoglobin (E) Severe atherosclerotic narrowing of the anterior descending branch of the left coronary artery with overlying thrombus formation

The answer is B. Intracellular pH is decreased in severe hypoxic cell injury. This change is caused by mitochondrial damage, which in turn results in decreased oxidative phosphorylation and diminished ATP synthesis. Decreased ATP stimulates glycolysis with lactate formation, thus resulting in decreased intracellular pH. Decreased ATP also diminishes the activity of the membrane-associated Na1K1 pump, allowing an influx of sodium and water. The final steps leading to cell death in severe hypoxic injury are associated with massive influx of extracellular calcium.

A 56-year-old man has severe chest pain, and angiography demonstrates acute occlusion of the circumflex branch of the left coronary artery. Blood flow is successfully restored by percutaneous transluminal coronary angioplasty (PTCA) with stenting. During the period of cellular hypoxic injury, which of the following intracellular changes is likely to have occurred? (A) Decreased calcium (B) Decreased pH (C) Decreased sodium (D) Increased activity of NaKpump (E) Increased adenosine triphosphate (ATP)

The answer is B. Chest pain, ECG findings of QS waves with elevated ST segments, and elevated serum CK-MB and cTn-I are diagnostic of acute MI. MI is the prototype of coagulative necrosis.

A 56-year-old man is seen in the emergency room because of the acute onset of severe crushing precordial chest pain that began on the golf course 1 hour earlier and has persisted until the time of admission. The ECG reveals precordial QS waves and elevated ST segments. Although normal at admission, both the serum CK-MB and cTn-I are significantly elevated 12 hours later. These changes are related to which type of necrosis? (A) Caseous (B) Coagulative (C) Fat (D) Gangrenous (E) Liquefactive

The answer is B. Persistent chest pain unrelieved by nitroglycerin and the abnormal ECG findings are diagnostic of MI. It would be unusual to observe significant elevations of CK-MB and cTn-I as early as 1 hour following an MI. These markers rise in parallel and are weakly positive in about 6 hours and reach peak levels in about 10 to 15 hours following an MI. CK-MB returns to normal levels in 3 to 7 days, while cTn-I can remain elevated a week or longer.

A 56-year-old man with a history of stable angina was seen in the emergency room 1 hour following the onset of unrelenting substernal pain not relieved by nitroglycerin. An electrocardiogram (ECG) revealed deep Q waves across the precordium, ST segment elevations, and inverted T waves. Serum levels of creatinine kinase MB (CK-MB) and cardiac troponin I (cTn-I) were within the normal range. What is the best explanation for these findings? (A) Lactate dehydrogenase (LDH) should have been ordered rather than CK-MB and cTn-I. (B) The diagnosis is acute myocardial infarction (MI), and CK-MB and cTn-I were determined too early in the course of the disease. (C) The diagnosis is unstable angina rather than acute MI. (D) The findings are indicative of a dissecting aneurysm (dissecting hematoma) of the aorta. (E) CK-MB alone can be misleading, and more definitive information would have been expected from total CK determination.

The answer is B. Aspirin permanently acetylates the active site of cyclooxygenase (both COX-1 and COX-2), causing enzyme inhibition. This subsequently inhibits synthesis of the prothrombotic agent thromboxane A2. Thromboxane A2 causes activation and aggregation of platelets.

A 56-year-old physician who has had a recent episode of unstable angina is advised by his cardiologist to take one "baby aspirin" a day because of the antithrombotic effect of aspirin. What is the mechanism by which aspirin acts as an antithrombotic agent? (A) Acetylation and activation of both cyclooxygenase-1 (COX-1) and cyclooxygenase-2 (COX-2) (B) Acetylation and inhibition of both COX-1 and COX-2 (C) Selective inhibition of COX-1 (D) Selective inhibition of COX-2

The answer is C. Multiple myeloma often presents with diffuse demineralization of bone, even though punched-out lesions are more characteristic. Findings in this scenario that distinguish multiple myeloma from other conditions also characterized by bony demineralization include anemia, hypergammaglobulinemia, proteinuria, and normal (rather than increased) serum alkaline phosphatase.

A 56-year-old woman presents with bone pain, diffuse demineralization of bone, hypercalcemia, anemia, hypergammaglobulinemia, proteinuria, and normal serum alkaline phosphatase. This set of findings is most suggestive of (A) Ewing sarcoma. (B) hyperparathyroidism. (C) multiple myeloma. (D) osteomalacia. (E) Paget disease of bone.

The answer is D. Myxoma of the heart, although rare, is the most common primary cardiac tumor. Because of the jelly-like appearance and myxoid histology similar to that of some organized thrombi, the neoplastic nature of this lesion was debated for many years; however, it is now generally believed that myxoma is a true neoplasm. The most common location of myxoma is in the left atrium. Due to its location, complications may develop due to physical obstruction of blood flow through the mitral valve, resulting in symptoms of congestive heart failure. Note that while angiosarcoma is the most common primary cardiac malignancy, it is not the most common primary cardiac tumor.

A 56-year-old woman presents with dyspnea on exertion, orthopnea, paroxysmal nocturnal dyspnea, and pulmonary edema. She also presents with severe dizziness and syncope, fatigue, weight loss, and arthralgias. After undergoing several tests, she is diagnosed with a primary heart tumor that is causing a "ball-valve obstruction" of her mitral valve. Which of the following is the most likely tumor? (A) Fibroma (B) Leiomyoma (C) Lipoma (D) Myxoma (E) Rhabdomyoma

The answer is D. Infiltration of tumor cells from cancers, such as those of breast and prostate, displaces bone marrow elements, thereby causing myelophthisic anemia. Chloramphenicol causes aplastic anemia in some individuals. Diphyllobothrium latuminfestation can result in megaloblastic anemia due to vitamin B12 deficiency. Sickle cell anemia is more common in black and Mediterranean populations.

A 56-year-old woman with a history of breast cancer that was treated 5 years ago with lumpectomy and radiation but with no chemotherapy returns with bone pain, fatigue, and weakness. A complete blood count reveals severe anemia, as well as decreased white blood cells and platelets. Examination of a peripheral blood smear reveals small numbers of nucleated red cells, as well as an occasional "blast" cell and myelocyte. A likely cause of the hematologic abnormalities is (A) chloramphenicol. (B) Diphyllobothrium latuminfestation. (C) megaloblastic anemia. (D) myelophthisic anemia. (E) sickle cell anemia.

The answer is D. The presentation is that of inflammatory carcinoma of the breast, which typically has an extremely poor prognosis. In this variant, cancer cells have invaded the skin and suspensory ligaments of the breast, causing dimpling and distortion of the normal breast architecture.

A 57-year-old woman who has not seen a physician in more than 20 years now presents with left breast pain. On examination, the left breast is markedly erythematous, swollen, and warm to touch. There is also significant dimpling of the breast (peau d'orange). In addition, the left nipple is completely retracted. Which of the following is the likely diagnosis? (A) Mastitis (B) Invasive ductal carcinoma (C) Medullary carcinoma (D) Inflammatory carcinoma (E) Mucinous carcinoma

The answer is C. Exposure to asbestos (common in construction workers, shipyard workers, or people who have worked with insulation or fire safety materials) markedly predisposes one to mesothelioma of the pleura or peritoneum and is also closely linked to primary lung carcinoma (especially in smokers).

A 60-year-old man presents with the new onset of dyspnea, chest pain, cough, and weight loss. In the past, he had worked in construction, installing insulation in buildings. Chest radiograph shows a rightsided pleural effusion with marked pleural thickening and mass formation. A malignant neoplasm is demonstrated by biopsy. What is the most likely diagnosis? (A) Small cell carcinoma (B) Squamous cell carcinoma (C) Mesothelioma (D) Adenocarcinoma (E) Carcinoid tumor

The answer is A. Serum alkaline phosphatase is an indicator of osteoblastic lesions in this advanced and unfortunate patient presentation. Both PSA and serum prostatic acid phosphatase are increased in prostatic cancer. Although prostatic cancers are usually quite androgen-responsive, androgens are not monitored for diagnosis or treatment of prostate cancer. Carcinoembryonic antigen-125 is a tumor marker for ovarian cancer. Serum α-fetoprotein and serum hCG are elevated in various germ cell neoplasms of the testis.

A 58-year-old African-American man presents to the emergency department with severe back pain. His history is negative for trauma and he has no other complaints. He denies urinary frequency, hesitancy, or dysuria. A digital rectal examination confirms the presence of a firm, hard, asymmetrical, and stony prostate. Imaging of the spine suggests osteoblastic involvement of the spine at lumbar vertebrae L3 to L4. In addition to an increase in PSA, which serum marker might also be elevated? (A) Alkaline phosphatase (B) Androgens (C) Carcinoembryonic antigen-125 (D) α-fetoprotein (E) Human chorionic gonadotropin

The answer is E. The scenario is typical of neuroblastoma, the most frequently occurring tumor in infants less than 1 year of age. The tumor may occasionally undergo spontaneous differentiation to a benign ganglioneuroma. Marked amplification of N-mycis characteristic, and greater amplification is a negative prognostic indicator. Most neuroblastomas are peripheral, and the most frequent site of origin is the adrenal medulla or adjacent tissues. CNS neuroblastomas are less common, and most often involve the cerebral hemispheres. Origin in the posterior cranial fossa is rare. Catecholamine production is characteristic.

A 6-month-old boy with hypertension is found to have a very large tumor arising in the left adrenal gland. Microscopically, the tumor consists of sheets of "small round blue cells" with minimal cytoplasm. Some of the cells contain pink cytoplasm and nucleoli, suggesting differentiation toward ganglion cells, and neurosecretory granules are visualized by electron microscopy. Urinary VMA is markedly increased. Which of the following is a major characteristic of this neoplasm? (A) Characteristic gene deletions on chromosome 5 (B) Highly differentiated form of malignant ganglioneuroma (C) Posterior cranial fossa origin with metastasis to both adrenals (D) Serotonin production (E) Spontaneous differentiation in some instances

The answer is B. Barrett esophagus is columnar metaplasia of the esophageal squamous epithelium (squamous-to-columnar). The columnar epithelium is often of the intestinal type with goblet cells. Barrett esophagus is a complication of long-standing GERD and is a precursor of esophageal adenocarcinoma. The most common location is in the distal (lower) third of the esophagus.

A 60-year-old Caucasian man with a 5-year history of gastroesophageal reflux disease (GERD) presents with persistent pyrosis (heartburn) and acid regurgitation. He has had similar symptoms for the past 5 years. Because this patient has a long history of GERD, an esophagogastroduodenoscopy is performed to screen for Barrett esophagus, a well-known complication of long-standing GERD. Results reveal that Barrett esophagus is indeed present. Which of the following is true of Barrett esophagus? (A) A biopsy will show a histologic finding of columnar-to-squamous metaplasia. (B) It is a known precursor of adenocarcinoma of the esophagus. (C) It is a known precursor of carcinoma of the stomach. (D) It is a known precursor of squamous cell carcinoma of the esophagus. (E) The most common location is the proximal (upper) third of the esophagus.

The answer is A. The liver is the site of production of all coagulation factors except vWF, and severe hepatic dysfunction can thus be associated with multiple factor deficiencies, excluding vWF.

A 60-year-old chronic alcoholic with known alcoholic cirrhosis presents with upper gastrointestinal hemorrhage. Despite prolonged tamponade, bleeding is persistent. A coagulation defect related to the liver disease is suspected. Which of the following abnormalities is most consistent with this possibility? (A) Deficiency of all clotting factors except for vWF (B) Deficiency of factors II, VII, IX, and X (C) Deficiency of factors II, V, VII, and X (D) Deficiency of factors IX, X, XI, and XII (E) Deficiency of vWF

The answer is B. The diagnosis is CLL. Hypogammaglobulinemia may occur early in the course of the disease, leading to frequent bacterial infections. The mean survival is 3 to 7 years after diagnosis, although much longer symptom-free survivals are quite common. CLL is characterized by a proliferation of neoplastic mature lymphoid cells, which are almost always B cells.

A 60-year-old man is referred because of splenomegaly and generalized lymphadenopathy. The total white blood cell count is markedly elevated, and the differential count reveals a preponderance of matureappearing lymphocytes. Bone marrow examination reveals a diffuse infiltration with similar-appearing lymphocytes. Which of the following statements best characterizes this disorder? (A) A progressive increase in the number of myeloblasts and promyelocytes is indicative of acceleration of the disease process. (B) Bacterial infections are common early in the disease due to hypogammaglobulinemia. (C) Mean survival is less than 1 year after diagnosis. (D) Myelofibrosis is a common complication. (E) The neoplastic lymphoid cells are most often T cells, not B cells.

The answer is A. The diagnosis is polycythemia vera (primary polycythemia), one of the myeloproliferative syndromes. The disorder is characterized by prominent erythrocytosis, moderate granulocytosis, and thrombocytosis. Because of hyperviscosity and sludging of blood, there is a frequent association with thrombosis or hemorrhagic phenomena. Marked splenomegaly and decreased erythropoietin are other classic characteristics. Cushing syndrome and hypoxic states are associated with secondary polycythemia, not polycythemia vera. About 3% of patients terminate in acute leukemia, not CML.

A 60-year-old man is referred for evaluation of marked erythrocytosis and splenomegaly. Laboratory studies confirm an elevated red blood cell count and additionally demonstrate a moderate increase in circulating granulocytes and platelets. Oxygen saturation studies are normal, and isotopic studies reveal an increase in total red cell mass. Which of the following is characteristic of this disorder? (A) Frequent association with thrombosis or hemorrhagic phenomena (B) Increased erythropoietin concentration (C) Manifestation of Cushing syndrome (D) Most often secondary to hypoxia (E) Usual termination in CML

The answer is D. Nonbacterial thrombotic endocarditis, or marantic endocarditis, has been associated with a variety of wasting diseases and is observed most often in patients with cancer.

A 70-year-old woman has a long history of metastatic colon cancer, and she donates her body for use in medical school anatomy courses. At death, the body is emaciated and cachectic, and gross dissection reveals small fibrin deposits arranged around the line of closure of the leaflets of the mitral valve. The valvular lesions most likely represent (A) bacterial endocarditis. (B) endocarditis of the carcinoid syndrome. (C) Libman-Sacks endocarditis. (D) nonbacterial thrombotic (marantic) endocarditis. (E) rheumatic endocarditis.

The answer is A. This is a case of syphilitic (luetic) aortitis. In syphilitic aortitis, the elastica of the aorta undergoes calcification and is replaced by fibrous tissue, resulting in dilation of the ascending aorta and separation of the aortic valve commissures, with resultant aortic insufficiency. Thus, echocardiography and computed tomography of the heart reveal calcification in a linear pattern along the ascending aorta, calcification in the coronary arteries (leading to anginal symptoms), and aortic valvular insufficiency.

A 60-year-old man presents with angina. He has no past medical history of heart disease. On questioning, the patient reveals that he had repeated sexually transmitted diseases in the past, including a painless chancre (a hard, round sore) on his penis, for which he never sought medical attention. Rapid plasma reagin (RPR), Venereal Disease Research Laboratory (VDRL) slide test, and fluorescent treponemal antibody (FTA) serologic tests (indicative of syphilis infection) are positive. Echocardiography and computed tomography of the heart are performed. The history of untreated syphilis suggests that these tests will most likely detect which of the following abnormalities? (A) Aortic valvular insufficiency and linear calcification along the ascending aorta (B) Bicuspid aortic valve with aortic stenosis (C) Large valvular vegetations from bacterial endocarditis (D) Right-sided heart failure from the carcinoid syndrome (E) Small fibrin deposits on the mitral valve from nonbacterial thrombotic (marantic) endocarditis

The answer is D. Subdural hematoma can be acute (e.g., due to moderate head trauma) or chronic (e.g., due to whiplash). Subdural hematomas are caused by venous bleeding, most often from laceration of the bridging veins, which join the cerebral vessels to the venous sinuses within the dura. The classic CT scan finding is formation of a "crescent" along the inner skull, which is due to subdural venous bleeding.

A 60-year-old man presents with chronic headache and personality changes since he had been in a car accident a month ago, during which he had suffered a whiplash neck injury. A CT scan of the head demonstrates a hyperdense crescentic region along the inner skull. Which of the following is the most likely diagnosis? (A) Bruising of the brain substance of the cerebral hemisphere (B) Enlargement of the cerebral ventricles (C) Epidural hematoma (D) Subdural hematoma (E) Subarachnoid hemorrhage

The answer is E. Asbestosis is caused by inhalation of asbestos fibers, characterized by yellow-brown, rod-shaped ferruginous bodies with clubbed ends that stain positively with Prussian blue. Asbestosis results in a marked predisposition to malignant mesothelioma of the pleura or peritoneum. Exposure to asbestos is also a risk factor for primary lung carcinoma, as well as for carcinoma of the oropharynx, esophagus, and colon. The risk of primary lung carcinoma is greatly increased in cigarette smokers with exposure to asbestos.

A 60-year-old man presents with dyspnea on exertion and a nonproductive cough. He has never smoked, but he worked as a shipbuilder, with known asbestos exposure approximately 20 years ago. To which of the following conditions is this patient especially predisposed? (A) Acute respiratory distress syndrome (B) Goodpasture syndrome (C) Idiopathic pulmonary fibrosis (D) Idiopathic pulmonary hemosiderosis (E) Malignant mesothelioma of the pleura

The answer is D. The most common cause of lobar pneumonia is S. pneumoniae. The organism is also known as the pneumococcus, and the disease entity is often referred to as pneumococcal pneumonia.

A 60-year-old man presents with fever and chills, productive cough with rusty sputum, pleuritic pain, and shortness of breath for the past several days. A complete blood count reveals neutrophilia and an increase in band neutrophils. A chest radiograph reveals consolidation involving the entire left lower lobe. Which of the following microorganisms is the most likely etiologic agent? (A) Hemophilus influenzae (B) Klebsiella pneumoniae (C) Staphylococcus aureus (D) Streptococcus pneumoniae (E) Streptococcus pyogenes

The answer is C. Of course, the immediate problem in this patient is life-threatening upper gastrointestinal hemorrhage, an important complication of peptic ulcer disease. Peptic ulcer disease occurs most frequently in the first portion of the duodenum, the lesser curvature of the stomach, or the distal esophagus. Duodenal peptic ulcers are associated with hypersecretion of gastric acid and pepsin and are closely related to gastric H. pyloriinfection. Apparently, H. pyloriincreases gastric acid secretion and impairs mucosal defenses. Other predisposing factors include aspirin or NSAID intake, smoking, ZollingerEllison syndrome, primary hyperparathyroidism, and multiple endocrine neoplasia type I.

A 60-year-old man presents with hematemesis, melena, guaiac-positive stools, and signs of circulatory collapse. He has a 20-year history of burning midepigastric pain and tenderness relieved by food, milk, or antacids. Also, he has been taking high doses of NSAIDs to relieve the pain of long-standing arthritis. Esophagogastroduodenoscopy reveals a peptic ulcer in the upper duodenum. Which of the following is an important association of duodenal peptic ulcer disease? (A) Barrett esophagus and columnar intestinal metaplasia of esophageal squamous epithelium (B) Evolution into carcinoma as a likely sequela (C) H. pyloriinfection (D) Hiatal hernia and incompetent lower esophageal sphincter (E) Pernicious anemia and achlorhydria

The answer is A. Transitional cell carcinoma is the most common tumor of the urinary collecting system and can occur in renal calyces, pelvis, ureter, or bladder.

A 60-year-old man presents with painless hematuria. Transurethral biopsy of the bladder is performed, and histologic evaluation demonstrates the presence of carcinoma of the urinary bladder. Which of the following is the most common type of carcinoma of the urinary bladder? (A) Transitional cell carcinoma (B) Squamous cell carcinoma (C) Adenocarcinoma (D) Clear cell carcinoma (E) Small cell carcinoma

The answer is E. The history is suggestive of Paget disease of bone, which is also marked by bone pain, anterolateral bowing of long bones, and sometimes high-output heart failure. This disorder is complicated by osteosarcoma in about 1% of cases. Paget disease of bone should not be confused with Paget disease of the breast, which is closely associated with an underlying ductal carcinoma.

A 60-year-old man presents with progressive bone deformity and pain, progressive hearing loss, and increasing skull size. Workup revealed generalized increased bone density with cortical thickening, normal serum calcium and phosphorus, and markedly elevated serum alkaline phosphatase. Biopsy revealed a characteristic "mosaic" pattern. Which one of the following tumors is a known complication of the disorder suggested by this scenario? (A) Ewing sarcoma (B) Giant cell tumor (C) Metastatic duct carcinoma of the breast (D) Multiple enchondromas (E) Osteosarcoma

The answer is A. The clinical and microscopic picture is that of GIST. These tumors most commonly arise in the submucosa of the stomach but can arise anywhere along the GI tract and occasionally occur outside the GI tract. Microscopically they are comprised of spindled cells which stain for CD117. On the molecular level, GISTs show mutations in the oncogene c-kit. A minority of tumors also show PDGFR-αmutations. Although most cases are indolent and cured by resection, a subset shows aggressive behavior. Size, mitotic count, and location are the most important predictors of prognosis, with small size, low mitotic activity, and gastric location suggesting relatively benign behavior.

A 60-year-old man undergoes upper endoscopy for dyspepsia. The endoscopist notes a well-circumscribed submucosal tumor. The tumor is resected and demonstrates a spindled cell population on microscopy. Immunostaining for CD117 is positive. The diagnosis is (A) Gastrointestinal stromal tumor (GIST). (B) Melanoma. (C) Leiomyoma. (D) Adenocarcinoma. (E) Lymphoma.

The answer is C. The illustration demonstrates thrombotic obliteration of glomerular capillary loops and is typical of DIC. In DIC—sometimes termed consumption coagulopathy—coagulation factors, fibrinogen, and platelets are depleted by the widespread thrombotic process. Thus the coagulation assays (APPT, PT, and thrombin time) are prolonged and the platelet count is decreased. Increased fibrin and fibrinogen degradation products are sensitive indicators of DIC.

A 60-year-old man with pancreatic cancer dies following a terminal episode that began 3 days earlier, characterized by generalized bleeding with oozing from intravenous infusion sites, widespread petechial and ecchymotic cutaneous bleeding, and intractable epistaxis. At autopsy, a glomerular change similar to that shown in the illustration was demonstrated (the lesion is stained with a special stain that stains fibrin purple). Prior to death, which of the following laboratory measures of blood coagulation would have shown a decrease in the reported value? (A) Activated partial thromboplastin time (B) Fibrin and fibrinogen degradation products (C) Platelet count (D) Prothrombin time (E) Thrombin time

The answer is C. Emphysema is an example of COPD. Due to the destruction of alveolar walls, a lack of elastic recoil causes air to become trapped in alveoli, and, thus, airflow obstruction occurs on expiration. In COPD, FEV1 is decreased, whereas FVC is normal or increased; therefore, patients with COPD have a decreased FEV1:FVC ratio.

A 60-year-old man, a heavy smoker, presents for advice to stop smoking. On physical examination, he is thin and has a ruddy complexion. He has a productive cough and a barrel-shaped chest. He sits leaning forward with his lips pursed to facilitate his breathing. A diagnosis of emphysema is made. Which of the following is the most likely histologic finding in the lungs? (A) Bronchial smooth muscle hypertrophy with proliferation of eosinophils (B) Diffuse alveolar damage with leakage of protein-rich fluid into alveolar spaces (C) Dilation of air spaces with destruction of alveolar walls (D) Hyperplasia of bronchial mucus-secreting submucosal glands (E) Permanent bronchial dilation caused by chronic infection, with bronchi filled with mucus and neutrophils

The answer is B. GBM is the most common primary intracranial neoplasm. This brain tumor has a very poor prognosis, with death occurring in less than 1 year.

A 60-year-old man, a suicide victim, comes to autopsy. Before dying, he had been despondent after being informed that he had an extremely aggressive brain tumor. There had been a recent onset of headache, seizures, and mental status changes, and MRI had demonstrated an infiltrating neoplasm invading the cerebral hemispheres and crossing the midline, with areas of necrosis and abnormal blood vessels. The autopsy confirms the MRI findings and also demonstrates hemorrhage and a pseudopalisade arrangement of tumor cells. The tumor is most likely a(n) (A) ependymoma. (B) glioblastoma multiforme. (C) meningioma. (D) neurilemmoma (schwannoma). (E) oligodendroglioma.

The answer is D. Sessile serrated adenomas can be very subtle and are easily missed on endoscopy. Under the microscope, they closely resemble hyperplastic polyps but show more complicated glands, often with "boot-shaped" configurations at the gland bases. Unlike tubular adenomas, they lack the overtly dysplastic epithelium and often do not form polypoid lesions. Hyperplastic polyps, Peutz-Jeghers polyps, and inflammatory polyps are not malignant precursors.

A 60-year-old woman develops a rightsided colonic adenocarcinoma. She has a history of alleged colonoscopies; however, review of images from her most recent colonoscopy reveals a prominent fold in the region that subsequently developed cancer. Biopsy from this area would have most likely revealed which of the following? (A) Tubular adenoma (B) Hyperplastic polyp (C) Peutz-Jeghers polyp (D) Sessile serrated adenoma (E) Inflammatory polyp

The answer is B. The illustration demonstrates a renal cell carcinoma, which most often arises in one of the renal poles, frequently the upper pole. Hematuria is the most frequent presenting sign. The tumor cells often have a clear cell appearance, which led to an earlier erroneous concept that this tumor was of adrenal origin and to the older name "hypernephroma." The tumor is often quite large and may result in a palpable mass. Malignant cells are only rarely detected in the urine in renal cell carcinoma.

A 60-year-old woman dies of a tumor that had invaded the renal vein and entered the inferior vena cava. At autopsy, the kidney has the appearance shown in the figure. Which of the following is a characteristic or association of this neoplasm? (A) Adrenal origin (B) Hematuria (C) Hypocalcemia (D) Tumor cells in the urine (E) Typical symmetrical bilateral involvement

The answer is C. Medullary carcinoma is characterized histologically by sheets of tumor cells in an amyloid-containing stroma. This neoplasm is a calcitonin-producing tumor derived from "C" cells of the thyroid. Medullary carcinoma can occur singly or as a component of multiple endocrine neoplasia (MEN) syndromes types IIa and IIb.

A biopsy of a "cold" nodule from the left lobe of the thyroid from an otherwise asymptomatic 30-year-old woman is found to demonstrate a malignant proliferation of C cells and an amyloid stroma that stains positively with a Congo red stain. This tumor is referred to as which of the following types of carcinoma? (A) Epidermoid (B) Follicular (C) Medullary (D) Papillary (E) Undifferentiated

The answer is C. The findings are those of follicular lymphoma, the most frequently occurring form of non-Hodgkin lymphoma. This particular neoplasm is marked by the presence of the 14;18 translocation with increased expression of bcl-2, an inhibitor of apoptosis.

A 60-year-old woman presents with a painless cervical lymph node mass that has been progressively enlarging over the past month. Splenomegaly is noted on abdominal examination. A cervical lymph node biopsy reveals effacement of the architecture by angulated grooved cells in a nodular pattern. Which of the following statements about this disorder is correct? (A) The findings are those of a benign neoplasm of lymphoid cells. (B) The findings are those of the least frequently occurring form of non-Hodgkin lymphoma. (C) The most likely common cytogenetic and molecular change is t(14;18) with increased expression of the oncogene bcl-2. (D) Special stains are required for the diagnosis because the description is that of an anaplastic carcinoma. (E) This diagnosis cannot be confirmed in the absence of Reed-Sternberg cells.

The answer is B. This is a case of osteoarthritis (degenerative joint disease), the most common form of arthritis. Osteoarthritis is a chronic noninflammatory joint disease most often related to mechanical trauma and long-term use of affected joints ("wear-and-tear" arthritis). Osteoarthritis is characterized by eburnation (polished, ivory-like appearance of bone, due to erosion of overlying cartilage), cystic changes in subchondral bone, and new bone formation. Osteophytes (bony spurs) can form at the distal interphalangeal joints (Heberden nodes) or at the proximal interphalangeal joints (Bouchard nodes).

A 60-year-old woman presents with deep, achy joint pain in her fingers. She states that the pain gets worse with extensive use of her fingers. Physical examination reveals reduced range of motion and crepitus, with pain noted asymmetrically in the distal interphalangeal (DIP) joints, proximal interphalangeal joints, and metacarpophalangeal joints. Heberden nodes are noted on the DIP joints of one hand. Which of the following is the most likely etiology of this condition? (A) Autoimmune disease (B) Mechanical injury ("wear and tear") (C) N. gonorrhoeaeinfection (D) Secondary manifestation of chronic lung disease or cyanotic cardiac disease (E) Urate crystal deposition

The answer is D. Renal papillary necrosis is a well-known complication of chronic analgesic nephritis, which is caused by long-term abuse of phenacetin or its metabolite, acetaminophen, most often in combination with aspirin or a NSAID. Another major cause of renal papillary necrosis is diabetes mellitus. Phenacetin abuse is also associated with a markedly increased incidence of transitional cell carcinoma of the renal pelvis.

A 60-year-old woman presents with fever, chills, dysuria, hematuria, and pain. She reports a long-standing history of use of phenacetin, acetaminophen, nonsteroidal anti-inflammatory drugs (NSAIDs), and aspirin. Abuse of these medications is associated with necrosis of which of the following sites? (A) Basal ganglia (B) Hilar lymph nodes (C) Myocardium (D) Renal papillae (E) Splenic arterioles

The answer is E. Felty syndrome is the combination of splenomegaly, neutropenia, and rheumatoid arthritis.

A 60-year-old woman with a 10-year history of severe rheumatoid arthritis presents with splenomegaly and neutropenia. Which of the following is the most likely diagnosis? (A) Ankylosing spondylitis (B) Hypertrophic osteoarthropathy (C) Osteoarthritis (D) Rheumatoid arthritis (E) Felty syndrome

The answer is D. Chronic bronchitis is an example of COPD. The pathologic hallmark of chronic bronchitis is marked hyperplasia of bronchial submucosal glands and bronchial smooth muscle hypertrophy, which can be quantified by the Reid index, a ratio of glandular layer thickness to bronchial wall thickness.

A 60-year-old woman with a heavy smoking history presents with chronic productive cough that has been present for three consecutive months over the past two consecutive years. On physical examination, her skin has a bluish tinge, and she is overweight. The patient is diagnosed with chronic bronchitis. Which of the following is the most likely histologic finding in this patient's lungs? (A) Bronchial smooth muscle hypertrophy with proliferation of eosinophils (B) Diffuse alveolar damage with leakage of protein-rich fluid into alveolar spaces (C) Dilation of air spaces with destruction of alveolar walls (D) Hyperplasia of bronchial mucussecreting submucosal glands (E) Permanent bronchial dilation caused by chronic infection, with bronchi filled with mucus and neutrophils

The answer is B. The middle cerebral arteries are the most common sites of cerebral embolic occlusion in general, and, in this instance, the involvement is almost certainly left-sided. Middle cerebral artery lesions are manifest by contralateral upper extremity weakness and ipsilateral facial weakness. Also, in right-handed individuals, the left side of the cerebral cortex is usually dominant, and verbal aphasias are caused by left-sided lesions in the great majority of cases. Anterior cerebral artery occlusions cause contralateral lower extremity weakness and altered mental status. Posterior artery occlusions cause contralateral homonymous hemianopsia (e.g., right-sided posterior artery occlusion causes left visual field loss).

A 60-year-old woman with chronic atrial fibrillation is seen in the emergency department because of acute onset of marked weakness of her right arm, drooping of the left side of her face, and verbal aphasia. T2-weighted magnetic resonance imaging reveals a cerebral infarct. Assuming that the lesion has been caused by embolization from the left atrium, which vessel is the most likely site of embolic arrest? (A) Right middle cerebral artery (B) Left middle cerebral artery (C) Right anterior cerebral artery (D) Left anterior cerebral artery (E) Right posterior cerebral artery

The answer is E. Granulosa cell tumors are sex cord-stromal tumors that typically secrete estrogen. For this reason, endometrial hyperplasia or endometrial carcinoma may be a concomitant finding in women diagnosed with a granulosa cell tumor.

A 60-year-old woman with uterine bleeding is found to have endometrial hyperplasia and an ovarian tumor. Which is the most likely ovarian tumor? (A) Serous cystadenocarcinoma (B) Krukenberg tumor (C) Dysgerminoma (D) Teratoma (E) Granulosa cell tumor

The answer is D. Rupture of the left ventricle, a catastrophic complication of acute myocardial infarction, usually occurs when the necrotic area has the least tensile strength, about 4 to 7 days after an infarction, when repair is just beginning. The anterior wall of the heart is the most frequent site of rupture, usually leading to fatal cardiac tamponade. Internal rupture of the interventricular septum or of a papillary muscle may also occur. The risk of arrhythmia is greatest within the first 6 hours after myocardial infarct. Arrhythmias are the most important early complication of acute myocardial infarction, accounting for almost 50% of deaths shortly after myocardial infarction. Myocardial, or pump, failure and mural thrombosis are other complications that may develop as a result of permanent damage to the heart after infarct. Ventricular aneurysms may develop in the fibrotic scar within 3-6 months after myocardial infarct.

A 60-year-old-man is discharged after being observed in the hospital for 4 days following a myocardial infarction. He returns to his normal activities, which include sedentary work only. This point in time following a myocardial infarct is noteworthy for the special danger of which of the following? (A) Arrhythmia (B) Mural thrombosis (C) Myocardial (pump) failure (D) Myocardial rupture (E) Ventricular aneurysm

The answer is A. The illustration demonstrates an adenocarcinoma diffusely infiltrating the wall of the colon, with elevated borders and ulceration of the mucosa.

A 62-year-old man is seen because of a change in bowel habits. A lesion similar to that illustrated below is resected from the sigmoid colon. The diagnosis is (A) adenocarcinoma. (B) Crohn disease. (C) non-Hodgkin lymphoma. (D) pseudomembranous colitis. (E) tubular adenoma.

The answer is B. Iron deficiency anemia is the most common cause of hypochromic microcytic anemia, and gastrointestinal bleeding is the most likely cause of iron deficiency in an adult male. Such a finding warrants a complete workup, including colonoscopy to detect the source of the bleeding. Dietary deficiency of iron and increased iron requirements are common causes of iron deficiency in women of child-bearing age, especially during pregnancy. Hemolytic anemia is usually accompanied by reticulocytosis. The reduced red cell parameters (hemoglobin, red blood cell count, hematocrit) that are observed with hemodilution are not truly representative of anemia, which is formally defined as a reduction in whole body red cell mass.

A 62-year-old man presents with pallor, fatigue, and dyspnea on exertion. A complete blood count reveals microcytic hypochromic anemia. The most likely cause of these findings is (A) dietary deficiency of iron. (B) gastrointestinal bleeding. (C) hemodilution. (D) hemolytic anemia. (E) increased iron requirement.

The answer is D. The Trousseau sign, or migratory thrombophlebitis, is associated with carcinoma of the pancreas. The finding of appearing and disappearing thrombosis can affect up to 10% of patients. Only about 20% of lesions are in the head of the pancreas, where they present relatively early with obstructive jaundice. Asterixis is a flapping tremor associated with hepatic encephalopathy. Gallstone ileus is a complication of cholelithiasis when the gallstone erodes through the gallbladder into the adjacent small bowel. The Murphy sign is associated with acute cholecystitis. The Whipple triad is associated with insulinomas, tumors of the endocrine (rather than exocrine) pancreas.

A 63-year-old chronic alcoholic presents with weight loss, anorexia, and abdominal pain radiating to the back. Physical examination indicates a palpably enlarged gallbladder, and laboratory studies demonstrate conjugated hyperbilirubinemia. Computed tomography demonstrates a mass in the head of the pancreas. Which of the following is associated with the diagnosis of pancreatic adenocarcinoma? (A) Asterixis (B) Gallstone ileus (C) Murphy sign (D) Trousseau sign (E) Whipple triad

The answer is B. Pleural and peritoneal mesotheliomas are associated with exposure to asbestos, and the apparent tumorigenic effect of asbestosis is markedly enhanced by cigarette smoking. Aflatoxin B1 is associated with hepatocellular carcinoma. Clear cell adenocarcinoma of the vagina has been a hazard to daughters exposed during intrauterine life to DES administered to their mothers to prevent spontaneous abortion. Ionizing radiation is associated with many cancers, including leukemias, breast cancer, and thyroid malignancies. β-naphthylamine and other aniline dyes are associated with transitional cell carcinoma of the bladder.

A 67-year-old man, a heavy smoker, is seen because of dyspnea and cough. A chest X-ray reveals abnormal densities, and a computed tomography (CT) scan is suggestive of a neoplasm involving the pleura. A biopsy confirms the diagnosis of mesothelioma. Other than cigarette smoke, this finding suggests exposure to which toxin? (A) Aflatoxin B1 (B) Asbestos (C) Diethylstilbestrol (DES) (D) Ionizing radiation (E) β-Naphthylamine

The answer is D. Chronic bronchitis, which is clearly linked to cigarette smoking, is defined as productive cough occurring during at least 3 consecutive months over at least 2 consecutive years.

A 67-year-old man, a two-pack-a-day smoker since age 18, has had a productive cough over the past 20 years. Although continuous through the years, there have been episodic exacerbations of these symptoms, which have worsened during the past 4 or 5 years, lasting 3 or 4 months at a time. Arterial pO2 is decreased and pCO2 is increased. Total lung capacity measurements are normal. These findings are most suggestive of which of the following pulmonary disorders? (A) Adult respiratory distress syndrome (ARDS) (B) Bronchial asthma (C) Lung carcinoma (D) Chronic bronchitis (E) Panacinar emphysema

The answer is D. Carcinoma of the urinary bladder, almost always transitional cell carcinoma, is associated with industrial exposure to aniline dyes, such as β-naphthylamine, usually many years in the past. This type of cancer most often spreads by local extension to surrounding tissues. Hematogenous dissemination is a late finding. Exposure to aflatoxin is associated with hepatocellular carcinoma, and Schistosoma haematobium infection is associated with squamous cell bladder carcinoma, not transitional cell. Methyldopa is an association of bile duct cancer.

A 68-year-old man presented with painless hematuria. Cystoscopic examination revealed a papillary tumor. Excisional biopsy was performed, and the diagnosis of transitional cell carcinoma was made. Which of the following is a well-known association of this type of cancer? (A) Early hematogenous spread (B) Exposure to aflatoxin (C) Long-term use of methyldopa, an antihypertensive agent (D) Industrial exposure to aniline dyes, such as β-naphthylamine (E) Schistosoma haematobiuminfection

The answer is E. Endometrial cancer is the most common gynecologic malignancy, and Type I cancers account for the majority (80%) of these cases. A major predisposing factor to Type I endometrial caricnoma is prolonged and unopposed exposure to estrogen. In the case of obesity, adipose tissue converts androgens into estrogens, fueling the proliferation of endometrial tissue. Similarly, diabetes, nulliparity (not multiparity), and hypertension are also predisposing factors. Early sexual activity with multiple partners is associated with cervical cancer. Salpingitis can predispose to ectopic pregnancy. Endometriosis is a benign condition and has no relation to endometrial cancer.

A 68-year-old postmenopausal woman presents for evaluation of the recent onset of vaginal bleeding, and a diagnosis of type I endometrial carcinoma is made on endometrial biopsy. Which of the following is a risk factor for this condition? (A) Endometriosis (B) Multiparity (C) Salpingitis (D) Early sexual activity with multiple partners (E) Obesity

The answer is D. Generalized lower abdominal pain, bloody stools, and signs of acute inflammation in an older patient are classic findings in diverticulitis. Appendicitis and Crohn disease occur more often in younger persons, and bloody stools would not be expected. Signs of acute inflammation would not be expected in carcinoma of the rectum or in tubular adenoma.

A 68-year-old woman has fever, generalized lower abdominal pain, and bright red blood in the stools. The white blood cell count is 15,000/mL, with 85% segmented neutrophils. The most likely diagnosis is (A) acute appendicitis. (B) carcinoma of the rectum. (C) Crohn disease. (D) diverticulitis. (E) tubular adenoma in sigmoid colon.

The answer is B. The clinical history is strongly suggestive of bacterial pneumonia. Pleural fluid from this patient would typically be an exudate rather than a transudate, and would be expected to be cloudy and contain many neutrophils. The fluid would also demonstrate reduced glucose, increased protein, and increased specific gravity.

A 68-year-old woman presents with fever, chills, and cough productive of blood-tinged sputum. Fluid aspirated from the right pleural space would most likely (A) be clear and straw-colored in appearance. (B) contain large numbers of neutrophils. (C) have a glucose content somewhat higher than the serum glucose. (D) have a protein content of less than 1 g/dL. (E) have a specific gravity of 1.012.

The answer is E. The illustration shows a chronic gastric peptic ulcer with characteristic radiating folds of the gastric mucosa starting at the ulcer margins. The lesion has a smooth base with a little fibrin attached and nonelevated, punched-out margins, in contrast to gastric carcinoma, which often has an irregular necrotic base and firm, raised margins. Despite these characteristic findings, the distinction between gastric peptic ulcer and ulcerated carcinoma must be established by biopsy. In contrast to carcinoma, peptic ulcer will usually heal with conservative management.

A 69-year-old man was seen for vague abdominal distress. The gastric lesion shown in the figure was resected following initial endoscopic discovery. Which of the following statements about this condition is correct? (A) It has been decreasing in frequency over the past several decades. (B) It is more frequent in Japan than in the United States. (C) It is related to the use of nitrites as food preservatives. (D) It may result in Krukenberg tumors. (E) It will most likely heal with conservative management.

The answer is D. The clinical description is that of Henoch-Schönlein purpura, a form of leukocytoclastic angiitis (hypersensitivity vasculitis) resulting from an immune reaction that damages the vascular endothelium. Henoch-Schönlein purpura is closely related to IgA nephropathy, a glomerulopathy resulting in nephritic syndrome, and may represent a systemic version of this disease.

A 7-year-old boy presents with palpable purpura on the buttocks and legs, fever, abdominal pain and vomiting, arthritis in his knees and ankles, melena, and hematuria. His mother states that he had an upper respiratory illness approximately 1 week ago, but has otherwise been well. Blood tests reveal mild renal insufficiency. The most likely cause of the bleeding into the skin observed in this patient is (A) coagulation factor deficiency. (B) qualitative platelet dysfunction. (C) quantitative platelet dysfunction. (D) vasculitis. (E) vitamin deficiency.

The answer is D. Merkel cell carcinoma is a neuroendocrine tumor of the skin which most commonly arises in the head and neck of the elderly. It is comprised of small round blue cells that express neuroendocrine markers and show perinuclear staining for cytokeratin 20. Although the morphology and synaptophysin staining could be seen in a small cell carcinoma of the lung, the cytokeratin 20 staining would not be expected.

A 70-year-old man has a 2-cm red plaque on his scalp. On microscopy the tumor is comprised of small, round, blue cells with scant cytoplasm and granular chromatin. Immunostaining with cytokeratin 20 shows perinuclear dot-like positivity and synaptophysin is also positive. The diagnosis is (A) Malignant melanoma. (B) Squamous cell carcinoma. (C) Metastatic small cell carcinoma of the lung. (D) Merkel cell carcinoma. (E) Basal cell carcinoma.

The answer is A. This is a case of squamous cell carcinoma of the skin. Squamous cell carcinoma is a common skin tumor associated with excessive sun exposure. Clusters of darkly staining basaloid cells with a palisading arrangement of nuclei are characteristic of basal cell carcinoma. Malignant melanocytes are found in malignant melanoma. A keloid is a tumor-like scar resulting from abnormal proliferation of connective tissue with deranged arrangement of collagen fibers.

A 70-year-old man presents with a scaling, indurated, ulcerated nodule on the back of his left hand. He states that the nodule has been growing larger over time. The patient has had much direct sun exposure in the past. Which of the following is the most likely histologic finding in this patient's skin lesion? (A) Invasion of the dermis by sheets and islands of neoplastic epidermal cells, often with "keratin pearls" (B) Clusters of darkly staining basaloid cells, with a palisading arrangement of the nuclei of the cells at the periphery of the clusters (C) Malignant melanocytes with numerous mitotic figures (D) Abnormal proliferation of the connective tissue, with deranged arrangement of collagen fibers

The answer is E. Adenocarcinoma of the colon most commonly develops through a progression of mutations in oncogenes and tumor suppressor genes in a multistep process. Normal mucosa evolves into a tubular adenoma with malignant potential, which then further evolves into carcinoma (the adenoma-carcinoma sequence). Carcinoma of the rectosigmoid (left-sided) tends to present as early obstruction, with change in bowel habits and decreased caliber of stool, whereas carcinoma of the right colon (right-sided) tends to present late, with iron deficiency anemia due to chronic blood loss from the lesion.

A 70-year-old man presents with fatigue, weight loss, abdominal pain, and overt blood in the stools. A complete blood count reveals anemia with hemoglobin of 10.0 g/dL. A colonoscopy and colon biopsy reveal adenocarcinoma. Which of the following is the most likely predisposing lesion that led to this condition? (A) FAP syndrome (B) Hyperplastic polyp (C) Long-standing ulcerative colitis (D) Peutz-Jeghers polyp (E) Tubular adenoma

The answer is A. Alzheimer disease is a major cause of dementia and is characterized by relatively slow, progressive memory loss followed in later stages by motor problems, contractures, and paralysis. Morphologic findings in Alzheimer disease include neurofibrillary tangles within neurons in the cerebral cortex, neuritic (senile) amyloid plaques, Hirano bodies, and generalized cerebral atrophy.

A 70-year-old man presents with loss of memory for recent events. He has forgotten his grandchildren's names, and he has been unable to manage his personal finances. Also, he has lost his way while driving to familiar locations. Which of the following is the most likely diagnosis? (A) Alzheimer disease (B) ALS (C) Creutzfeldt-Jakob disease (D) Huntington disease (E) Parkinson disease

The answer is B. The diagnosis is multiple (plasma cell) myeloma, a neoplastic proliferation of malignant plasma cells (mature B cells, not T cells). Death is often caused by renal insufficiency caused by myeloma kidney. The average age of presentation is approximately 70 years of age. IgM myeloma is very uncommon. Both the neoplastic cells and the serum protein spike are monoclonal rather than polyclonal, and the monoclonal spike protein is most frequently an IgG or an IgA.

A 70-year-old man presents with severe bone pain and frequent respiratory infections. Serum protein electrophoresis demonstrates an M protein spike in the gamma region. Radiographs of the skull, long bones, and spine demonstrate multiple "punchedout" lesions, and bone marrow aspiration demonstrates large numbers of neoplastic plasma cells. Which of the following statements is true of this disorder? (A) Although this patient presents at 70 years of age, the average age of presentation is 50 years of age. (B) Renal insufficiency is a common cause of death. (C) The M spike is most often an IgM. (D) The M spike is most often polyclonal in nature. (E) This disorder is the most common T-cell neoplasm.

The answer is D. The basal cell carcinoma shown in the figure has typical palisading of the nuclei of the cells at the periphery of the tumor cell clusters. Unlike squamous cell carcinoma, this tumor does not originate in preexisting actinic keratosis.

A 70-year-old retired farm worker is seen for evaluation of a pearly-appearing papule on the face just below and lateral to the left eye. The lesion is covered by small telangiectatic vessels. An excisional biopsy is performed, and the microscopic appearance is similar to that seen in the figure. Which of the following is characteristic of this disorder? (A) Distal metastases common at the time of initial diagnosis (B) Frequent origin in a preexisting actinic keratosis (C) Hamartomatous non-neoplastic lesion (D) Most frequent occurrence is on head or neck

The answer is A. This is the typical presentation of temporal, or giant cell, arteritis. Along with involvement of the branches of the carotid artery, such as the temporal or ophthalmic arteries, there is a close association with a complex of symptoms of proximal muscles termed polymyalgia rheumatica. Hypersensitivity vasculitis manifests with palpable purpura and is associated with drugs, food, or infectious agents. Takayasu arteritis involves the vessels of the aortic arch, whereas Kawasaki disease involves the coronary arteries. Wegener granulomatosis involves vessels of the respiratory tract and kidney.

A 74-year-old woman presents to her primary care physician with malaise, proximal muscle pain, morning stiffness, and claudication of the jaw, along with occasional visual impairment. On examination, palpation along the lateral aspect of the head elicits tenderness. This is likely a result of (A) giant cell arteritis. (B) hypersensitivity vasculitis. (C) Kawasaki disease. (D) Takayasu arteritis. (E) Wegener granulomatosis.

The answer is C. Subdural hematoma is characteristically caused by venous bleeding, most often from veins that join the cerebrum to venous sinuses within the dura. The venous hemorrhage typically arrests early, but the volume of the hematoma gradually increases because of osmotic imbibing of water. This results in a slowly enlarging tumorlike mass characterized clinically by gradual signs of cerebral compression occurring hours to days or even weeks after head injury.

A 75-year-old woman appears well after slipping on wet pavement and striking the right side of her head. When questioned, she says that she does not remember the fall. Subsequently she complains of persistent headache and confusion. Magnetic imaging studies reveal a subdural hematoma over the lateral aspect of the right cerebral hemisphere. Which of the following is a well-known association or characteristic of this disorder? (A) Bleeding from arteries of the circle of Willis (B) Rapidly progressive cerebral compression (C) Characteristically caused by venous hemorrhage (D) Laceration of branches of middle meningeal artery (E) Causally associated with hypertension

The answer is C. The combination of coarse asymmetric corticomedullary scarring, deformity of the renal pelvis and calyces, and tubular atrophy is characteristic of chronic pyelonephritis. When the atrophic tubules contain eosinophilic proteinaceous casts, the resultant similarity in appearance to thyroid follicles is referred to as "thyroidization." Although an infectious etiology is assumed, the infectious agent is often not demonstrable.

A 78-year-old man with long-standing prostatic nodular hyperplasia dies of a stroke. At autopsy, both kidneys demonstrate coarse asymmetric renal corticomedullary scarring, deformity of the renal pelvis and calyces, interstitial fibrosis, and atrophic tubules containing eosinophilic proteinaceous casts. These findings are most suggestive of (A) Berger disease. (B) chronic analgesic nephritis. (C) chronic pyelonephritis. (D) membranoproliferative glomerulonephritis. (E) renal papillary necrosis.

The answer is C. The most common cause of death that occurs during acute rheumatic fever is cardiac failure secondary to myocarditis.

A 9-year-old girl is diagnosed with acute rheumatic fever. Instead of recovering as expected, her condition worsens, and she dies. Which of the following is the most likely cause of death? (A) Central nervous system involvement (B) Endocarditis (C) Myocarditis (D) Pericarditis (E) Streptococcal sepsis

The answer is C. Neonatal respiratory distress syndrome (hyaline membrane disease) is the most common cause of respiratory failure in newborns and results from a deficiency of surfactant and immature development of the lungs. Surfactant reduces surface tension within the lung, facilitating expansion by inspiration and thus preventing atelectasis during expiration. The classically referenced indicator of fetal pulmonary maturity is a lecithin:sphingomyelin ratio of approximately 2:1 in the amniotic fluid, although techniques like lamellar body counts and the fluorescence polarization assay are now more commonly used to evaluate fetal lung maturity. Predisposing factors include prematurity, maternal diabetes mellitus, and birth by cesarean section. Known complications of this condition include bronchopulmonary dysplasia, patent ductus arteriosus, intraventricular brain hemorrhage, and necrotizing enterocolitis.

A female infant is born prematurely at 28 weeks' gestation. Shortly after birth, she shows signs of dyspnea, cyanosis, and tachypnea. She is placed on a ventilator for assisted breathing, and a diagnosis of neonatal respiratory distress syndrome (hyaline membrane disease) is made. Which of the following is the cause of this syndrome? (A) Bronchopulmonary dysplasia (B) Intraventricular brain hemorrhage (C) Lack of fetal pulmonary maturity and deficiency of surfactant (D) Necrotizing enterocolitis (E) Patent ductus arteriosus

The answer is D. The illustration demonstrates linear immunofluorescence, which is characteristic of disease caused by antiglomerular basement membrane antibodies. In Goodpasture syndrome, antibodies directed against antigens in the basement membranes of the glomeruli as well as the pulmonary alveoli result in both hemorrhagic pneumonitis with hemoptysis and glomerular disease with hematuria.

A glomerular immunofluorescent pattern for IgG similar to that shown in the figure would be expected in which of the following patients? (A) A 3-year-old girl with recurrent bouts of the nephrotic syndrome (B) A 9-year-old boy with "smoky" urine 2 weeks after recovery from a streptococcal infection (C) An 18-year-old woman with nephrotic syndrome and progressive chronic renal disease (D) A 25-year-old man with hemoptysis and hematuria (E) A 26-year-old woman with a "butterfly" rash

The answer is E. This patient has anaplastic large cell lymphoma (aLCL)which is characterized by pleomorphic large lymphocytes with abundant cytoplasm and frequent mitoses. Morphologically it may mimic metastatic carcinoma. The constituent cells are positive for CD30. Scattered cells showing kidney or donut-shaped nuclei with an eosinophilic region adjacent to the nucleus are known as "Hallmark cells." They do not show lacunar cells, which are a feature of NS classical Hodgkin lymphoma. The majority of cases in children shows a t(2;5) translocation which correlates with ALK positivity and improved survival. Although it is more common in children, it shows a bimodal incidence with a second peak in older individuals.

A lymph node from a 10-year-old boy reveals large pleomorphic lymphocytes with CD30 positivity, frequent mitoses, and scattered cells bearing kidney-shaped nuclei. Which of the following statements is true? (A) t(2;5) translocation imparts a worsened prognosis. (B) ALK protein immunostaining is seen in a minority of cases. (C) This tumor occurs exclusively in children. (D) Lacunar cells are a feature. (E) Hallmark cells are a feature.

The answer is A. The findings described are characteristic of congenital agammaglobulinemia of Bruton, an X-linked disorder characterized morphologically by agammaglobulinemia, absence of plasma cells, and absent or poorly defined germinal centers in the lymph nodes. Leukocyte functions, such as phagocytosis and bacterial killing, are unimpaired. T lymphocytes are unaffected.

A male infant was seen for recurrent bacterial infections beginning at age 6 months. Immunoglobulin assay reveals absent IgG. An additional expected finding is (A) absence of germinal centers in the lymph nodes. (B) autosomal recessive inheritance. (C) decreased CD4+ to CD8+ T lymphocyte ratio. (D) defective leukocytic bacterial killing. (E) impaired phagocytosis.

The answer is D. Maternal antibodies provide passive immunization and protection from bacterial infection during the first months of life in children with congenital agammaglobulinemia.

A male infant was seen for recurrent bacterial infections beginning at age 6 months. In this infant, the period of well-being for the first 6 months of life is best explained by (A) antibacterial substances supplied by breast-feeding. (B) deficient opsonization due to immaturity of complement synthesis. (C) delayed responsiveness of lymphocytes to mitogenic stimuli. (D) protection by maternal antibodies. (E) need for a viral infection to trigger immune destruction of thymic tissue.

The answer is B. Administration of anti-D antiserum to a D-negative mother at the time of delivery of a D-positive child prevents maternal alloimmunization by removing fetal red cells from the maternal circulation.

A primiparous D-negative (Rh-negative) mother has just delivered a D-positive child. Administration of which of the following substances would be indicated? (A) Anti-D IgG to child (B) Anti-D IgG to mother (C) D-positive red cells to child (D) D-positive red cells to mother

The answer is D. Sensitivity, the ability of a test to be positive in patients with a specific disease, is expressed as the ratio of true positives to the total number of patients known to have the disease. In this example, the number of true positive results is the same as the total number of positive results because the test is stated to always be negative in healthy persons; that is, the specificity is 100%, and there are no false positives. Thus, the sensitivity is (160/[0.2 31000]) 3100, or 80%. In perhaps simpler terms, a population of 1,000 with a prevalence of 20% has 200 persons with the disease. Of these, 160 test positive. Thus, the sensitivity is 160/200 3100, or 80%.

A medical student participating in a summer fellowship program begins to work on an inborn error of metabolism referred to as Z disease. In the course of his investigations, he develops a diagnostic procedure for the disorder. A survey of unaffected normal subjects demonstrates the Z test to always be negative in this group; however, the Z test is found to be positive in 160 of 1,000 subjects from a geographically isolated population group in which the prevalence of the disorder (as assessed by clinical features) is known to be 20%. What is the sensitivity of the test? (A) 65% (B) 70% (C) 75% (D) 80% (E) 85%

The answer is A. A total lack of glucuronyl transferase results in Crigler-Najjar syndrome I, which is invariably fatal by 18 months secondary to kernicterus. Gilbert syndrome is typically mild and usually not detected until later in life. Hemolytic disease of the newborn is due to blood group incompatibility between mother and child, and the bilirubinemia is secondary to the inherently decreased glucuronyl transferase with superimposed hemolytic anemia in the child, further overwhelming the conjugation machinery. Neonatal glucuronyl transferase is relatively deficient, although present, in normal infants, contributing to the transient condition termed physiologic jaundice of the newborn. Rotor syndrome is a relatively benign condition resulting in conjugated hyperbilirubinemia.

A neonate has been persistently jaundiced from birth despite aggressive phototherapy and exchange transfusions. Laboratory studies demonstrate significantly elevated unconjugated bilirubin. Tests from an outside laboratory confirm the total absence of glucuronyl transferase activity. The neonate most likely has which of the following condition? (A) Crigler-Najjar syndrome (B) Gilbert syndrome (C) Hemolytic disease of the newborn (D) Physiologic jaundice of the newborn (E) Rotor syndrome

The answer is C. A meningomyelocele is a neural tube defect in which both the meninges and spinal cord are included in the herniated tissue. A meningocele is a defect in which the herniated membranes consist only of meninges, and spina bifida occulta does not manifest any apparent abnormalities. Anencephaly is the diminution or absence of fetal brain tissue. In neural tube defects, an increase in α-fetoprotein in both the maternal serum and the amniotic fluid is noted.

A newborn girl is found to have herniation of both the spinal cord and meninges through a defect in the vertebral arch of the spinal column. Her mother had not had prenatal care and had not taken nutritional supplements during pregnancy. Which of the following best describes this defect? (A) Spina bifida occulta (B) Meningocele (C) Meningomyelocele (D) Anencephaly

The answer is D. In this very high-prevalence subgroup, the predictive value of a positive result is 92%. These three examples illustrate the profound effect of prevalence on the usefulness of a diagnostic test. Although these examples are centered on a fictitious laboratory-based employment standard, the real-life corollary is the usefulness of screening procedures in medical practice. A test procedure is more likely to be useful when directed to patients who have a high likelihood of the disorder, as dictated by good clinical judgment, than when used in an indiscriminate, unselective fashion.

A newly established physician in a small community is asked to advise a local committee that is designing a set of physical standards for potential volunteer firefighters. He is told that firefighters must test negative for a new blood chemistry test, the S test, which predicts susceptibility to smoke inhalation damage. It is known (for unexplained reasons) that susceptibility to smoke damage is uncommon in brown-eyed persons, more common in green-eyed persons, and very common in blue-eyed persons. The S test is found to have a sensitivity of 99% and a specificity of 99%. The S test is further evaluated in three highly selected population subgroups, each with differing eye colors (and, thus, with different prevalences of susceptibility to smoke inhalation damage). What is the likelihood that the S test will correctly identify a potential firefighter as one with an unacceptable sensitivity to smoke inhalation damage if the applicant is blue-eyed(prevalence of susceptibility is known to be 1 in 10)? (A) 1% (B) 9% (C) 50% (D) 92% (E) 99%

The answer is E. Of the choices listed, only membranous glomerulonephritis is an immune complex disease.

A renal biopsy taken from a 23-year-old woman with nephrotic syndrome prominently features glomerular immune complex deposits. Of the following disorders that affect the glomerulus, which is suggested by the findings? (A) Amyloidosis (B) Diabetic nephropathy (C) IgA nephropathy (D) Minimal change disease (lipoid nephrosis) (E) Membranous glomerulonephritis

The answer is B. A population arbitrarily set at 100,000 (to avoid the use of decimal fractions) includes 100 potential firefighters with unacceptable smoke damage susceptibility (0.1% of 100,000) and 99,900 individuals resistant to smoke damage (100,000 2100). Of the 100 applicants, 99 test positive (99% of 100). Of the 99,900 unaffected persons, 999 test positive ([100 299]% of 99,900). The total number of positive results is thus 99 (TP) 1999 (FP). The predictive value of a positive result is then 99/(99 1999), or approximately 9%.

A newly established physician in a small community is asked to advise a local committee that is designing a set of physical standards for potential volunteer firefighters. He is told that firefighters must test negative for a new blood chemistry test, the S test, which predicts susceptibility to smoke inhalation damage. It is known (for unexplained reasons) that susceptibility to smoke damage is uncommon in brown-eyed persons, more common in green-eyed persons, and very common in blue-eyed persons. The S test is found to have a sensitivity of 99% and a specificity of 99%. The S test is further evaluated in three highly selected population subgroups, each with differing eye colors (and, thus, with different prevalences of susceptibility to smoke inhalation damage). What is the likelihood that the S test will correctly identify a potential firefighter as one with an unacceptable sensitivity to smoke inhalation damage if the applicant is brown-eyed(prevalence of susceptibility in this group is known to be 1 in 1,000)? (A) 1% (B) 9% (C) 50% (D) 92% (E) 99%

The answer is C. A similar computation yields a predictive value of a positive result of 50%.

A newly established physician in a small community is asked to advise a local committee that is designing a set of physical standards for potential volunteer firefighters. He is told that firefighters must test negative for a new blood chemistry test, the S test, which predicts susceptibility to smoke inhalation damage. It is known (for unexplained reasons) that susceptibility to smoke damage is uncommon in brown-eyed persons, more common in green-eyed persons, and very common in blue-eyed persons. The S test is found to have a sensitivity of 99% and a specificity of 99%. The S test is further evaluated in three highly selected population subgroups, each with differing eye colors (and, thus, with different prevalences of susceptibility to smoke inhalation damage). What is the likelihood that the S test will correctly identify a potential firefighter as one with an unacceptable sensitivity to smoke inhalation damage if the applicant is green-eyed(prevalence of susceptibility is known to be 1 in 100)? (A) 1% (B) 9% (C) 50% (D) 92% (E) 99%

The answer is C. Medullary carcinoma of the thyroid is a calcitonin-producing tumor of C cells of the thyroid. Calcitonin contributes to amyloid deposition within the tumor. Tumor cells with "Orphan Annie" nuclei and the presence of psammoma bodies are seen in papillary carcinoma of the thyroid. Infiltrates of lymphocytes with germinal center formation are seen in Hashimoto thyroiditis. In Riedel thyroiditis, the thyroid is replaced by fibrous tissue and can clinically mimic carcinoma.

A palpable mass is noted in the right lobe of the thyroid of a 45-year-old man who visits his physician for a periodic checkup. A biopsy is performed and results in a diagnosis of medullary carcinoma of the thyroid. Which of the following histologic features of thyroid disease would most likely be present in this biopsy specimen? (A) Tumor cells with "Orphan Annie" nuclei (B) Psammoma bodies (C) Tumor cells embedded in an amyloid-laden stroma (D) Infiltrates of lymphocytes with germinal center formation (E) Replacement of the thyroid with fibrous tissue

The answer is B. Amplification of the HER-2/neuoncogene is frequently observed in breast cancer, and an increased degree of amplification is thought to be a negative prognostic indicator.

A pathologist examines an excisional biopsy specimen and confirms the working diagnostic impression of adenocarcinoma. Because of the nature of the tumor, he requests molecular diagnostic evaluation by the molecular pathology laboratory. Amplification of the HER-2/neuoncogene is demonstrated. This finding is a negative prognostic indicator in carcinoma of the (A) adrenal. (B) breast. (C) kidney. (D) stomach. (E) thyroid.

The answer is E. Granulation tissue is formed in healing wounds and consists of young fibroblasts and newly formed capillaries. Cat-scratch disease, foreign body reaction, histoplasmosis, and tuberculosis are all well-known causes of granulomatous inflammation and have nothing to do with granulation tissue.

A pathologist examining a histologic preparation from an autopsy finds a lesion with abundant granulation tissue. This finding is most likely to be indicative of which of the following? (A) Cat-scratch disease (B) Foreign body reaction (C) Histoplasmosis (D) Tuberculosis (E) Wound healing

The answer is E. Megaloblastic anemia due to deficiency of vitamin B12 or folate is characterized by oval macrocytes, hypersegmented neutrophils, and decreased platelets.

A patient with severe anemia has a peripheral blood smear with oval macrocytes, hypersegmented neutrophils, and decreased platelets. The most likely cause of the anemia is (A) a red cell membrane protein defect. (B) an amino acid substitution in the β-globin chain. (C) iron deficiency. (D) marrow hypoplasia. (E) vitamin B12 or folate deficiency.

The answer is D. Megaloblastic anemia associated with severe malnutrition is most often due to folate deficiency.

A patient with severe anemia has a peripheral blood smear with oval macrocytes, hypersegmented neutrophils, and decreased platelets. The patient is found to be a severely malnourished alcoholic. The most likely cause of this disorder is (A) aberrant intestinal bacterial flora. (B) Crohn disease. (C) fish tapeworm infestation. (D) folate deficiency. (E) pernicious anemia.

The answer is A. This question is actually the same as question 5, just expressed in more familiar terms. The likelihood that this patient actually has ovarian cancer is determined by the predictive value of a positive result, which in this instance is 66%.

A physician participating in this study finds that one of his patients has a positive test result. What is the likelihood that this patient has ovarian cancer? (A) 66% (B) 80% (C) 90% (D) 95%

The answer is A. Tests designed to screen for disease should be highly sensitive in order to capture everyone in the population with the disease. If a test is highly sensitive, then a negative result essentially rules out disease. Test B should be used as a confirmatory test. If a test is highly specific, then a positive result essentially rules in disease.

A screening program is started to test potential blood donors for human immunodeficiency virus infection. Two screening tests are under consideration, and you are asked for your opinion on the use of these testing procedures. Test A has a known sensitivity of 99.9% and a specificity of 99.9%. Test B has a sensitivity of only 90%, but has a specificity of 100%. Which of the following should be your advice on the use of these procedures for screening? (A) For screening purposes, Test A should be used instead of Test B, since it has the higher sensitivity. (B) For screening purposes, Test B should be used instead of Test A, since it has the higher specificity.

The answer is A. In a typical well-differentiated bronchioloalveolar carcinoma, tumor cells line the walls of terminal air spaces, as shown in the illustration. When the tumor is localized to a single nodule, it is potentially curable by surgical resection.

A single nodule was resected from the peripheral portion of the right lower lobe of the lung of a 45-year-old woman. The microscopic findings were similar to those shown in the illustration. The diagnosis is (A) bronchioloalveolar carcinoma. (B) carcinoid. (C) mesothelioma. (D) small cell carcinoma. (E) squamous cell carcinoma.

The answer is C. Deficiency of 21-hydroxylase is the cause of the most common of the adrenogenital syndromes. This enzyme deficiency results in decreased cortisol, decreased mineralocorticoids, and an increase in sex hormones, with resultant salt-losing hypotension and virilization (masculinization). Deficiency of 11-hydroxylase causes clinical findings similar to those of 21-hydroxylase deficiency, except for hypertension secondary to increased deoxycorticosterone, which has aldosterone-like activity. Deficiency of 17-hydroxylase results in decreased cortisol, increased mineralocorticoids, and decreased sex hormones. Amylin accumulates in diabetic islets, and 1α-hydroxylase is deficient in vitamin D-dependent rickets (type I).

A tentatively female newborn has ambiguous genitalia. What appears to be a vagina is associated with a significantly enlarged clitoris resembling a penis. Other findings include hyponatremia, hyperkalemia, and hypotension. Deficiency of which of the following is suggested by these findings? (A) 11-Hydroxylase (B) 17-Hydroxylase (C) 21-Hydroxylase (D) Amylin (E) 1α-Hydroxylase

The answer is B. The lesion shown is a papillary carcinoma of the thyroid, which is the most common form of thyroid cancer. This tumor most often remains localized to the thyroid and adjacent tissues for many years, even when local lymph nodes are involved. Papillary carcinoma is almost always nonfunctional.

A tumor similar to that shown in the illustration is observed in a biopsy specimen from the thyroid of a 50-year-old woman. An adjacent lymph node is also involved. Which of the following descriptions of this tumor is most appropriate? (A) Functional tumor resulting in thyrotoxicosis (B) Slow-growing lesion with relatively good prognosis (C) Origin from C cells (D) Calcitonin-producing tumor (E) Tumor with amyloid-containing stroma

The answer is B. If a test is used to monitor progressive changes in a patient, the test results must be highly reproducible, that is, they must have a high degree of precision. The CV, a measure of precision, describes the variability of repetitive measurements on the same sample. For most laboratory procedures, precision is determined by successive daily measurements on a stored, properly preserved sample. The standard deviation of the repetitive measurements is divided by the mean of the values, yielding the CV. The rule is that the more reproducible the test, the lower the CV.

A urologist is using prostate-specific antigen (PSA) measurements to monitor the progress of one of his patients, a 73-year-old man who has high-grade prostatic intraepithelial neoplasia. At a prior office visit, the PSA had been 6 ng/mL (normal less than 4 ng/mL). At the current visit, PSA is 6.2 ng/mL. Which of the following laboratory measurements is most useful to the urologist in assessing the significance of the change in PSA? (A) Accuracy (B) Coefficient of variation (C) Positive predictive value (D) Sensitivity (E) Specificity

The answer is A. The combination of aortic diastolic murmur, "water-hammer" pulse, and wide pulse pressure is an indicator of aortic valve insufficiency. Although now rare in the United States, tertiary syphilis remains the most common cause of this abnormality in many parts of the world. Aneurysm of the ascending aorta commonly accompanies this valvular lesion. The other listed choices are all complications of atherosclerosis, which infrequently involves the ascending aorta.

A volunteer physician in a village in Rwanda sees a 45-year-old man who has an aortic diastolic murmur and a "water-hammer" pulse. Blood pressure is 200/70. Which of the following associated abnormalities is most likely? (A) Aneurysm of ascending aorta (B) Cerebral infarct (C) Gangrene of small bowel (D) Mural thrombosis (E) Myocardial infarct

The answer is B. The most common primary malignant neoplasm of bone is osteosarcoma. The classic radiographic findings are the Codman triangle (periosteal elevation by new bone formation) and the "sunburst" appearance (extension of tumor cells through the periosteum).

An 18-year-old man presents with left knee pain he's had for several months. Knee radiograph demonstrates elevation of the periosteum of the bone, with areas of a "sunburst" appearance. What is the likely diagnosis? (A) Osteochondroma (B) Osteosarcoma (C) Giant cell tumor (D) Ewing sarcoma (E) Chondrosarcoma

The answer is A. Sensitivity is defined as the ability of a test to be positive in patients with a specific disease; it is the percentage of known positives who actually test positive. It is calculated by dividing the number of positive results in patients (TP) by the total number of patients (TP 1FN).

A worker in a research laboratory is developing a new diagnostic procedure making use of a newly discovered marker protein for ovarian cancer. The laboratory has at its disposal 1,000 preserved serum samples from patients who are known to have had this tumor. The percentage of the samples that test positive with this new procedure is referred to as the (A) sensitivity. (B) specificity. (C) precision. (D) accuracy. (E) predictive value.

The answer is D. Nuclear pyknosis, along with karyorrhexis and karyolysis, is a sign of necrosis and is, of course, irreversible. Fatty change, formation of cell blebs or myelin figures, and swelling of the cell or of organelles are all reversible changes.

After a football injury, a 16-year-old boy survives a ruptured spleen, hemorrhagic shock, and profound hypotension; however, a period of severe oliguria follows, so the boy undergoes a kidney biopsy. Which one of the following renal tubular changes found represents irreversible cellular injury? (A) Fatty change (B) Formation of cell blebs (C) Formation of myelin figures (D) Nuclear pyknosis (E) Swelling of organelles

The answer is E. Progressive somnolence leading to metabolic acidosis (low bicarbonate with significant anion gap), coma, and severe dehydration, often with prerenal azotemia, are all strongly suggestive of diabetic ketoacidosis. Expected findings in this condition include increased serum and urine glucose and ketones.

After being picked up by the police, a runaway adolescent girl became increasingly somnolent, lapsing into a deep coma 72 hours later. Her respirations were rapid and deep, and she appeared to be severely dehydrated. Laboratory studies revealed a marked reduction in serum bicarbonate and a significant anion gap, as well as neutrophilic leukocytosis. The most likely additional laboratory abnormality is (A) a blood urea nitrogen (BUN):creatinine ratio of less than 1:10. (B) decreased serum cortisol. (C) decreased serum thyroxine (T4). (D) increased CSF protein with no parallel increase in cell count. (E) increased serum glucose.

The answer is A. Distinguishing endogenous insulin production from exogenous insulin (therapeutically or surreptitiously administered) is done by quantitation of C-peptide, a fragment of the proinsulin molecule split off during the synthesis of insulin. Circulating C-peptide is characteristically increased in patients with insulinoma. C-peptide is not increased by exogenous insulin administration because it is removed during the purification of commercial insulin preparations.

After suffering a seizure, a 23-year old woman is found to have profound hypoglycemia. Determination of which of the following would aid in differentiating exogenous hyperinsulinemia from endogenous hyperinsulinemia? (A) C-peptide (B) Gastrin (C) Glucagon (D) Proinsulin (E) Vasoactive intestinal peptide

The answer is A. Within the first few hours of acute blood loss, findings of hypovolemia predominate, especially with signs of hypovolemic shock, such as decreased blood pressure. It is likely that red cell indices (red blood cell counts, hemoglobin, and hematocrit) eventually decrease as a result of hemodilution. Increased MCHC is an unusual finding but is expected in hereditary spherocytosis.

An 18-year-old man is transported to the emergency department within 20 minutes of sustaining a stab wound to the chest. The patient is poorly responsive. Given that he may have lost as much as 1.5 liters of blood at the scene, which of the following is the most likely finding in this patient? (A) Decreased blood pressure (B) Decreased hematocrit (C) Decreased hemoglobin (D) Decreased red blood cell count (E) Increased MCHC

The answer is C. Gilbert syndrome is an extremely common cause of clinically insignificant unconjugated hyperbilirubinemia. The hyperbilirubinemia is episodic, with increases related to stress, fatigue, alcohol use, or recurrent infection. Crigler-Najjar syndrome I is often lethal, although Crigler-Najjar syndrome II is compatible with life and often presents with extreme jaundice and neurologic defects secondary to kernicterus. Both DubinJohnson syndrome and Rotor syndrome cause conjugated hyperbilirubinemia. Infectious mononucleosis can cause liver damage, but would likely be accompanied by malaise and fatigue.

An 18-year-old man presents to his family physician for a routine physical prior to moving away for college. Other than feeling slightly "stressed" by his soon-to-be new life situation, he has no complaints. A comprehensive metabolic panel reveals a modestly elevated unconjugated bilirubin but a near-normal aspartate aminotransferase and alanine aminotransferase. It is likely that the patient may have which of the following conditions? (A) Crigler-Najjar syndrome (B) Dubin-Johnson syndrome (C) Gilbert syndrome (D) Infectious mononucleosis (E) Rotor syndrome

The answer is E. Chickenpox (varicella), caused by the varicella-zoster virus, is a viral infection of childhood characterized by fever and a predominantly vesicular rash. Following overt varicella, the virus can remain latent for years in dorsal root ganglia and reappear several years later as herpes zoster (shingles).

An 8-year-old boy presents with an intensely pruritic vesicular rash and fever. Several playmates have had similar findings. The etiology of this common infectious exanthem is (A) autoimmune. (B) bacterial. (C) fungal. (D) parasitic. (E) viral.

The answer is B. Medulloblastoma, a highly malignant tumor, is one of the most frequently occurring malignancies of early childhood. The posterior cranial fossa is the most common site of this tumor. Other high-incidence tumors of early childhood include acute leukemia, Wilms tumor, and adrenal neuroblastoma.

An 8-year-old boy presents with irritability, vomiting, and gait ataxia. CT scan demonstrates a midline tumor of the cerebellum. Which of the following is the most likely diagnosis? (A) Glioblastoma multiforme (B) Medulloblastoma (C) Meningioma (D) Schwannoma (Neurilemmoma) (E) Oligodendroglioma

The answer is D. t(12;21) and high hyperdiploidy are associated with positive outcomes in ALL, whereas t(9;22), t(1;19), 11q23 rearrangements, and hypodiploidy are all associated with poor prognosis in these leukemias. Note that the 9;22 translocation seen in ALL is cytogentically identical but molecularly distinct from the one typically seen in CML.

An 8-year-old girl is diagnosed with ALL. Which of the following cytogenetic changes would confer a good prognosis? (A) t(9;22) (B) 11q23 rearrangement (C) Hypodiploidy (D) Hyperdiploidy (E) t(1;19)

The answer is A. Seborrheic keratosis is an extremely common benign neoplasm occurring in older persons. This neoplasm is manifested by sharply demarcated, raised papules or plaques with a "pasted-on" appearance on the head, trunk, and extremities.

An 80-year-old man presents with sharply demarcated, light brown, flat macules varying markedly in size. The lesions have the appearance of being "stuck on" or "pasted on," and they are particularly numerous on the trunk. Microscopically, sheets of small basaloid cells with some melanin production are seen. Keratin production occurs at the surface, and numerous small keratinfilled cysts are apparent. Which of the following is the likely diagnosis? (A) Seborrheic keratosis (B) Dermatofibroma (C) Keratoacanthoma (D) Actinic keratosis (E) Acanthosis nigricans

The answer is C. Vitamin C is required for hydroxylation of proline and lysine residues, which are required steps in collagen and osteoid matrix synthesis. Poor collagen formation contributes to impaired wound healing and fragility of capillary walls, which in turn leads to abnormal bleeding. Vitamin C also maintains the reduced state of metabolically active agents, such as iron and tetrahydrofolate. The maintenance of iron in its divalent ferrous form is required for intestinal iron absorption. Thus iron absorption is decreased, rather than increased, in vitamin C deficiency. Defective osteoid matrix formation occurs in vitamin C deficiency.

An 80-year-old woman on a "tea and toast" diet presents with bleeding gums, petechiae and easy bruising, and pain in her arms and legs. She states that she almost never eats fruits and vegetables. She is diagnosed with vitamin C deficiency. Which of the following is the basis of the clinical abnormalities that occur as a result of lack of vitamin C? (A) Defective calcification of osteoid matrix (B) Increased intestinal absorption of iron (C) Defective hydroxylation of proline and lysine (D) Increased proliferation of collagen and fibrous tissue (E) Destruction of endothelial cells

The answer is C. Vitamin C deficiency occurs in infants aged 6 to 12 months who are fed a diet deficient in citrus fruits or vegetables, or in elderly persons who maintain a "tea and toast" diet. Vitamin C cannot be synthesized by the body, and thus must be supplied by the diet. The body's reserve of vitamin C is approximately 1 to 3 months with complete dietary absence. Early signs of vitamin C deficiency include those found in this patient.

An 80-year-old woman presents with recent onset of primary hemostatic (mucocutaneous) bleeding. Questioning reveals that she has been maintaining a "tea and toast" diet for the past 4 months. Her gums are hemorrhagic and spongy in consistency, and gingival bleeding is evident. Perifollicular hyperkeratotic papules, each surrounded by a hemorrhagic halo, are scattered over the lower extremities, and each papule surrounds a twisted, corkscrew-like hair. A nutritional deficiency is suspected. Deficiency of which of the following nutrients is most likely related to the findings in this patient? (A) Vitamin A (B) Vitamin B12 (C) Vitamin C (D) Vitamin K (E) Protein

The answer is A. Antibody responses to the more than 80 differing carbohydrate capsular antigens of the various strains of S. pneumoniaeare generally T-cell-independent, and antibody formation is entirely B-cell-mediated. Because of this, memory cells are not formed, and long-lasting immunity is not achieved.

An 80-year-old woman, a retirement home resident, has multiple bouts of pneumonia caused by Streptococcus pneumoniae. In an attempt to prevent such infections, polyvalent vaccines directed at multiple serotypes of the organism have been administered but have not elicited long-acting immunity. Which of the following is the probable explanation for this phenomenon? (A) Memory T lymphocytes respond poorly to polysaccharide antigens. (B) S. pneumoniaeevades host immune response by forming capsular coatings composed of host proteins and recognized as "self" antigens. (C) The bacterial capsule binds C3b, facilitating activation of the alternative complement pathway, inducing complement-mediated lysis, and preventing immunization. (D) The capsular polysaccharides of S. pneumoniaehave limited hapten potential. (E) The surface carbohydrate capsule on the surface of the organism acts as an opsonin, facilitating phagocytosis by neutrophils, thus preventing immunization.

The answer is B. Osteoporosis is characterized by a decrease in bone mass due to loss of bone matrix, and it is unassociated with abnormalities in mineral metabolism. This condition is the most common bone disorder in older persons. It occurs most commonly in elderly women and is associated with the postmenopausal state and estrogen deficiency. Characteristics include fractures, kyphosis, and shortened stature. Predisposing factors include physical inactivity, hypercorticism, hyperthyroidism, and calcium deficiency. Serum calcium and phosphate levels are typically normal.

An 88-year-old woman with marked kyphosis and loss of height that had been gradually progressive over many years experiences the sudden onset of acute back pain following a sudden change in position. Radiographic examination demonstrates generalized osteopenia and a fracture of a lower thoracic vertebra. Which of the following is an association or characteristic of the patient's generalized bone disorder? (A) Increased serum calcium and phosphate levels (B) Postmenopausal state and estrogen deficiency (C) Physical inactivity (D) Hypothyroidism (E) Excessive calcium intake

The answer is E. Hemolytic disease of the newborn is a type II hypersensitivity reaction mediated by antibodies directed against intrinsic cell surface antigens (with complement-induced cell lysis). Modern management requires administration of IgG anti-Rh antibodies to the mother at the birth of a first Rh-positive child in order to avoid maternal sensitization. Another type of type II reaction is exemplified by the reaction of antibodies with cell-surface receptors, as occurs in Graves disease. Activation of sensitized CD4+T cells with release of cytokines is descriptive of type IV cell-mediated reactions, such as the delayed-type cell-mediated hypersensitivity exemplified by poison ivy dermatitis. "Graftversus-neonate" is a fictitious condition, but transfusion-mediated transplant rejection is an ever-present danger when immunodeficient persons receive transfusions that contain immunocompetent precursor cells. IgE-mediated degranulation of mast cells and basophils underlies type I hypersensitivity reaction, such as allergic asthma, hay fever, or anaphylaxis. Immune complex deposition with secondary inflammation occurs in generalized immune complex disorders, such as SLE.

An Rh-positive neonate born to an Rh-negative mother has severe anemia and rapidly progressive jaundice. An older sibling, now also known to be Rh-positive, was delivered by a neighbor at home. The mother is unaware of any prior special testing or therapy related to anemia, jaundice, or "Rh problems." Which of the following mechanisms underlies the clinical scenario described here? (A) Activation of sensitized CD4T cells with release of cytokines (B) "Graft-versus-neonate" response of immunologically incompetent newborn (C) Immunoglobulin (Ig)E-mediated degranulation of mast cells and basophils (D) Immune complex deposition with secondary inflammatory reaction (E) Interaction of IgG antibodies with intrinsic cell-surface antigens

The answer is D. The patient presents with signs and symptoms of the WaterhouseFriderichsen syndrome, a devastating consequence of disseminated meningococcal infection. The disease is characterized by hemorrhagic destruction of the adrenals complicated by disseminated intravascular coagulation.

An acutely ill 18-year-old female college student is brought to the emergency department by her roommate. The patient is febrile and markedly hypotensive, and her mental status is obtunded. Numerous petechial and purpuric hemorrhages are scattered over the trunk, and aspiration of a lesion reveals neutrophils engulfing gram-negative diplococci. Serum sodium is markedly decreased, and serum potassium is increased. Coagulation testing reveals increased prothrombin time, activated partial thromboplastin time, and fibrin-fibrinogen split products. Which of the following is most likely? (A) Conn syndrome (B) Hyperprolactinoma (C) Neuroblastoma (D) Waterhouse-Friderichsen syndrome (E) Sipple syndrome

The answer is D. A rapidly fatal course with severe hypertension, left ventricular hypertrophy and failure, papilledema, and renal dysfunction is characteristic of malignant hypertension. This syndrome is most frequently seen in relatively young African-American men. The defining renal arteriolar lesion, malignant nephrosclerosis (hyperplastic arteriolosclerosis, fibrinoid necrosis, necrotizing arteriolitis), and the associated necrotizing glomerular lesion result in capillary rupture and the consequent "flea-bitten" appearance of the surfaces of the kidneys due to petechial hemorrhages.

An autopsy is performed on a 35- year-old African-American man who died after a brief illness characterized by papilledema, severe hypertension, left ventricular hypertrophy and failure, and renal dysfunction. The most likely findings in the kidney are (A) finely granular renal surface and hyaline arteriolosclerosis of afferent arterioles. (B) swollen, hypercellular, "bloodless" glomeruli. (C) nodular mesangial accumulations of basement membrane-like material and hyaline arteriolosclerosis of afferent and efferent arterioles. (D) surface covered with multiple petechial hemorrhages, hyperplastic arteriolosclerosis, and necrotizing glomerulitis. (E) swollen, pale kidneys and marked accumulation of lipid in convoluted tubules.

The answer is B. Sustained ethanol abuse and progressive dementia are strongly suggestive of the Wernicke-Korsakoff syndrome, which is due to thiamine deficiency, most often in association with chronic alcoholism. Clinical characteristics include the Wernicke triad (confusion, ataxia, and ophthalmoplegia) and often Korsakoff psychosis, characterized by memory loss and confabulation (making up stories in an attempt to hide the inability to remember). The morphologic counterparts of these changes include degeneration of the mamillary bodies and of paramedian masses of gray matter.

An autopsy is performed on a 60- year-old man with a history of sustained ethanol abuse. There had been a history of progressive dementia with marked memory loss manifested by a tendency to fabricate false accounts of recent events. Additionally, confusion, ataxic gait, and paralysis of eye movements had been noted. The most likely findings in the brain are (A) amyloid-containing neuritic plaques within cerebral cortex, amygdala, and hippocampus. (B) degeneration of mamillary bodies and paramedian masses of gray matter. (C) depigmentation of substantia nigra and locus ceruleus. (D) diffuse cortical atrophy with hydrocephalus ex vacuo. (E) multiple lacunar infarcts and progressive subcortical demyelination.

The answer is C. Idiopathic Parkinson disease is manifested morphologically by depigmentation of cells of the substantia nigra and locus ceruleus.

An autopsy is performed on a 75-yearold man, who for the past several years had had a pill-rolling tremor in his hand, slowing of his movements, and muscle rigidity. Autopsy findings reveal depigmentation of the substantia nigra and locus ceruleus. Which of the following is the most likely diagnosis? (A) Alzheimer disease (B) Huntington disease (C) Idiopathic Parkinson disease (D) Myasthenia gravis (E) Wernicke-Korsakoff syndrome

The answer is D. The most important factor in the pathogenesis of endometrial carcinoma appears to be prolonged estrogen stimulation, such as that associated with estrogen therapy or estrogen-secreting tumors. Obesity and conditions associated with it, such as diabetes mellitus or hypertension, may contribute to hyperestrinism because estrone can be synthesized in peripheral fat cells. Arsenic exposure is associated with carcinomas of the lung and skin and with hepatic angiosarcoma. BRCAmutations are associated with breast and ovarian cancer. Endometriosis is not a neoplasm, although interestingly its incidence is increased in patients with certain gynecologic malignancies, most notably clear cell carcinomas of the ovary and endometrium. Sexual promiscuity is a risk factor for cervical cancer, not endometrial cancer.

Cytologic findings consistent with carcinoma of the endometrium are found during diagnostic evaluation of a 65-year-old woman with uterine bleeding. Which of the following is associated with this condition? (A) Arsenic exposure (B) BRCAmutations (C) Endometriosis (D) Prolonged exposure to estrogens (E) Sexual promiscuity

The answer is E. The illustration shows Hodgkin lymphoma. A prominent Reed-Sternberg cell can be seen. The diagnosis is based entirely on the biopsy findings, and there are no confirmatory laboratory tests. In particular, flow cytometry is not currently useful because present techniques fail to reliably detect the neoplastic cell population.

Examination of a lymph node from the neck of a 26-year-old man reveals total effacement of architecture, and at higher power, the characteristic cell shown below. Which additional studies are needed to confirm the diagnosis? (A) Angiotensin-converting factor (B) Gene rearrangement studies (C) Osteoclastic factor assay (D) Urine for Bence Jones protein (E) No additional studies

The answer is C. The laboratory abnormalities (increased serum calcium, parathyroid hormone, and alkaline phosphatase and decreased serum phosphorus) are diagnostic of primary hyperparathyroidism, a well-known cause of hypercalcemia. One of the consequences of hypercalcemia (regardless of cause) is deposition of calcium salts in previously undamaged organs or tissues, a phenomenon known as metastatic calcification.

During a pre-employment evaluation for an executive position, a 35-year-old man is found to have a serum calcium of 12.4 mg/100 mL (normal 9 to 11), a serum phosphorus (phosphate) of 2.0 mg/100 mL (normal 3.0 to 4.5), a serum alkaline phosphatase of 150 U/L (normal 20 to 70), and a serum parathyroid hormone (PTH) of 800 pg/mL (normal 225 to 650). If untreated, the likely lesion responsible for these findings will result in consequences mediated by which of the following mechanisms? (A) Liquefactive necrosis (B) Apoptosis (C) Metastatic calcification (D) Oxygen toxicity (E) Amyloid deposition

The answer is D. The history is strongly suggestive of idiopathic myxedema. Expected laboratory abnormalities include decreased serum free T4, increased TSH, and increased cholesterol. Also, because hypothyroidism, with secretion of less thyroid hormone, results in less saturation of binding sites on TBG (or increased unbound binding sites), the T3 resin uptake, which is inversely proportional to the number of unbound sites, will be decreased. Note:Total T4 and T3 resin uptake, although falling into clinical disuse, may still appear in examination questions. The most appropriate test today would be TSH (which is greatly elevated in hypothyroidism) and free T4 (which is greatly reduced in hypothyroidism).

During a yearlong training program, a 23-year-old female Air Force officer falls in class rank from first place to last place. She has also noted a lower pitch to her voice and coarsening of her hair, along with an increased tendency toward weight gain, menorrhagia, and increasing intolerance to cold. Which of the following laboratory abnormalities is expected? (A) Increased serum free T4 (B) Increased serum T3 resin uptake (C) Increased saturation of thyroid hormone-binding sites on TBG (D) Increased serum TSH (E) Decreased serum cholesterol

The answer is A. Berry aneurysm of the circle of Willis sometimes leading to subarachnoid hemorrhage is a well-known association of adult polycystic kidney disease. Inheritance is by an autosomal dominant mode of inheritance. Secondary polycythemia, not polycythemia vera, is a complication and is caused by increased secretion of erythropoietin.

Enormously enlarged kidneys similar to the one shown in the figure are found at autopsy in a 65-year-old woman. Which of the following is a well-known association or characteristic of this disease process? (A) Berry aneurysm of the circle of Willis (B) Nephrotic syndrome (C) Polycystic ovaries (D) Polycythemia vera (E) X-linked inheritance

The answer is A. An epidural hematoma is an arterial hemorrhage between the dura and the skull, most often resulting from skull fracture and laceration of the middle meningeal artery. Epidural hematomas are characterized clinically by a short period of consciousness (lucid interval) followed by loss of consciousness and signs of cerebral compression. A subdural hematoma is venous hemorrhage underneath the dura, resulting from laceration of the bridging veins. Subdural hematomas are characterized clinically by gradual signs of cerebral compression occurring hours, days, or weeks after injury. Subarachnoid hemorrhage is commonly associated with rupture of a berry aneurysm in the circle of Willis. A transient ischemic attack is a brief episode of impaired neurologic function caused by a brief disturbance in cerebral circulation.

Following a bar fight, a 22-year-old man is brought unconscious to the emergency department. Several minutes earlier, he had been hit on the head with a heavy iron club and had been briefly unconscious, but had then apparently recovered. One or two minutes later, he had again lost consciousness. Which of the following is the most likely diagnosis? (A) Epidural hematoma (B) Subarachnoid hemorrhage (C) Subdural hematoma (D) Transient ischemic attack (E) Stroke

The answer is B. Pseudomembranous colitis is caused by overgrowth of C. difficile. This organism produces exotoxin that induces necrosis of the superficial mucosa, leading to pseudomembrane formation. The bacteria itself does not invade the mucosa. This condition most often occurs in patients with a history of broad-spectrum antibiotic use, because elimination of normal intestinal flora promotes overgrowth of C. difficile.

For the past week, a 65-year-old woman has been treated for a severe infection with broad-spectrum antibiotics, and she had recovered well. Over the past day, however, she has developed foul-smelling, voluminous, greenish, watery diarrhea, as well as abdominal pain and fever. She is diagnosed with pseudomembranous colitis. Which of the following is the mechanism associated with this condition? (A) Aggregation of bacterial colonies on the lumen, forming pseudomembranes (B) Bacterial release of exotoxin, inducing necrosis of the mucosa (C) Physical invasion of bacteria into the superficial mucosa, leading to pseudomembrane formation (D) Selective killing of C. difficilebacteria by antibiotics (E) Spread of the previous infection to the colon

The answer is E. von Willebrand disease (congenital deficiency of von Willebrand factor [vWF]) is characterized by defective platelet adhesion, resulting in a prolonged bleeding time even though the platelets are qualitatively and quantitatively normal. The APTT is also prolonged because of a secondary deficiency of factor VIII. Factor VIII normally circulates in a complex with vWF and is unstable when vWF is deficient.

In a laboratory exercise, a 23-year old female pathology student was found to have a prolonged bleeding time and a prolonged APPT. Her platelet count was normal. These findings are strongly suggestive of (A) Christmas disease. (B) classic hemophilia. (C) congenital afibrinogenemia. (D) Glanzmann thrombasthenia. (E) von Willebrand disease.

The answer is B. The most frequent causes of mild anemia with hypochromia and microcytosis include iron deficiency anemia, the anemia of chronic disease, and b-thalassemia minor. In the latter, the diagnosis is confirmed by demonstrating increased concentration of hemoglobin A2 and the characteristic CBC findings of marked microcytosis, as evidenced by a very low MCV with only moderate reduction of the hemoglobin and hematocrit. In iron deficiency anemia, serum iron is decreased, TIBC is increased, and storage iron is depleted, as indicated by decreased serum ferritin and absent bone marrow hemosiderin on Prussian blue stain. Additionally, a source of blood loss is often apparent. The anemia of chronic disease is most often normochromic and normocytic, but can be hypochromic and microcytic. In such cases, a decrease in TIBC and the presence of an obvious chronic disease are indicative of the cause of the anemia.

In a pre-employment examination, a 23-year-old woman is found to have laboratory values consistent with mild anemia, and a blood smear is reported as demonstrating hypochromia and microcytosis. Which of the following determinations would be most useful in demonstrating that these findings were manifestations of β-thalassemia minor, as contrasted to other causes of hypochromia and microcytosis? (A) Hemoglobin A1c (B) Hemoglobin A2 (C) Hemoglobin value on routine complete blood count (CBC) (D) Histochemical demonstration of α-chain aggregates (E) Microscopic examination of peripheral blood smear

The answer is D. The illustration shows a tubular adenoma, which is the most common form of adenomatous polyp. These lesions can be single or multiple, or they can occur as components of various multiple polyposis syndromes. Notable among these syndromes are Gardner (associated with osteomas and soft tissue tumors), Turcot (associated with central nervous system tumors), and FAP. All of the foregoing are associated with an increased incidence of colon malignancy. In contrast, the Peutz-Jeghers polyp is a nonneoplastic hamartomatous lesion. Even though the polyp itself does not transform into colon cancer, the Peutz-Jeghers syndrome is associated with an increased incidence of colon cancer and malignancies elsewhere.

In a routine colonoscopy, a 76-year-old man is found to have a lesion similar to that shown in the illustration. The lesion shown is a classic example of which of the following? (A) Hamartoma (B) Invasive adenocarcinoma (C) Peutz-Jeghers polyp (D) Tubular adenoma (E) Villous adenoma

The answer is D. Carbon tetrachloride (CCl4) injury is the classic model of membrane injury incurred by free radical formation. In this instance, CCl4 is converted to the active free radical CCl3 in the smooth endoplasmic reticulum by the P450 system of mixed function oxidases.

In an experimental model, an autopsy study of a rat exposed to toxic doses of carbon tetrachloride (CCl4) revealed fatty change and necrosis of hepatocytes. The mechanism of cell injury exemplified here is (A) activation of apoptosis. (B) acute phase reaction. (C) arrest of cell cycle. (D) free radical injury. (E) thrombosis and ischemia.

The answer is D. Neurons are permanent cells, and the classic teaching has been that neurons do not proliferate during adult life (recent evidence casts some doubt on this concept). Bronchial epithelial cells, gastric mucosal cells, and skin epithelial cells are labile cells. Hepatocytes and renal tubular cells are stable cells. Both labile and stable cells are capable of proliferation and regeneration.

In preparation for a stem cell transplant in a patient with leukemia, the oncologist considers the differential susceptibility of various tissues and cells to high-dose chemotherapy and lethal irradiation. So-called permanent cells are the least vulnerable to injury, but also have the least capacity to regenerate. Which of the following is a permanent cell? (A) Bronchial epithelial cell (B) Gastric mucosal cell (C) Hepatocyte (D) Hippocampal neuron (E) Renal tubular cell

The answer is E. Whipple disease, a systemic illness almost always involving the small intestine, is characterized morphologically by distinctive PAS-positive macrophages within affected organs. On electron microscopy, the PAS-positive material is seen to consist of numerous bacillary forms of the gram-positive actinomycete Tropheryma whippelii. The disorder responds to a number of antibacterial agents, but without therapy the course is usually progressive and fatal.

In the diagnostic workup of a 42-year old man with chronic malabsorption and diarrhea, bacillary forms within periodic acid Schiff (PAS)-positive macrophages in the lamina propria of the small intestinal mucosa were demonstrated by electron microscopy. This finding is characteristic of (A) celiac disease. (B) Crohn disease. (C) disaccharidase deficiency. (D) tropical sprue. (E) Whipple disease.

The answer is B. In 1971, it became widely known that there was a danger of development of clear cell adenocarcinoma of the vagina and cervix in young women with a history of in utero exposure to DES while their mothers were pregnant. DES has not been administered to pregnant women in this country for more than 30 years.

On routine examination, it is discovered that a 35-year-old woman had been exposed in utero to DES administered to her mother, who had had a history of recurrent spontaneous abortion. This history suggests that the patient might be at increased risk of (A) adenomyosis. (B) clear cell adenocarcinoma. (C) lichen sclerosus. (D) sarcoma botryoides. (E) squamous cell carcinoma.

The answer is C. The combination of membranous glomerulonephritis, pleuritis, and Libman-Sacks endocarditis (vegetations on both surfaces of the mitral or tricuspid valves) as well as proliferative splenic arteriolitis is characteristic of SLE. Diffuse interstitial pulmonary fibrosis also occurs in SLE. A variety of antinuclear antibodies (ANAs) are found; the most specific are antibodies to the Sm antigen, antibodies to double-stranded DNA, and antibodies that result in a peripheral rim pattern of nuclear immunofluorescence.

Membranous glomerulonephritis is found at autopsy in a 25-year-old woman who died in renal failure. Other autopsy findings include pleuritis, diffuse interstitial fibrosis of the lungs, concentric rings of collagen surrounding splenic arterioles, and warty vegetations of the mitral and tricuspid valves affecting the surfaces behind the cusps, as well as the surfaces exposed to the forward flow of blood. Which of the following is an expected laboratory finding? (A) Increased titer of antistreptolysin O (ASO) (B) Lymphocytosis (C) Peripheral rim pattern of antinuclear antibody fluorescence (D) Positive blood cultures for Streptococcus viridans (E) Serum antibodies reactive with glomerular and pulmonary alveolar basement membranes

The answer is B. Drugs that cause oxidative stress (e.g., primaquine, sulfa-containing drugs) result in intravascular hemolytic anemia in subjects (most often male) with G6PD deficiency. Current military policy exempts G6PD-deficient persons from the requirement for primaquine chemoprophylaxis or sets up screening criteria for individuals whose G6PD status is unknown. Duffy antigen is a minor red blood cell antigen, the absence of which confers some resistance to malarial infection. Intrinsic factor may be absent in pernicious anemia. PIG-A deficiency results in paroxysmal nocturnal hemoglobinuria. Spectrin is deficient in hereditary spherocytosis.

Military policy dictates that flight personnel in Iraq receive primaquine chemoprophylaxis for Plasmodium vivax malaria on redeploying to a non-malarious area. Several days after beginning such a regimen, a 26-year-old African-American pilot develops anemia, hemoglobinemia, and hemoglobinuria. Special studies will likely reveal an abnormality in which of the following? (A) Duffy antigen (B) G6PD (C) Intrinsic factor (D) PIG-A (E) Spectrin

The answer is E. The absence of both fetal kidneys leads to a decreased volume of amniotic fluid (oligohydramnios) because fetal urine is a major source of amniotic fluid. This leads to a group of secondary abnormalities, often including distorted facies and unilateral pulmonary hypoplasia. This sequence of events is referred to as the Potter progression. The other choices refer to a group of related syndromes often grouped together as the TORCH complex (produced by a group of organisms: Toxoplasma, Other etiologies, Rubella virus, Cytomegalovirus, or Herpes simplex virus; the "Other" designation includes diverse viruses and the treponemal spirochete of syphilis). These infections are associated with a similar group of fetal abnormalities, including microcephaly, brain lesions with focal calcifications, and ocular and cardiac anomalies. Microcephaly is also a common feature of the fetal alcohol syndrome.

Prior to the birth of a stillborn infant, decreased amniotic fluid (oligohydramnios) for gestational age was demonstrated on ultrasound examination. Which of the following autopsy findings of the stillborn is most consistent with this maternal-fetal abnormality? (A) Infection with Toxoplasma gondii, plus microcephaly, hydrocephaly, chorioretinitis, and microphthalmia (B) Infection with rubella virus, along with microcephaly and heart malformations (C) Infection with CMV, plus microcephaly, hepatitis, and intracranial calcifications (D) Microcephaly, facial anomalies, and a number of central nervous system (CNS) anomalies, as well as a history of major maternal alcohol abuse (E) Bilateral renal agenesis and hypoplasia of one lung

The answer is A. Widespread "punched-out" lytic bone lesions in a patient in the older age group are highly suggestive of multiple (plasma cell) myeloma. IgG or IgA M proteins are almost always found in multiple myeloma. Frequent additional laboratory abnormalities include hypercalcemia and urinary excretion of Bence Jones protein (free kappa or lambda monoclonal light chains), red cell Rouleaux formation resulting from hyperglobulinemia, and indicators of renal insufficiency.

Radiographic examination of a 65-year old man with back pain caused by a compression fracture of T12 reveals multiple "punched-out" lytic bone lesions. Which of the following additional abnormalities is likely? (A) A serum IgG kappa M protein (B) Hypocalcemia (C) Increased serum alkaline phosphatase (D) Marked splenomegaly (E) Polyclonal urinary light chains

The answer is E. Parkinson disease, or parkinsonism, is characterized by a resting pillrolling tremor, masked facies, slowness of movements, muscular rigidity, and a festinating (shuffling) gait. Parkinsonism can be idiopathic or due to trauma, drugs, toxins (e.g., methyl-phenyl-tetrahydropyridine), or Shy-Drager syndrome. In the past, a major cause was von Economo encephalitis.

Several years ago, a 60-year-old woman had presented with bradykinesia, rigidity, a resting pill-rolling tremor in her right hand, and "mask-like," expressionless facies. She currently presents with gait problems, taking short, shuffling steps and losing her balance easily. Which of the following is the most likely diagnosis? (A) Alzheimer disease (B) ALS (C) Creutzfeldt-Jakob disease (D) Huntington disease (E) Parkinson disease

The answer is E. The diagnosis is amniotic fluid embolism, which is characterized by sudden peripartal respiratory difficulty, progressing to shock and often to death. Sheehan syndrome is associated with obstetric blood loss, with resultant pituitary ischemia leading to postpartal hypopituitarism. Chorioamnionitis is infection of the placenta, which can have devastating consequences for both the mother and the child. Abruptio placentae is premature separation of the placenta and can lead to antepartal bleeding and fetal death. Amniotic fluid aspiration can occur in the child and can result in difficulties for the infant, particularly if meconium is aspirated. Amniotic fluid embolism and abruptio placentae are well-known causes of DIC, as are retained dead fetus and toxemia.

Soon after an uncomplicated delivery at 38 weeks' gestation, a mother develops respiratory difficulties and uncontrolled vaginal bleeding progressing to shock, multiorgan failure, and death. At autopsy, masses of debris and epithelial squamous cells are apparent in the pulmonary microcirculation. Which of the following was the likely cause of death? (A) Sheehan syndrome (B) Chorioamnionitis (C) Abruptio placentae (D) Amniotic fluid aspiration syndrome (E) Amniotic fluid embolism

The answer is D. The illustration shows hypochromia and microcytosis. Iron deficiency is the most frequent cause of hypochromic microcytic anemia. In infants and preadolescents, iron deficiency is most often nutritional in origin and is particularly common in non-breastfed babies supplemented with cow's milk, rather than iron-fortified formula, within the first 12 months of life. In young women, the cause is most often related to menstrual blood loss compounded by deficient dietary intake. In men and postmenopausal women, the usual cause is occult gastrointestinal blood loss.

The peripheral blood smear of an anemic 1-year-old child is shown in the illustration. The most likely diagnosis is (A) anemia of chronic disease. (B) aplastic anemia. (C) hereditary spherocytosis. (D) iron deficiency anemia. (E) thalassemia major.

The answer is C. The illustration shows predominance of mature-appearing lymphocytes, characteristic of CLL. CLL most often affects older persons, many of whom are asymptomatic for many years. Generalized lymphadenopathy and hepatosplenomegaly are frequent findings.

The peripheral blood smear of an asymptomatic 68-year-old white man exhibiting generalized lymphadenopathy and hepatosplenomegaly is shown in the illustration. Which of the following is the most likely diagnosis? (A) ALL (B) AML (C) CLL (D) CML

The answer is D. Clear cell adenocarcinoma of the vagina is a rare malignant tumor that is markedly increased in incidence in daughters of women who received DES therapy during pregnancy.

Which of the following is a predisposing factor in the pathogenesis of clear cell adenocarcinoma of the vagina? (A) Excess estrogen stimulation (B) Herpes simplex virus infection (C) HPV infection (D) In utero exposure to DES (E) Oral contraceptive therapy

The answer is D. The illustration demonstrates carcinoma in situ of the uterine cervix, which is characterized by disordered epithelial growth manifested by loss of polarity and nuclear hyperchromasia beginning at the basal layer and extending outward. In this illustration, the changes involve the entire epithelial thickness and would thus be classified as CIN grade 3 or carcinoma in situ. Although mild forms can be reversible, the principal significance of CIN is its precursor role in the genesis of invasive cervical carcinoma. Invasion and metastases are not associated with CIN but only with fully developed invasive carcinoma; however, genomic integration of HPV DNA sequences, most often types 16, 18, 31, or 33, is associated with CIN, as well as with frank malignant change.

The cervical lesion shown is similar to that obtained in a cone cervical biopsy from a 28-year-old sexually active woman who had had a "positive" Pap smear. The type of cervical change seen here is often characterized by (A) local invasion. (B) penetration of the basement membrane. (C) lymphatic spread. (D) genomic integration of HPV sequences. (E) hematogenous dissemination.

The answer is C. Cavitation occurs only in secondary tuberculosis. Both primary and secondary tuberculosis are characterized by caseating granulomas, often with Langhans giant cells, which heal by scarring and calcification. The skin test result for tuberculin sensitivity is positive in both forms.

The chest radiograph of a 23-year-old medical student reveals a calcified cavitary pulmonary lesion. The tuberculin test is positive, but sputum smears and cultures are negative for Mycobacterium tuberculosis. A presumptive diagnosis of secondary tuberculosis is made. If further studies, including a biopsy, were performed, which of the following findings would justify the diagnosis of secondary tuberculosis, as contrasted to primary tuberculosis? (A) Calcification (B) Caseating granulomas (C) Cavitation (D) Langhans giant cells (E) Positive tuberculin test result

The answer is B. The lesion shown in the illustrations is a glioblastoma multiforme, the most frequently occurring primary neoplasm of the CNS. Characteristic features include the intracerebral location, prominent hemorrhage and necrosis, and the pseudopalisading appearance of the pleomorphic tumor cells.

The figures shown here are representative of the gross and microscopic appearance of an autopsy specimen from a 55-year-old woman who had a 1-year history of progressive headache and seizures leading to aphasia. The diagnosis is (A) ependymoma. (B) glioblastoma multiforme. (C) medulloblastoma. (D) meningioma. (E) oligodendroglioma.

The answer is E. The observation of thickened glomerular capillary loops apparent on light microscopy permits the diagnosis of membranous glomerulonephritis. This condition is most frequent in young women and is characterized clinically by the nephrotic syndrome and progressive azotemia.

The most likely diagnosis in a 24-yearold woman with the nephrotic syndrome, progressive azotemia, and thickening of glomerular capillary loops apparent on light microscopy is (A) Alport syndrome. (B) diabetic nephropathy. (C) focal segmental glomerulosclerosis. (D) lipoid nephrosis. (E) membranous glomerulonephritis.

The answer is A. The figure illustrates an Aschoff body, the characteristic lesion of rheumatic fever. This myocardial lesion is most often oval in shape and characterized by swollen, fragmented collagen and fibrinoid material and by characteristic large mesenchymal cells (Anitschkow myocytes) and multinucleated cells (Aschoff cells). Sydenham chorea is a major manifestation of rheumatic fever.

The myocardial lesions shown in the figure were observed at the autopsy examination of a pediatric patient who died after a short illness. During life, which of the following manifestations of his illness was most likely? (A) Chorea (B) Systemic embolization (C) Systemic lupus erythematosus (D) Unstable angina (E) Wasting diseases

The answer is C. The history is consistent with ARDS. ARDS is a cause of severe lifethreatening respiratory insufficiency and may be caused by a variety of etiologic agents, among them severe trauma, such as a gunshot wound. The common feature, regardless of etiology, is diffuse alveolar damage.

Three days after being admitted to the hospital for treatment of a gunshot wound, a 29-year-old man suffered the onset of acute respiratory distress, and diffuse bilateral infiltrates were seen in both lung fields on chest X-ray. A lung biopsy revealed the presence of intra-alveolar edema, along with hyaline membrane formation. These findings are indicative of (A) bacterial pneumonia. (B) viral pneumonia. (C) diffuse alveolar damage. (D) pulmonary hypertension. (E) left-sided heart failure.

The answer is B. The combination of hematuria (with red cell casts), oliguria, azotemia, and hypertension constitutes the nephritic syndrome, the prototype of which is poststreptococcal glomerulonephritis. An indicator of the prior streptococcal infection is an increased titer of antistreptolysin O. Fluid retention is usually minimal, often limited to periorbital edema, and is the result of reduced renal excretion of salt and water, not hypoalbuminemia. Hypertension, not hypotension, is expected. This disorder is an immune complex disorder, and infection of the kidney does not occur.

Two weeks after recovery from a severe bout of pharyngitis, an 11-year-old girl is seen because of the acute onset of periorbital edema, hematuria, malaise, nausea, and headache. Which of the following findings is expected? (A) Hypotension (B) Increased antistreptolysin O titer (C) Marked hypoalbuminemia (D) Polyuria (E) Positive urine cultures for β-hemolytic streptococci

The answer is C. Peptic ulcer of the duodenum is not a precursor lesion to carcinoma. The risk of malignant transformation in familial multiple polyposis approaches 100%. Colorectal villous adenomas undergo malignant change in about 30% of cases. There is a markedly increased incidence of colon cancer in long-standing cases of ulcerative colitis. The incidence of colon cancer is also increased in Crohn disease, but to a lesser degree than in ulcerative colitis.

Upper and lower gastrointestinal endoscopic examinations are performed on a 45-year-old man. A lesion is found and the patient is told that the lesion has NO malignant potential. Of the following choices, which is the lesion that was most likely found? (A) Colorectal villous adenoma (B) Crohn disease (C) Duodenal peptic ulcer (D) Familial multiple polyposis (E) Ulcerative colitis

The answer is C. A mosaic pattern of bone caused by increases in both osteoblastic and osteoclastic activity is characteristic of Paget disease of bone (osteitis deformans). Serum alkaline phosphatase is markedly increased. Hearing loss is common (from narrowing of the auditory foramen and compression of the eighth cranial nerve), and an increase in hat size due to frontal bossing is often noted.

When ordering academic attire for a recent graduation, a 65-year-old university professor is surprised to find that his hat size has increased. Shortly thereafter, in a routine checkup, serum alkaline phosphatase activity is found to be markedly elevated. Serum calcium and phosphorus are normal. Examination reveals enlargement of the skull with frontal bossing and enlarged maxilla, and hearing loss is evident. Which of the following abnormalities is associated with the bone disorder suggested by these findings? (A) "Brown tumor" (B) Defective calcification of osteoid matrix (C) Mosaic pattern of bone (D) Polyostotic fibrous dysplasia with severe deformity (E) Subperiosteal hemorrhage and osteoporosis

The answer is D. The combination of increased serum calcium and alkaline phosphatase along with decreased serum phosphorus is most consistent with primary hyperparathyroidism. The most frequent cause of this endocrine abnormality is a parathyroid adenoma. Decreased phosphorus would not be an expected finding in metastatic carcinoma. The normal serum proteins mitigate against multiple myeloma and sarcoidosis. Additionally, the alkaline phosphatase is usually normal in multiple myeloma. Hypercalcemia from increased intake of calcium (as in the milk-alkali syndrome) is usually unaccompanied by significant changes in phosphorus or alkaline phosphatase.

While being investigated for long-standing hypertension, a 55-year-old woman is found to have the following serum laboratory test values: normal creatinine, total protein, albumin, and globulin; increased calcium and alkaline phosphatase; and decreased phosphorus. These findings suggest the presence of (A) carcinoma metastatic to bone. (B) excessive dietary calcium intake. (C) multiple myeloma. (D) parathyroid adenoma. (E) sarcoidosis.

The answer is E. Hepatitis E is a common sporadic cause of viral hepatitis in India. It has close to 20% mortality in pregnant women. Like hepatitis A, it is transmitted enterically. Hepatitis C can be transmitted parenterally, as can hepatitis B and hepatitis D. Hepatitis D requires coinfection with hepatitis B.

While on an international medical rotation, you encounter a pregnant woman in a rural village in India who presents with fever, jaundice, and malaise. The patient unexpectedly expires. This is the second case this month with a similar presentation. Which of the following is the most likely form of hepatitis? (A) Hepatitis A (B) Hepatitis B (C) Hepatitis C (D) Hepatitis D (E) Hepatitis E

The answer is A. By 24 hours well-developed microscopic changes of coagulative necrosis can be detected in infarcted tissue. There is loss of nuclei in cells and infiltration of neutrophils into tissue.

Yesterday, a 60-year-old man presented to the emergency department with dyspnea, diaphoresis, and crushing substernal chest pain that radiated to his neck and left arm. When asked to describe the pain, he put his fist to the center of his chest and stated that it felt "as if someone is squeezing my heart." An electrocardiogram demonstrated changes consistent with myocardial infarction, and serum troponin I levels were elevated. If the patient unexpectedly dies today, which of the following would almost certainly be found on histologic examination of the affected myocardium? (A) Coagulative necrosis with neutrophil infiltration (B) Fibrotic tissue replacing infarcted tissue (C) No histologic changes (D) Slight swelling of tissue and change of color (E) Young fibroblasts and new vessels growing into the infarcted tissue

The answer is A. The diagnosis is X-linked agammaglobulinemia of Bruton. Failure of maturation of pre-B cells is associated with absence of mature B lymphocytes and plasma cells; failure of antibody synthesis; marked serum hypogammaglobulinemia; and recurrent bacterial infections, especially sinopulmonary infections. Histologic examination of lymphoid tissue reveals marked underdevelopment of germinal centers. T cells are unaffected, as are T-cell functions such as cell-mediated immunity and resistance to most viral infections. The disease is X-linked and is due to mutations in the B cell tyrosine kinase (Btk) gene.

A 1-year-old boy has had repeated pyogenic infections with streptococci, staphylococci, and Haemophilusfor the past 6 months. Tests for T-cell function, granulocyte function, and complement activity have all been unaffected. Serum IgG is 50 mg/dL (normal 500 mg/dL). Flow cytometry revealed absent expression of heavy-chain µon blood lymphocytes. T lymphocytes were slightly increased in number, with a normal CD4+to CD8+ ratio. Expected findings on examination of a lymph node biopsy from this patient include which of the following? (A) Absent germinal centers (B) Follicular hyperplasia with exuberant proliferation of immature B cells (C) Massive TH1 cell infiltration into lymphoid follicles (D) Normal lymphoid tissue development (E) Plasma cell hyperplasia

The answer is A. The diagnosis is cystic fibrosis, the most common lethal genetic disease in Caucasian populations. The disorder is due to a defect in the cystic fibrosis transmembrane conductance regulator protein, and about 70% of cases have a deletion of phenylalanine in position 508 (DF508 mutation). Affected patients often have multiple pulmonary infections and pancreatic insufficiency with steatorrhea and failure to thrive. Death is often due to respiratory failure secondary to repeated pulmonary infections, facilitated by the buildup of thick, tenacious mucus in the airways. Increased concentration of chloride in sweat and tears is characteristic, and the sweat test is an important diagnostic adjunct.

A 1-year-old female infant is hospitalized for pneumonia. Bacterial cultures of the sputum have grown Pseudomonas aeruginosa. She has had two prior hospitalizations for severe respiratory infections. Her mother has noted that when she kisses her child, the child tastes "salty." The child has had weight loss that the mother attributes to frequent vomiting and diarrhea with bulky, foul-smelling fatty stools. The child is small for her age. Which of the following critical proteins is altered in this condition? (A) Cystic fibrosis transmembrane conductance regulator (B) Dystrophin (C) α-1,4-Glucosidase (D) α-L-Iduronidase (E) Lysyl hydroxylase

The answer is E. Rejection of the stem cell transplant is occurring, as evidenced by reappearance of the marker for the patient's original gene. Small nucleotide polymorphisms are the most frequent form of DNA variation. As the name implies, they are typically small in size, often a single nucleotide. They may occur in any portion of the gene, even in intergenic regions of the genome, and are of increasing importance as genetic markers, as illustrated by the example presented here.

A 1-year-old girl with an inborn error of metabolism resulting in a lysosomal storage disease receives a hematopoietic stem cell transplant intended to replace her macrophage population. The gene of interest has a "marker" small nucleotide polymorphism within a noncoding intron of the affected gene in which an A (patient gene) is substituted for a G (donor gene). She does quite well for the first 3 weeks. She tests positive for the missing enzyme, her previously abnormally enlarged organs begin to diminish in size, and assay of peripheral blood lymphocytes reveals increasing numbers of cells with the G polymorphism. However, the attending physicians are now concerned because repeated genetic testing reveals a progressive increase in lymphocytes with the A polymorphism. Which of the following is the best explanation for this finding? (A) Generalized immune complex formation (B) Graft-versus-host disease (C) Immune paralysis (D) Immune tolerance (E) Rejection of the stem cell transplant

The answer is C. Although most patients with Turner syndrome have a 45,XO karyotype, the syndrome is thought to be caused by the absence of one set of genes from the short arm of one X chromosome, and a variety of chromosome abnormalities may be found. Many patients are mosaics (e.g., 45,XO/46,XX or 45,XO/47,XXX), and the phenotype is highly variable. A deletion of the SHOXgene can cause an identical phenotype and may be considered to be a variant of Turner syndrome.

A 14-year-old girl with amenorrhea is concerned because of the delayed onset of menses. She has shortened stature and a wide, webbed neck; broad chest; and secondary sexual characteristics consistent with those of a much younger girl. Which of the following chromosomal changes is most consistent with these findings? (A) 5p- (B) 22q11- (C) 45,XO (D) 46,XY (E) 47,XXY

The answer is A. The clinical presentation is that of the cri du chat (or 5p-) syndrome.

A 2-year-old child has been followed for mental retardation and slow development, as well as multiple birth defects. The child has a high-pitched catlike cry. On examination, microcephaly, hypertelorism, micrognathia, epicanthal folds, low-set ears, and hypotonia are noted. Karyotypic analysis would be expected to show (A) 5p-. (B) 22q11-. (C) 45,XO. (D) 46,XY. (E) 47,XXY.

The answer is A. Yellowing of the sclerae, skin, and oral mucosa are all characteristic of jaundice, the accumulation of bilirubin, the catabolic product of the heme moiety of hemoglobin. Jaundice can occur by diverse mechanisms: hemolytic (see Chapter 11), hepatocellular (see Chapter 16), or obstructive (see Chapter 16).

A 20-year-old man presents with yellowing of the sclerae, skin, and oral mucosa. Which of the following accumulations underlies these findings? (A) Bilirubin (B) Hemosiderin (C) Lead (D) Melanin (E) Silver

The answer is E. The clinical findings are typical of acute appendicitis, another example of severe acute inflammation. Because the danger of perforation is great, early appendectomy is the treatment of choice. Suppurative or purulent inflammation is characterized by the prominent areas of edema resulting from increased vascular permeability, congestion, and a purulent (pus-containing) exudate consisting of necrotic cells and large numbers of neutrophils. In addition, other signs of acute inflammation, such as congestion, are prominent. The patient responds with the sensation of pain (induced by increased hydrostatic pressure in tissue and by chemical mediators such as bradykinin) and the acute phase reaction (in this instance, fever and neutrophilic leukocytosis with a "shift to the left").

A 16-year-old boy presents with a 24-hour history of severe abdominal pain, nausea, vomiting, and low-grade fever. The pain is initially periumbilical in location but has migrated to the right lower quadrant of the abdomen, with maximal tenderness elicited at a site one-third of the way between the crest of the ileum and the umbilicus (McBurney point). The leukocyte count is 14,000/mm3, with 74% segmented neutrophils and 12% bands. Surgery is performed. Which of the following describes the expected findings at the affected site? (A) Fistula (abnormal duct or passage) connecting to the abdominal wall (B) Granulation tissue (new vessels and young fibroblasts) with a prominent infiltrate of eosinophils (c) Granulomatous inflammation with prominent aggregates of epithelioid cells and multinucleated giant cells (d) Massive infiltration of lymphocytes and plasma cells (e) Prominent areas of edema, congestion, and a purulent reaction with localized areas of abscess formation

The answer is A. The patient has renal agenesis, absence of the kidney due to failure of organ development. The congenital lack of one kidney differs from atrophy, in which a decrease in the size of an organ results from a decrease in the mass of preexisting cells. Unilateral renal agenesis is usually a harmless malformation, and the opposite kidney is often enlarged due to compensatory hypertrophy. Bilateral renal agenesis is incompatible with life and is of special interest since it can lead to the Potter progression (see Chapter 17).

A 16-year-old girl undergoes radiologic imaging of her abdomen and is found to have only one kidney. She had been entirely unaware of this problem. Which of the following terms is most descriptive of this finding? (A) Agenesis (B) Atrophy (C) Hyperplasia (D) Hypoplasia (E) Metaplasia

The answer is D. Marfan syndrome, an autosomal dominant disorder caused by mutations of the fibrillin gene on chromosome 15, is a frequent cause of ectopia lentis. Other cardinal features include skeletal and cardiovascular abnormalities. Patients are tall and thin, with notably long limbs and digits. An anterior chest deformity known as pectus excavatum is sometimes seen, and vertebral abnormalities include scoliosis and lordosis. In addition, a highly arched palate and crowding of the teeth may occur. Cardiovascular complications include mitral valve prolapse and mitral regurgitation. Cystic medial necrosis can lead to dilation of the aortic root and aortic regurgitation. Life-threatening complications are aortic aneurysm and aortic dissection.

A 19-year-old college sophomore is referred by his ophthalmologist because of the finding of ectopia lentis (dislocation of the lens), which has resulted in visual difficulties that have interfered with his performance on the varsity basketball team. The patient is very tall, with long limbs and long, slender, spiderlike fingers. His chest has a "caved-in" appearance, and he also has a modest degree of scoliosis. A midsystolic "click" is heard, and an echocardiogram reveals mitral valve prolapse. The most likely diagnosis is (A) Ehlers-Danlos syndrome. (B) Fabry disease. (C) Hurler syndrome. (D) Marfan syndrome. (E) Pompe disease.

The answer is A. Either of two coreceptors, CCR5 or CXCR4, is involved in the initial binding of the virus to the cell surface molecule gp120 on CD4+cells. It is of considerable interest that certain mutations in the CCR5 gene are associated with what appears to be total resistance to infection with some common strains of HIV. Homozygotes are totally resistant, and heterozygotes develop a more slowly progressive disease.

A 19-year-old intravenous drug user has regularly sought human immunodeficiency virus (HIV) testing, always with negative results. He admits to carelessly sharing needles on multiple occasions with individuals later found to be HIV-positive. He has heard that there is an inherited genetic basis for some people to be relatively "immune" to HIV infection. The genetic change that he is referring to is a mutation in a gene coding for which of the following proteins? (A) CCR5 (B) CD4 (C) gp120 (D) gp41 (E) Reverse transcriptase

The answer is C. Wiskott-Aldrich syndrome is an X-linked disorder characterized by eczema, thrombocytopenia, recurrent infections, and poor antibody response to polysaccharide antigens. Bloody diarrhea is also common. Death before 6 years of age occurs frequently and is most often due to bleeding, infection, or malignancy (most often lymphoma).

A 2-year-old boy has eczema and thrombocytopenia. There is also a history of recurrent infection. His brother has similar abnormalities, but none of his three sisters is affected. Patients with this disorder are known to have impaired antibody response to which of the following types of antigen? (A) Deoxyribonucleic acid (B) Phospholipid (C) Polysaccharide (D) Ribonucleic acid (E) Steroid

The answer is B. Chronic granulomatous disease of childhood, a condition characterized by repeated infections and most commonly X-linked inheritance, is marked by failure of the myeloperoxidase-halide system of killing within phagocytic cells. It is caused by the deficiency of NADPH oxidase activity. This results in a secondary deficiency of reactive oxygen metabolites, including H2O2, which, along with halide ions, functions as a substrate for myeloperoxidase. A hallmark of the disorder is the failure of intracellular killing of catalase-positive organisms, exemplified by staphylococci. These organisms are ingested but not killed. The impaired phagocytic cell is incapable of producing H2O2, and any H2O2 produced by the microorganism itself is inactivated by endogenous catalase. In contrast, catalase-negative microorganisms, such as streptococci, are ingested and killed. They too produce endogenous H2O2, which is thus available as one of the substrates for myeloperoxidase. In a sense, the microorganisms assist in their own killing.

A 2-year-old boy presents with recurrent infections involving multiple organ systems. Extensive investigation results in a diagnosis of chronic granulomatous disease of childhood. Which of the following most closely characterizes the abnormality in this patient's phagocytic cells? (A) Decreased killing of microorganisms because of enhanced production of hydrogen peroxide (B) Deficiency of NADPH oxidase activity (c) Impaired chemotaxis and migration caused by abnormal microtubule formation (d) Inability to kill streptococci (e) Increased myeloperoxidase-halidemediated killing of catalase-positive organisms when compared with catalase-negative organisms

The answer is C. Theoretically, a person who carries a robertsonian translocation with chromosome 21 and a second acrocentric chromosome has a 1 in 3 chance of having a child with trisomy 21; however, the risk of a live birth of a child with Down syndrome is actually much less, presumably because of a high incidence of spontaneous abortion of such fetuses. The important point is that a robertsonian translocation predisposes to a hereditable form of Down syndrome. The risk is not related to maternal age and is much higher than the risk in the general population, which is 1 in 1500 for women 20 years of age and younger, increasing to 1 in 25 in women older than 45 years of age.

A 20-year-old woman has a robertsonian translocation involving chromosome 21 and a second acrocentric chromosome. What is the theoretic likelihood of a functional trisomy 21 if one of her ova is fertilized by a normal sperm? (A) 1 in 1 (B) 1 in 2 (C) 1 in 3 (D) 1 in 4 (E) 1 in 1500

The answer is C. The diagnosis is systemic lupus erythematosus, and the most characteristic lesions are mediated by immune complex deposition (type III hypersensitivity). In this form of hypersensitivity, antibody combines with antigen, resulting in antigen-antibody complexes. Insoluble aggregates of immune complex are deposited in vessel walls, serosal surfaces, and other extravascular sites, and complement is bound. The antigen-antibody-complement complexes are highly chemotactic for neutrophils, which release lysosomal enzymes and other mediators of tissue damage (prostaglandins, kinins, and free radicals).

A 20-year-old woman presents with malar rash, arthralgias, low-grade fever, and high titer antibodies to double-stranded DNA and to the Sm (Smith) antigen. Which of the following forms of hypersensitivity is the primary mechanism of the abnormalities found in this disorder? (A) Type I (immediate or anaphylactic) hypersensitivity (B) Type II (antibody-mediated or cytotoxic) hypersensitivity (C) Type III (immune complex-mediated disorders) hypersensitivity (D) Type IV (cell-mediated) hypersensitivity

The answer is B. Graft-versus-host disease is most often manifested by clinical findings related to the three principal target organs: the skin, liver, and gastrointestinal tract. The skin manifestations are often initiated by a pruritic rash. Elevation of bilirubin and liver enzymes signals the hepatic involvement. Effects on the ileum and colon present as diarrhea and abdominal pain. The lesions are caused by donor lymphocytes, with targeting of host epithelial cells by CD8+T cells.

A 22-year-old woman with acute myeloblastic leukemia receives an allogeneic bone marrow transplant with apparent successful engraftment. Three weeks later, early jaundice, as well as a generalized maculopapular rash, is noted. Profuse diarrhea follows. A skin biopsy reveals vacuolar changes, necrotic epidermal cells, and a lymphocytic infiltrate. These findings are most likely caused by (A) antibody-dependent cellular cytotoxicity. (B) attack on host epithelial cells by donor CD8+T cells. (C) contamination of the donor transplant cells with hepatitis C virus. (D) IgE and mast cell-mediated anaphylactic hypersensitivity. (E) secretion of IL-2 and interferon-γ by TH1 cells.

The answer is C. The patient has fat embolism syndrome, which is characterized by pulmonary distress, cutaneous petechiae, and various neurologic manifestations. Fat embolism is a well-known complication of fractures of long bones, such as the femur, and other bones with abundant fatty marrow. On fracture, marrow fat can enter the circulation, and small fat droplets can lodge in the vessels of the skin, lung, and microvasculature of the brain, resulting in the clinical manifestations of this disorder.

A 23-year-old man undergoes surgery for fractures of the pelvis and left femur resulting from a high-speed motor vehicle accident. The following day he develops dyspnea, speech difficulties, and a petechial skin rash. Which of the following types of embolism is the likely cause of these findings? (A) Air (B) Amniotic fluid (C) Fat (D) Paradoxical (e) Thrombotic

The answer is A. Isolated IgA deficiency is most often asymptomatic but can be characterized by anaphylactic reactions to transfused blood. It is critical to notify the transfusion service about a patient with possible IgA deficiency prior to giving blood products so that products can be washed to remove any immunoglobulins prior to administration. This disorder can also be associated with frequent episodes of diarrhea and recurrent infections, especially those involving mucosal surfaces. This inherited B-cell defect is due to inability of IgA B cells to mature into IgA-producing plasma cells. Interestingly, the defect leading to systemic anaphylaxis involves both IgA and IgE antibody formation. Patients lacking IgA can develop IgE antibodies against the IgA antibodies present in transfused blood. This sensitization can result in susceptibility to anaphylaxis on subsequent transfusion.

A 24-year-old woman who had previously been uneventfully transfused receives a blood transfusion during surgery and shortly thereafter develops itching, generalized urticaria, laryngeal edema, and dyspnea with wheezing respiration. She has a past history of recurrent upper respiratory tract infections and frequent episodes of diarrhea. Laboratory studies are most likely to reveal decreased concentrations of which of the following immunoglobulins? (A) IgA (B) IgD (C) IgE (D) IgG (E) IgM

The answer is A. A leiomyoma (fibroid) is a benign tumor of the smooth muscle of the uterus and thus is an example of a benign tumor of mesenchymal origin. Profuse, painful menses and infertility are major complications of this most common tumor of the female genital tract. A leiomyosarcoma is the malignant counterpart. Malignant tumors of epithelial cells are carcinomas, and these are known as adenocarcinomas if they involve glandular epithelium. Benign tumors of surface epithelium are termed papillomas.

A 24-year-old woman with a history of heavy and painful menstrual periods has been having difficulty conceiving despite months of trying to become pregnant. Further workup includes a bimanual pelvic examination and an ultrasound, which demonstrates a mass in the uterus that is presumed to be a leiomyoma. This mass is a (A) benign tumor of mesenchymal tissue. (B) benign tumor of surface epithelium. (C) malignant tumor of epithelial tissue. (D) malignant tumor of glandular epithelium. (E) malignant tumor of mesenchymal tissue.

The answer is B. Acute cellular rejection is characterized by an infiltrate of both CD4+ and CD8+ lymphocytes. Acute rejection occurs over a variable time period, ordinarily days to weeks to months after transplant.

A 25-year-old woman with membranous glomerulonephritis receives a kidney transplant. The donor is her HLA-matched sister. She does well initially, but after several weeks, there is a progressive increase in serum creatinine. Assuming that this represents acute cellular rejection, an infiltrate with which of the following types of inflammatory cells is most likely to be a prominent finding on renal biopsy? (A) Eosinophils (B) Lymphocytes (C) Mast cells (D) Monocytes-macrophages (E) Neutrophils

The answer is D. The histologic hallmark of sarcoidosis is the finding of noncaseating granulomatous inflammation. Although this finding is not entirely specific, a non-necrotizing granulomatous response of the lung is rarely seen in patients with tuberculosis or deep-seated fungal infections. These infections usually have a necrotizing component.

A 26-year-old African-American woman has bilateral hilar adenopathy, and radiography reveals multiple reticular densities in both lung fields. A bronchoscopic biopsy reveals granulomatous inflammation with multiple giant cells of the Langhans type and no evidence of caseous necrosis. Which of the following is the most likely diagnosis? (A) Aspergillosis (B) Coccidioidomycosis (c) Histoplasmosis (d) Sarcoidosis (e) Tuberculosis

The answer is E. The history is typical of amniotic fluid embolism, one of the major obstetric causes of disseminated intravascular coagulation (DIC). Other obstetric complications associated with DIC include retained dead fetus and abruptio placentae (premature separation of the placenta). Nonobstetric causes include neoplasms or tissue damage from infection, immunologic mechanisms, or trauma. Neoplastic causes include tumors of the lung, pancreas, prostate, and stomach, and FAB M3 acute myeloblastic (promyelocytic hypergranular) leukemia. Tissue damage can result from trauma such as lung surgery, from hemolysis or hemolytic transfusion reactions, and from inflammatory causes, such as gram-negative sepsis and immune complex disease.

A 26-year-old woman dies after a short illness beginning in the late stages of labor. At autopsy, blood vessels in the lungs contained fetal debris (e.g., squamous cells, vernix, mucin), as did other vessels of multiple organs. Review of the clinical history reveals that she had become acutely ill with dyspnea, hypotension, and seizures, and a chest radiograph had demonstrated evidence of pulmonary edema. This was all followed by prolonged hemorrhage from the vagina and generalized bleeding from multiple other sites. The changes that were found within multiple blood vessels most likely are (A) bone marrow emboli. (B) fat emboli. (C) gas emboli. (D) septic emboli. (e) widespread thrombosis.

The answer is B. Punctate ulcers associated with extensive burn injuries are known as Curling ulcers. A similar phenomenon occurs in patients with head trauma, in which the lesions are known as Cushing ulcers. Aplastic anemia can result from an idiosyncratic reaction in patients taking the antibiotic chloramphenicol. Likewise, sulfonamides can cause a necrotizing eruption around mucous membranes in some individuals. Reye syndrome is associated with extensive microvesicular fatty change of the liver in children taking aspirin during an acute viral illness. The Stevens-Johnson syndrome is characterized by erosions and crusts of the lips and oral mucosa as a component of an extensive form of erythema multiforme, a maculopapular, vesiculobullous eruption often related to drugs (such as sulfonamides), neoplasia, or connective tissue disorders.

A 27-year-old man who was badly burned in an industrial accident requires multiple skin grafting. While in the intensive care unit, he is found to have blood in his stools, and endoscopy confirms the presence of many small ulcers in his stomach. This complication is referred to as (A) aplastic anemia. (B) Curling ulcer. (C) Cushing ulcer. (D) Reye syndrome. (E) Stevens-Johnson syndrome.

The answer is E. Li-Fraumeni syndrome is a hereditary syndrome characterized by sarcomas, breast cancer, leukemia, and brain tumors. Xeroderma pigmentosum results from a defect in repair of UV damage. Numerous transmembrane tyrosine kinases, such as Her-2/neu, are implicated in numerous cancers. C-ablcodes for a cytoplasmic tyrosine kinase that forms a fusion protein with bcr in chronic myelogenous leukemia. NF-1, mutated in neurofibromatosis, normally activates the GTPase activity of the gene product of the Rasoncogene. The gene p53, the "guardian of the genome," arrests the cell cycle in G1 in the event that DNA damage is detected.

A 27-year-old woman has recently been diagnosed with a glioma (a malignant brain tumor). Further family history reveals that her 4-year-old son has been diagnosed with leukemia and has been undergoing chemotherapy. In addition, the patient's mother died at 36 years of age due to metastatic breast cancer. Li-Fraumeni syndrome is suspected, given the familial clustering of this group of malignancies. The gene mutated in Li-Fraumeni syndrome normally functions in what capacity? (A) Activates the GTPase activity of the gene product of the Rasoncogene (B) Excises ultraviolet light-induced thymidine dimers (C) Functions as a cytoplasmic tyrosine kinase (D) Functions as a transmembrane tyrosine kinase (E) Halts the cell cycle if DNA damage is detected

The answer is A. The combination of a prolonged activated partial thromboplastin time (APTT), a positive VDRL test for syphilis, recurrent thromboses (arterial or venous), and spontaneous abortion is highly suggestive of the antiphospholipid antibody syndrome. As the name implies, antibodies directed at phospholipids are a characteristic finding. Because of the prolonged APTT and frequent association with systemic lupus erythematosus (SLE), the antibody has been referred to as the "lupus anticoagulant," a misleading term because affected subjects have a thrombotic rather than hemorrhagic diathesis and not all subjects have SLE. The term primary antiphospholipid antibody syndrome is used when there is no evident underlying disease. It should be contrasted to secondary antiphospholipid antibody syndrome, in which the patient has a well-defined autoimmune disorder such as SLE.

A 28-year-old woman is evaluated for possible thrombophilia since she has had two episodes of deep vein thrombosis, as well as two pregnancies that terminated in spontaneous abortion. The activated partial thromboplastin time is prolonged, and she has a positive VDRL screening test for syphilis. This combination of findings is most suggestive of (A) antiphospholipid antibody syndrome. (B) disseminated intravascular coagulation. (C) factor V Leiden. (D) methylene tetrahydrofolate reductase mutation. (e) prothrombin 20210A transition.

The answer is C. T helper (TH1) cells secrete IL-2 and interferon-γ, which in turn facilitate T-cell expansion and macrophage activation.

A 28-year-old woman is found to have pulmonary sarcoidosis. Flow cytometric analysis of T cells isolated from the alveoli and lung interstitium reveals the presence of large numbers of T helper (TH1) cells. These cells are known to secrete which of the following substances? (A) Complement component C5A (B) Elastase and lysyl-hydroxylase (C) Interleukin-2 (IL-2) and interferon-γ (D) IL-8 and transforming growth factor-β (E) Leukotrienes C4and C5

The answer is A. Caseous necrosis occurs as part of granulomatous inflammation, typified by the lesions of tuberculosis.

A 29-year-old man hospitalized for acquired immunodeficiency syndrome (AIDS) is found to have pulmonary tuberculosis. Which type of necrosis is found in the granulomatous lesions (clusters of modified macrophages) characteristic of this increasingly frequent complication of AIDS? (A) Caseous (B) Coagulative (C) Enzymatic (D) Fibrinoid (E) Liquefactive

The answer is C. This is a typical presentation of lead poisoning. Lead inhibits hemoglobin synthesis by inhibiting both aminolevulinic acid dehydratase and ferroketolase. This inhibits not only the synthesis of hemoglobin but also the incorporation of iron into the molecule. Cyanide and carbon monoxide (CO) inhibit cytochrome oxidase. CO also binds to hemoglobin, preventing adequate oxygen transport. Iron deficiency can lead to similar hematologic findings.

A 3-year-old child is brought to the emergency department because of a week-long history of abdominal discomfort, irritability, and weakness. A complete blood count and blood smear demonstrate microcytic hypochromic anemia with basophilic stippling of the red blood cells. Further history reveals that the family lives in an old apartment complex. The most likely cause of the hematologic findings is (A) binding of carbon monoxide to hemoglobin. (B) inhibition of cytochrome oxidase. (C) inhibition of hemoglobin production. (D) iron deficiency. (E) iron intoxication.

The answer is D. A laceration, especially of internal organs, is common in cases of blunt trauma, as in a motor vehicle accident. By definition, a laceration is a jagged tear, often with stretching of the underlying tissue. An incision is a clean cut by a sharp object, as in a clean cut by a surgeon. A contusion is a bruise caused by disruption of underlying blood vessels. A puncture is a deep tubular wound produced by a sharp, thin object. An abrasion is a superficial tearing away of epidermal cells.

A 32-year-old man is involved in a highspeed motor vehicle accident and brought by ambulance to a Level I trauma center because of the severity of his injuries. On arrival, it is determined that he is bleeding internally, and he is taken immediately to surgery, where the surgeons find that the source of his bleeding is a severe liver laceration. A laceration is a (A) bruise caused by disruption of underlying blood vessels. (B) clean cut by a sharp object. (C) deep tubular wound produced by a sharp, thin object. (D) jagged tear, often with stretching of the underlying tissue. (E) superficial tearing away of epidermal cells.

The answer is B. "Heart failure cells" are intra-alveolar hemosiderin-laden macrophages and are indicative of marked chronic passive congestion of the lung. Red cells leak from congested alveolar capillaries into the alveoli, where they are engulfed and degraded by macrophages.

A 36-year-old man dies during cardiac surgery. He had a history of long-standing rheumatic heart disease with mitral stenosis. At autopsy, the pathologist reports findings consistent with mitral stenosis and noted the presence of "heart failure cells." This finding results from (A) activation of the coagulation cascade. (B) chronic passive congestion of the lungs. (C) hypoxic myocardial injury. (D) myocardial hyperemia.

The answer is B. Gastrointestinal mucosal cells are examples of labile cells, and thus are among the most radiosensitive of the tissues or organs listed in the question. The symptoms associated with damage to these cells (nausea, diarrhea, and malabsorption) are likely to limit the total dose of radiation the patient can receive. The other tissues listed are relatively radioresistant and are unlikely to limit the amount of radiation a patient can receive.

A 38-year-old woman is receiving radiation therapy to her abdomen as adjuvant therapy for the treatment of cervical cancer. The radiosensitivity of organs or tissues within the treatment field is a limiting factor in determining the dose of radiation that can be administered. Which of the following is most susceptible to radiation damage? (A) Bone (B) Gastrointestinal mucosa (C) Peripheral nervous tissue (D) Renal parenchyma (E) Skeletal musculature of the abdomen

The answer is A. Vitamin D is required for calcification of osteoid matrix, and defective calcification leads to bowing of the legs and the abnormalities in the costochondral junctions known as the rachitic rosary. Mutations in genes involved in the synthesis of type I collagen lead to osteogenesis imperfecta, and defects in fibrillin lead to Marfan syndrome. Defects in hydroxylation of proline residues occur both in the Ehlers-Danlos syndrome and in vitamin C deficiency, leading to defects in the synthesis of osteoid matrix.

A 4-year-old Inuit child from northern Alaska is brought to the pediatrician because of concern about progressive bowing of the legs and enlargement of the costochondral junctions (rachitic rosary). The underlying defect in this disorder is a defect in (A) calcification of osteoid matrix. (B) fibrillin. (C) formation of osteoid matrix. (D) hydroxylation of proline residues in collagen. (E) type I collagen.

The answer is D. First- and second-degree burns are both partial-thickness burns. Because the findings in this patient reveal both epithelial and dermal involvement, the burns are, by definition, second-degree in type. Both third- and fourth-degree burns are classified as full-thickness burns; they entail total destruction of dermis and epidermis along with underlying skin appendages that normally serve as a source of cells for regeneration. As such, these wounds require skin grafting.

A 4-year-old girl is brought by her mother to the emergency department after the girl was "placed in a hot bath." The patient appears to have extensive blistering of the thighs and buttocks, with slight damage to the underlying dermis. Her burns are best described as (A) first-degree burns. (B) fourth-degree burns. (C) full-thickness burns. (D) second-degree burns. (E) third-degree burns.

The answer is D. The clinical findings described in the question are typical of right-sided heart failure, as are the illustrations, which reveal the nutmeg-like appearance of hepatic chronic passive congestion. The gross morphologic appearance is caused by congested centrilobular areas alternating with pale portal areas.

A 40-year-old woman dies after a long history of an illness characterized by dyspnea, orthopnea, hepatomegaly, distended neck veins, and peripheral edema. The cut surface of the liver as it appears at autopsy is shown in the first panel. The second panel shows the microscopic appearance of the liver. Which of the following disorders is the most likely cause of these findings? (A) Chronic alcoholism (B) Diabetes mellitus (C) Niemann-Pick disease (D) Right-sided heart failure (e) Viral hepatitis

The answer is E. Liquefactive necrosis is characteristic of ischemic injury in the CNS and suppurative infections that cause abscess formation (see Chapter 2). The changes in the cerebrospinal spinal fluid characteristic of bacterial meningitis are detailed in Chapter 3.

A 64-year-old woman presents with fever, chills, headache, neck stiffness, vomiting, and confusion. The Kernig sign (passive knee extension eliciting neck pain) and Brudzinski sign (passive neck flexion eliciting bilateral hip flexion) are both positive. Examination of the cerebrospinal fluid reveals changes consistent with bacterial meningitis, and brain imaging demonstrates a localized abscess. Which of the following types of necrosis is most characteristic of abscess formation? (A) Caseous (B) Coagulative (C) Enzymatic (D) Fibrinoid (E) Liquefactive

The answer is C. This family likely has hereditary nonpolyposis colorectal cancer (HNPCC) syndrome, or Lynch syndrome. Often, gynecologic malignancies (usually endometrial, although patients are also prone to ovarian tumors) are diagnosed before colorectal tumors in female patients with HNPCC. This syndrome is due to defects in DNA mismatch repair genes, with MLH1, MSH2, MSH6, and PMS2 representing the primary culprits. WT-1 mutations predispose patients to Wilms tumors and a variety of other defects, while APCis mutated in familial adenomatous polyposis (FAP). Germline mutations in p53 are seen in Li Fraumeni syndrome, while BRCA-1mutations predispose women to breast carcinoma.

A 40-year-old woman presents with endometrial carcinoma. Her family history reveals that her mother died of endometrial cancer at age 50, while her 42-year-old brother was recently diagnosed with colon cancer. You begin to suspect a familial cancer syndrome. What gene is most likely to be mutated in this family? (A) WT-1 (B) APC (C) MSH2 (D) p53 (E) BRCA-1

The answer is B. In advanced form, primary (hereditary) hemochromatosis is characterized by the triad of cirrhosis, diabetes, and hyperpigmentation, or so-called bronze diabetes. The disease is most often caused by a mutation in the Hfegene on chromosome 6 and is characteristically familial rather than sporadic. The manifestations of the disorder are the result of iron overload and deposition of hemosiderin in tissues such as the liver, pancreas, skin, joints, and pituitary. Laboratory abnormalities of note include increased serum iron and decreased TIBC. The skin hyperpigmentation is due largely to increases in melanin and to lesser accumulations of hemosiderin.

A 45-year-old man is referred because of a recent diagnosis of hereditary hemochromatosis. Which of the following is a correct statement about this disorder? (A) Damage to organs results from abnormal deposition of lead (B) It can progress to liver cirrhosis, diabetes mellitus, and skin pigmentation (C) Most cases are due to spontaneous mutations (D) Skin hyperpigmentation is due to bilirubin accumulation (E) The TIBC is characteristically increased

The answer is C. Berry aneurysms, which occur in 10% to 15% of patients with adult polycystic kidney disease, are small saccular lesions that develop at the site of congenital weakness of cerebral arteries, especially those of the circle of Willis. Rupture of these aneurysms is the most common cause of subarachnoid hemorrhage. Arteriovenous fistulas are often secondary to trauma. Dissecting aneurysm is associated with hypertension or with diseases affecting the vascular media, most notably Marfan syndrome. Syphilitic (luetic) aneurysm is associated with tertiary syphilis.

A 45-year-old man presents with abdominal pain and hypertension. On physical examination, he is found to have an abdominal mass. Further workup confirms the diagnosis of adult polycystic kidney disease. Which of the following vascular complications is associated with this condition? (A) Arteriovenous fistula (B) Atherosclerotic aneurysm (C) Berry aneurysm (D) Dissecting aneurysm (E) Luetic aneurysm

The answer is A. The patient exhibits the cardinal findings of the CREST syndrome, a less severe variant of systemic sclerosis (scleroderma) characterized by Calcinosis, Raynaud phenomenon, Esophageal dysfunction, Sclerodactyly, and Telangiectasia. Although a number of antinuclear antibodies can be found in this disorder, the most characteristic is directed at centromeric proteins (the antibody is often referred to as anticentromere).

A 45-year-old woman is seen because of varied complaints. She has been troubled by small painful lumps under the skin of her fingers, some of which have ruptured and leaked a chalky white substance. She also complains of painful episodes in her fingers and toes, which blanch and turn blue on exposure to cold. In addition, questioning reveals increasing "heartburn" and difficulty swallowing. Examination reveals thickening of the skin of the fingers and toes, resulting in a claw-like appearance. Telangiectatic clusters of vessels, appearing as small focal red lesions, are observed in the skin of the face, upper trunk, and hands, and on the mucosal surface of the lips. Antibodies to which of the following are most characteristic of the findings presented by this patient? (A) Centromeric proteins (B) Histidyl-t-RNA synthetase (Jo-1) (C) Histones (D) Mitochondria (E) Native DNA

The answer is B. The translocation t(9;22) is the characteristic translocation associated with chronic myelogenous leukemia, forming the so-called "Philadelphia chromosome." The resultant fusion protein, p210, has increased tyrosine kinase activity that contributes to the uncontrolled proliferation in this form of leukemia. The translocation t(14;18) is seen in follicular lymphoma; t(8;14) in Burkitt lymphoma; t(15;17) in the M3 variant of acute promyelocytic leukemia (AML); and t(11;22) in Ewing sarcoma, a relatively uncommon tumor of bone.

A 46-year-old woman with prominent splenomegaly presents with a 3-month history of malaise, easy fatigability, weakness, weight loss, and anorexia. A complete blood count and differential demonstrates a white blood cell count of 250,000/mm3 (normal 3,000 to 10,000/mm3) with a predominance of myelocytes, metamyelocytes, band cells, and segmented neutrophils. Cytogenetic analysis is most likely to reveal which of the following translocations? (A) t(8;14) (B) t(9;22) (C) t(11;22) (D) t(14;18) (E) t(15;17)

The answer is D. Wernicke-Korsakoff syndrome presents with the classic triad of ataxia, confabulation, and ophthalmoplegia and is seen in alcoholics secondary to a deficiency of the vitamin thiamine. Fetal alcohol syndrome occurs in infants born to mothers who consume alcohol during pregnancy and results in microcephaly, mental retardation, and facial and cardiac defects. Li-Fraumeni syndrome and the Lynch syndrome are both familial defects in DNA repair that lead to a propensity to develop particular cancers. Wiskott-Aldrich syndrome is an inborn immune defect presenting with eczema, thrombocytopenia, and recurrent infections.

A 50-year-old chronic alcoholic is seen in the emergency department with ataxic gait, confusion, confabulation, and nystagmus. This constellation of findings is the classic presentation of (A) Fetal alcohol syndrome. (B) Li-Fraumeni syndrome. (C) Lynch syndrome. (D) Wernicke-Korsakoff syndrome. (E) Wiskott-Aldrich syndrome.

The answer is D. Cachexia, or wasting due to cancer, manifests with weakness, weight loss, anorexia, anemia, and infection. The principal cytokine responsible for such changes is tumor necrosis factor-α(TNF-α). Both platelet-derived growth factor and fibroblast growth factor are involved in wound healing. Interleukin-2 (IL-2) is an immunostimulating cytokine produced by activated T cells. Vascular endothelial growth factor is important in the proliferation of blood vessels in a growing tumor.

A 68-year-old man has a long history of prostate cancer that was metastatic at the time of diagnosis. Over the past 2 months, he has had significant weight loss, loss of appetite, and loss of energy. His current spectrum of conditions can be attributed to which of the following? (A) Platelet-derived growth factor (B) Fibroblast growth factor (C) Interleukin-2 (D) Tumor necrosis factor-α (E) Vascular endothelial growth factor

The answer is C. Cyanide is an inhibitor of the electron transport chain (ETC) within the mitochondria. Inhibition of the ETC results in the inhibition of intracellular oxidative phosphorylation, with depletion of cellular energy stores and, ultimately, death. Acute tubular necrosis can occur under many circumstances, as in cases of rhabdomyolysis secondary to crush injury. Inhibition of the oxygen-carrying capacity of hemoglobin can result from carbon monoxide poisoning. Induction of direct DNA damage can result from radiation exposure.

A 50-year-old man who works for a chemical company is being held on charges of murdering his wife. When the police found the woman's body, they noted no signs of physical injury but detected a scent of bitter almonds. A small sample of cyanide was recently reported missing from the husband's workplace. Based on these investigational findings, a justifiable working hypothesis for the forensic pathologist is that the death resulted from (A) acute tubular necrosis. (B) induction of direct DNA damage. (C) inhibition of intracellular oxidative phosphorylation. (D) inhibition of the oxygen-carrying capacity of hemoglobin. (E) rhabdomyolysis.

The answer is C. The findings are consistent with occlusion of the middle cerebral artery, the most common site of arrest of arterial emboli in branches of the carotid artery. Such emboli usually arise from a mural thrombus in the left atrium or left ventricle. Left atrial mural thrombi are especially associated with mitral stenosis with atrial fibrillation. Mural thrombi in the left ventricle are caused by myocardial infarction. Thrombi at the junction of the internal and external carotid arteries are a cause of thrombotic brain infarcts and can also be a site of origin of emboli.

A 50-year-old right-handed man with a long history of rheumatic heart disease with mitral stenosis and atrial fibrillation is brought to the emergency department after collapsing to the floor at home. He is unable to speak or walk and has right hemiplegia with a right extensor plantar response. These findings most likely result from embolism to which of the following arteries? (A) Anterior cerebral (B) Anterior communicating (C) Middle cerebral (D) Posterior communicating (e) Superior cerebellar

The answer is A. The clinical findings are those of type I Gaucher disease, which is a manifestation of glucocerebrosidase deficiency. The disorder is most often seen in persons of European (Ashkenazic) Jewish lineage. Prominent findings include bone pain and fractures, easy bruising, hepatosplenomegaly, anemia, and thrombocytopenia. Bone marrow aspiration reveals numerous typical Gaucher cells, but specific enzyme assay is required to confirm the diagnosis. This lysosomal storage disease is relatively mild compared to a number of other such entities, such as Tay-Sachs disease and Niemann-Pick disease, which are rarely seen in adults. The disease is highly variable in its clinical manifestations, and assays of chitotriosidase and angiotensin-converting enzyme, markers of macrophage proliferation, are useful measures of the extent of disease and of its control.

A 50-year-old woman of Eastern European Jewish ancestry has a history of recurrent fractures and easy bruising and is found to have hepatosplenomegaly and mild anemia. Serum assays reveal elevations of chitotriosidase and angiotensin-converting enzyme. Assay of cultured leukocytes most likely reveals marked deficiency of which of the following enzymes? (A) Glucocerebrosidase (B) α-1,4-Glucosidase (C) Hexosaminidase A (D) α-L-Iduronidase (E) Sphingomyelinase

The answer is D. Isoniazid is a competitive inhibitor of pyridoxine (vitamin B6), which is required for the synthesis of the inhibitory neurotransmitter γ-aminobutyric acid. Riboflavin deficiency is rare and can result in cheilosis, glossitis, and other epithelial changes. Niacin deficiency results in pellagra. Pyridoxine deficiency can manifest with convulsions. Vitamin C deficiency results in defects in collagen synthesis. Thiamine deficiency results in neuropathy, cardiomyopathy, and mental status changes.

A 52-year-old recent Asian immigrant is brought to the emergency department after experiencing several convulsions. Further history reveals that she has been diagnosed with tuberculosis and has recently been started on a multidrug regimen that includes isoniazid. Which of the following is the likely cause of her convulsions? (A) Vitamin B1 (thiamine) deficiency (B) Vitamin B2 (riboflavin) deficiency (C) Vitamin B3 (niacin) deficiency (D) Vitamin B6 (pyridoxine) deficiency (E) Vitamin C (ascorbic acid) deficiency

The answer is E. Desmoplasia refers to proliferation of non-neoplastic fibrous connective tissue within a tumor and is quite common in cases of breast cancer. An irregular accumulation of blood vessels is known as a hemangioma. An area of tissue misplaced within another organ is known as a choristoma.

A 54-year-old woman who has been diagnosed with early-stage breast cancer undergoes surgery for a lumpectomy to remove a small tumor detected by mammography. The pathology report confirms the early stage of the cancer and further comments on the fact that there is significant desmoplasia in the surrounding tissue. The term desmoplasia refers to (A) an irregular accumulation of blood vessels. (B) maturation and spatial arrangement of cells. (C) metastatic involvement of surrounding tissue. (D) normal tissue misplaced within another organ. (E) proliferation of non-neoplastic fibrous connective tissue.

The answer is E. Myelin figures, cell blebs, mitochondrial swelling, and glycogen depletion are all signs of reversible injury. Nuclear changes such as pyknosis, karyorrhexis, and karyolysis are signs of cell death and are, of course, irreversible.

A 56-year-old man dies 24 hours after the onset of substernal chest pain radiating down his left arm to the ulnar aspect of his fingertips. Which of the following morphologic myocardial findings is an indicator of irreversible injury? (A) Cell blebs (B) Depletion of glycogen (C) Mitochondrial swelling (D) Myelin figures (E) Pyknotic nuclei

The answer is E. Severe combined immunodeficiency disease is characterized by failure to thrive and increased susceptibility to bacterial, fungal, and viral infections. Laboratory studies reveal decreased numbers of both B cells and T cells and deficiency of immunoglobulins. The treatment of choice is bone marrow (or other sources of hematopoietic stem cells) transplantation and is based on maturation of donor lymphoid progenitor cells.

A 7-month-old boy has had multiple bouts of otitis media, sinusitis, bronchitis, oral candidiasis, and multiple viral infections. Cessation of the recurrent infections follows successful engraftment of a bone marrow transplant. The basis of the clinical improvement is (A) direct transfusion of antibodyproducing B cells. (B) direct transfusion of donor CD4+ and CD8+ lymphocytes. (C) donor suppression of recipient cytotoxic T cells. (D) infusion of donor-derived cytokines. (E) maturation of donor lymphoid progenitor cells.

The answer is B. The phenomenon of earlier and more severe manifestations of a disorder in successive generations (anticipation) is a characteristic of many trinucleotide repeat disorders, the best known examples of which are fragile X syndrome (discussed in this chapter) and Huntington disease (described in this clinical scenario and further discussed in Chapter 23). The degree of expansion is closely related to the gender of the parent with the genetic abnormality. In the fragile X syndrome, expansion occurs during oogenesis. In Huntington disease, expansion occurs during spermatogenesis. Even though trinucleotide repeats almost always involve guanine and cytosine (G and C), the third nucleotide is different in the two conditions: CGG in fragile X syndrome and CAG in Huntington disease.

A 56-year-old man dies of a 15-year progressive illness characterized by athetoid movements and deterioration leading to hypertonicity, fecal and urine incontinence, anorexia and weight loss, and eventually dementia and death. The disease is known to have an autosomal dominant mode of inheritance and to be due to an abnormality in a gene on chromosome 4 that is altered by increased numbers of intragenic trinucleotide repeats. In addition, this disorder has an earlier onset and is more debilitating in successive generations, a phenomenon that might be due to (A) a shift from trinucleotide repeats to pentanucleotide repeats. (B) an increase in the number of trinucleotide repeats in successive generations. (C) defects in membrane receptors and transport systems. (D) imprinting variability in successive generations. (E) increased medical awareness of the condition.

The answer is E. Deficiency of fat-soluble vitamins (vitamins A, D, E, and K) can occur in chronic pancreatitis due to loss of exocrine pancreas function. Vitamin B12 (cobalamin), folic acid, vitamin B2 (riboflavin), and vitamin B6 (pyridoxine) are all water-soluble vitamins. It should be noted that most patients with chronic pancreatitis also are alcoholics and that alcoholics often have multiple nutritional deficiencies, including lack of watersoluble vitamins.

A 57-year-old man is admitted to the hospital for treatment of chronic pancreatitis. In patients with chronic pancreatitis, deficiency of which of the following vitamins is most likely? (A) Folic acid (B) Vitamin B2 (riboflavin) (C) Vitamin B6 (pyridoxine) (D) Vitamin B12 (cobalamin) (E) Vitamin D

The answer is E. The man is likely to have a lung tumor, given his clinical presentation and the radiographic results. The patient's hypercalcemia is likely due to a paraneoplastic syndrome, such as that due to the elaboration of parathyroid-related hormone (PTrH). PTrH is produced by squamous cell carcinoma, whereas adrenocorticotropic-like substance and antidiuretic hormone are produced by yet another form of lung cancer—small cell carcinoma of the lung. Carcinoembryonic antigen is an oncofetal antigen produced by colon cancer cells. Erythropoietin causes secondary polycythemia and is related to renal cell carcinoma.

A 58-year-old man with a 700-pack-peryear smoking history presents to the emergency department with shortness of breath and hemoptysis. Portable chest radiography demonstrates a large mass centrally located within the left lung field. The serum calcium is 13.0 mg/dL (normal 8.5 to 10.2). The metabolic abnormality described here is likely due to elaboration of which substance? (A) Adrenocorticotropic hormone-like substance (B) Antidiuretic hormone (C) Carcinoembryonic antigen (D) Erythropoietin (E) Parathyroid-related hormone

The answer is D. Glycoprotein IIb-IIIa inhibitors prevent the action of the corresponding platelet surface receptor glycoprotein complex, which is required for formation of fibrinogen bridges between adjacent platelets.

A 60-year-old man with unstable angina (a form of acute coronary syndrome) is treated with an intravenously administered glycoprotein IIb-IIIa inhibitor. The mechanism of action of this agent is the ability to (A) dilate coronary arteries. (B) inhibit atherogenesis. (C) inhibit platelet adhesion. (D) inhibit platelet aggregation. (e) lyse thrombi.

The answer is D. Metastatic calcification, or deposition of calcium in previously normal tissue, is caused by hypercalcemia. In this patient, tumor metastases to the bone with increased osteolytic activity caused mobilization of calcium and phosphate, resulting in hypercalcemia. Metastatic calcification should be contrasted with dystrophic calcification, in which the serum calcium concentration is normal and previously damaged tissues are the sites of deposition.

A 60-year-old woman with breast cancer and widespread bony metastases is found to have calcification of multiple organs. The calcifications are best described as (A) dystrophic with decreased serum calcium. (B) dystrophic with increased serum calcium. (C) metastatic with decreased serum calcium. (D) metastatic with increased serum calcium.

The answer is E. Folate is required for DNA replication in rapidly dividing cells, such as red blood cell precursors, and a deficiency results in megaloblastic anemia. Hemorrhagic lesions of the mamillary bodies and high-output heart failure occur with thiamine deficiency. Impaired wound healing and defective osteoid matrix production result from vitamin C deficiency.

A 62-year-old woman with a long history of rheumatoid arthritis was recently placed on therapy with methotrexate (a folic acid antagonist). The physician should be on the alert for which of the following side effects of this newly added medication? (A) Defective osteoid matrix production (B) Hemorrhagic lesions of the mamillary bodies (C) High-output heart failure (D) Impaired wound healing (E) Megaloblastic anemia

The answer is D. Her-2/neu, also known as c-erbB2, is a receptor tyrosine kinase related to epidermal growth factor receptor and is amplified at the DNA level and overexpressed at the protein level in some breast cancers. Ras is a GTPase that is mutated in a number of cancers. NF-1 is a GTPase-activating protein (GAP) aberrantly expressed in neurofibromatosis. An aberrant version of a retinoic acid receptor is expressed in M3 AML.

A 63-year-old woman discovers a lump in her right breast. Mammography confirms the presence of a suspicious "lump," and a needle core biopsy is performed to determine whether the mass is malignant. The pathology report confirms that the mass is indeed cancerous and that the tissue demonstrates amplification of the Her-2/neu oncogene. The gene product of Her-2/neu is what kind of protein? (A) GTPase (B) GTPase-activating protein (C) Nuclear transcription factor (D) Receptor tyrosine kinase (E) A retinoic acid receptor protein

The answer is C. The clinical findings are those of "stroke," or cerebrovascular disease. This group of entities encompasses injury to the brain caused by disorders of the cerebral vasculature, such as thrombosis, embolism, and hemorrhage (see Chapter 3). The most important consequence is damage to neurons, because neurons are considered to be "permanent" cells, incapable of division and replication (however, this has been recently challenged as the result of provocative stem cell research). Permanent cells are exemplified by neurons and myocardial cells. Labile cells, such as cells of the epidermis and gastrointestinal mucosa, divide throughout the life of the individual. Stable cells, such as hepatocytes and renal tubular cells, do not divide regularly but have the capacity to divide and regenerate as needed.

A 70-year-old man presents with the sudden onset of left-sided weakness, spasticity, and hyperactive and pathologic reflexes. The most serious consequences of this disorder are the result of damage to which of the following cell types? (A) Labile cells (B) Multipotent adult progenitor cells (c) Permanent cells (d) Stable cells

The answer is B. A clear, straw-colored fluid with low protein and low specific gravity is a transudate, and the term hydrothorax refers to the accumulation of a significant volume of transudate within the pleural cavities (to be detected by chest radiograph, about 200 to 400 mL of pleural fluid must be present). The most common cause of hydrothorax is cardiac failure, which may be either unilateral or bilateral (bilateral is more common). It is incumbent on the clinician to distinguish pleural transudates from exudates, because the causes of each are quite different.

A 70-year-old man seeks medical attention because of shortness of breath on minimal exertion. A posteroanterior chest radiograph reveals blunting of the right costophrenic sulcus interpreted as a rightsided pleural effusion. The aspirated fluid is straw colored and clear. The protein concentration is low, and the specific gravity is 1.011. Microscopic examination reveals an occasional mesothelial cell. Which of the following is the most likely cause of the effusion? (A) Decreased oncotic pressure (B) Left ventricular heart failure (C) Mesothelioma (D) Pneumonia (e) Tuberculosis

The answer is E. The patient has bacterial pneumonia due to Streptococcus pneumoniae, a classic example of severe acute inflammation. In the early stages of acute inflammation, the neutrophil is the most prominent inflammatory cell. It is noteworthy that, in many instances, bacterial infections are characterized by neutrophilic infiltrates. It is also noteworthy that S. pneumoniae(also known as the "pneumococcus") is the most common etiologic agent of lobar pneumonia (see Chapter 14).

A 72-year-old man presents with a 3-day history of progressively worsening productive cough, fever, chills, and signs of toxicity. Prominent physical findings include signs of consolidation and rales over the right lung base. Sputum culture is positive for Streptococcus pneumoniae.An intra-alveolar exudate filling the alveoli of the involved portion of the lung is present. Which of the following types of inflammatory cells is most likely a prominent feature of this exudate? (A) Basophils (B) Eosinophils (c) Lymphocytes (d) Monocytes-macrophages (e) Neutrophils

The answer is D. C. difficilecauses diarrhea in patients in whom competing colonic flora has been obliterated by antibiotics. Candidal infections are also common in patients on antibiotics but typically manifest with oral or vaginal candidiasis, rather than diarrhea. MRSAis a growing problem both in the hospital and in the community but does not typically cause diarrhea (although it could well be the cause of the patient's drug-resistant pneumonia!). C. botulinumcauses botulism, a rare but sometimes fatal paralytic disorder, and is not associated with diarrhea.

A 73-year-old man develops severe, intractable diarrhea during hospitalization for bacterial pneumonia caused by a multidrug-resistant organism. What organism is most likely responsible for his gastrointestinal symptoms? (A) MRSA (B) Streptococcus pneumoniae (C) Candida (D) Clostridium difficile (E) Clostridium botulinum

The answer is B. Tobacco contributes to the development of many cancers, including those of the bladder, lung, throat, and esophagus. Asbestos exposure carries a risk of lung cancer, as well as mesothelioma, a cancer of the pleura. Tobacco and asbestos function as cocarcinogens in the pathogenesis of lung cancer, with an approximately 50-fold greater risk of developing bronchogenic cancer of the lung than in those without such exposure.

A 78-year-old Navy veteran with a 600-pack-per-year history of cigarette smoking presents with cancer. During his military career, he was involved in fireproofing naval combat ships with asbestos insulation. Given his environmental exposure to both tobacco and asbestos, to which cancer do both of these carcinogens contribute? (A) Bladder cancer (B) Bronchogenic cancer (C) Cancer of the throat (D) Esophageal cancer (E) Mesothelioma

The answer is B. Carbon monoxide (CO) binds to hemoglobin with approximately 200 times greater affinity than oxygen, preventing delivery of oxygen to tissues. In addition, CO is an inhibitor of cytochrome oxidase, impairing cellular respiration. Ethylene glycol (antifreeze) is associated with kidney damage and the formation of birefringent calcium oxalate crystals in the urinary tract. Hepatic necrosis and fatty change result from carbon tetrachloride poisoning. Lead inhibits a key enzyme in hemoglobin synthesis as well as iron incorporation in hemoglobin.

A 78-year-old man is found in his closed room unresponsive in bed after the first cold day of winter. There is a kerosene heater still on from the previous night. On attempts to arouse him, officers note the cherry hue of his lips, cheeks, and mucous membranes. The likely mechanism of his death was (A) accidental ingestion of ethylene glycol. (B) binding of carbon monoxide to hemoglobin. (C) hepatic necrosis with fatty change. (D) inhibition of hemoglobin production. (E) inhibition of incorporation of iron into hemoglobin.

The answer is B. The clinical description is characteristic of pulmonary infarction, which, in turn, most often results from thromboembolism originating from thrombosis in the lower extremity veins. Because venous thrombosis is associated with impaired blood flow, this condition is particularly characteristic of immobilization, which is often seen in elderly, debilitated, or chronically bedridden persons.

A bedridden elderly patient experiences the sudden onset of pleuritic pain and hemoptysis. The underlying lesion that led to this complication was most likely located in which of the following sites? (A) Hepatic veins (B) Lower extremity veins (C) Pelvic veins (D) Portal vein (e) Pulmonary veins

The answer is D. Lack of stippling is a characteristic of gunshot wounds inflicted from long distances. In addition, the wounds are often oval with clean margins. Such evidence is more consistent with a homicide than a "staged" suicide. In general, entrance wounds are smaller than exit wounds, which are often irregularly shaped. Stellate-shaped wounds are associated with wounds over bony areas. Wounds from close range, as in a suicide, would be expected to have stippling and tattooing of skin caused by unburned gunpowder.

A forensic pathologist is asked to evaluate a fatal gunshot wound involving the left thorax of a 27-year-old man, a known drug dealer. Even though a gun and suicide note were found next to the body, the pathologist has concluded that the wound was probably not self-inflicted. Which of the following findings is most supportive of that conclusion? (A) The entrance wound is smaller than the exit wound. (B) The entrance wound is stellate-shaped. (C) The exit wound is irregularly shaped. (D) There is no stippling of the skin, and the wound is oval with clean margins. (E) There is stippling of the skin from unburned gunpowder.

The answer is A. Several substances have chemotactic potential for neutrophils (see Table 2-1). C5a is a prominent example.

A laboratory experiment is performed to evaluate the chemotactic potential of a group of potential mediators. Which of the following substances most likely has the greatest affinity for neutrophils? (A) C5a (B) Fucosyl transferase (c) β2-Integrin (d) P-selectin (e) TNF-α

The answer is C. In mitochondrial inheritance, inheritance is entirely maternal in transmission. Affected males do not transmit the trait to any of their children, and affected females transmit the trait to all of their children. Abnormalities of mitochondrial inheritance typically involve genes that code for enzymes of oxidative phosphorylation.

A newly described neurologic disorder is found to affect multiple family members in three generations that were available for study. In the first generation, two sisters and one brother were affected. In the second generation, all of the children of the first-generation sisters were affected, but none of the descendants of the first-generation son. In the third generation, all of the children of the affected second-generation women were affected, but none of the descendants of the secondgeneration men. The mode of inheritance exemplified here is (A) autosomal dominant. (B) autosomal recessive. (C) mitochondrial. (D) X-linked dominant. (E) X-linked recessive.

The answer is B. A positive Congo red test confirms the presence of amyloid. Apple green birefringence is observed under polarized light.

A pathologist examines a renal biopsy from a 45-year-old man with nephrotic syndrome and requests a Congo red stain to confirm the nature of an amorphous acidophilic extracellular hyaline substance localized within the mesangial matrix of the glomeruli. A positive test confirms the presence of (A) α1-antitrypsin. (B) amyloid. (C) copper. (D) glycogen. (E) hemosiderin.

The answer is B. This type of reaction is primarily mediated by the release of histamine from tissue mast cells, and the associated cellular infiltrate and peripheral blood findings represent mobilization and increased numbers of eosinophils. The symptoms reported are those of seasonal rhinitis, better known as "hay fever," a manifestation of type I hypersensitivity (see Chapter 5).

A routine complete blood count performed on a 22-year-old medical student reveals an abnormality in the differential leukocyte count. She has been complaining of frequent sneezing and "watery" eyes during the past several weeks and reports that she frequently had such episodes in the spring and summer. Which of the following cell types is most likely to be increased? (A) Basophils (B) Eosinophils (c) Lymphocytes (d) Monocytes (e) Neutrophils

The answer is E. The concentration of phenylalanine in affected infants is usually normal at birth and increases rapidly during the first days of life. False-negative results are common immediately after birth but are rare on the second and third days of life. Consequently, the blood sample for phenylketonuria is usually taken from the infant's heel within 2 to 3 days after birth. If the test is performed too early, the diagnosis could be missed.

A screening test for phenylketonuria (PKU) is performed on umbilical cord blood from a fair-skinned blond, blue-eyed infant born to dark-complexioned parents. The test is reported as negative, and no dietary restrictions are imposed. At 1 year of age, the child is seen again, this time with obvious signs of severe mental retardation, and a diagnosis of PKU is made. The diagnosis was missed at birth because (A) cord blood is not a good source of fetal blood. (B) the screening (Guthrie) test has low sensitivity. (C) the test should have been performed on maternal blood. (D) the test should have been performed on urine rather than blood. (E) the test was performed too early.

The answer is E. Deficiency of vitamin K results in abnormal bleeding, based on the requirement of the vitamin for γ-carboxylation of clotting factors II, VII, IX, and X. Because newborns have inadequate intestinal flora, the primary source of vitamin K, vitamin K deficiency is relatively common and is estimated to occur in approximately 3% of neonates. The resultant bleeding disorder is termed hemorrhagic disease of the newborn. Folic acid deficiency during early embryogenesis can result in neural tube defects. Vitamin A deficiency results in changes in vision and defects in epithelial cell function. Vitamin D deficiency in children results in rickets.

A woman from a rural Appalachian community who had recently given birth to a newborn boy at home with the aid of a midwife now brings her infant to the hospital because of continued bleeding and oozing from the umbilical stump. It is likely that the bleeding problem is secondary to a deficiency of which of the following vitamins? (A) Folic acid (B) Vitamin A (C) Vitamin D (D) Vitamin E (E) Vitamin K

The answer is B. Pediatric renal cell carcinomas often demonstrate translocations involving Xp11, with t(X;17) representing the most common anomaly. This translocation is also seen in alveolar soft part sarcoma, a fact which illustrates the lack of specificity of translocations in some instances. The t(X;18) translocation is associated with synovial sarcoma, while the other translocations listed are seen in hematologic malignancies as described in the answer to question #6.

An 18-year-old patient presents with renal cell carcinoma. Given that this is typically a tumor of older adults, what translocation might you expect to find? This translocation is also seen in which mesenchmyal malignancy? (A) t(X;18), synovial sarcoma (B) t(X;17), alveolar soft part sarcoma (C) t(9;22), leiomyosarcoma (D) t(14;18), leiomyosarcoma (E) t(15;17), synovial sarcoma

The answer is B. Patients with cystic fibrosis often have impaired exocrine pancreas function, resulting in a deficiency of fat-soluble vitamins. Of the fat-soluble vitamins, vitamin A deficiency results in night blindness, xerophthalmia, keratomalacia, and squamous metaplasia of the urinary tract, contributing to the formation of recurrent renal calculi.

An 18-year-old young man with known cystic fibrosis presents to the physician with his third episode of kidney stones in the past year. In addition, he has begun to complain of difficulty seeing at night. Such changes can be attributed to a deficiency of which vitamin? (A) Pyridoxine (B) Vitamin A (C) Vitamin B1 (D) Vitamin B12 (E) Vitamin D

The answer is B. Xeroderma pigmentosum is a hereditary DNA defect with a deficiency in the ability to repair ultraviolet (sunlight)-induced thymidine dimers. Faulty repair leads to increased sun sensitivity, with a predilection to develop skin lesions and skin cancers on exposed skin, as well as ocular melanomas. Aberrant expression of the gene for a receptor tyrosine kinase, such as the Her-2/neu gene product, can cause breast cancer. A retrovirus is responsible for the development of T-cell leukemia/lymphoma. The DNA virus human papillomavirus can cause cervical cancer.

An 8-year-old boy is referred to the dermatologist for numerous "suspicious" pigmented lesions on the face and neck. Further history reveals that the patient has had difficulty seeing out of his right eye; he is referred to the ophthalmologist, who diagnoses an ocular melanoma. Based on the patient's symptoms, the diagnosis of xeroderma pigmentosum is considered. This condition results from (A) aberrant expression of a receptor tyrosine kinase. (B) an inborn defect in DNA repair. (C) chemical carcinogenesis. (D) DNA viral infection. (E) retroviral infection.

The answer is A. The normal function of NF-1 is to promote the intrinsic GTPase function of the Rasoncogene. When the Ras protein is bound to GTP, the growth-promoting function of the molecule is "ON." On hydrolysis of the GTP to GDP, Ras is converted to an inactive state. GAPs, such as NF-1, suppress cell growth by stimulating GTP hydrolysis. Patients with a mutation in NF-1 are susceptible to fibrosarcomas as a result of loss of function of this GAP. The molecule Bax is pro-apoptotic and antagonizes Bcl-2. Vascular endothelial growth factor promotes tumor angiogenesis. The protein p53 regulates the cell cycle if DNA damage is detected. Lastly, caspases function to cleave cellular proteins once apoptosis is triggered.

An 8-year-old child is evaluated by the pediatrician, who notes what appear to be 10 small café-au-lait spots on the child's torso. In addition, on close inspection of the eyes, the presence of Lisch nodules is noted. The patient is diagnosed with von Recklinghausen neurofibromatosis type 1. The protein that is mutated in this disorder normally (A) activates the GTPase activity of Ras. (B) cleaves cellular proteins during apoptosis. (C) functions as a regulator of the cell cycle. (D) promotes angiogenesis in the growing tumor mass. (E) promotes the cell to undergo apoptosis.

The answer is E. The diagnosis is septic shock, most likely a result of gram-negative sepsis originating from a urinary tract infection. Gram-negative organisms contain lipopolysaccharide in the outer membrane, which triggers the release of cytokines, such as tumor necrosis factor, resulting in a cascade of events culminating in increased capillary permeability and redistribution of circulatory volume into the interstitium. Anaphylactic shock is a result of a type I hypersensitivity. Cardiogenic shock often results from myocardial infarction. Hypovolemic shock is a result of blood or fluid loss. Neurogenic shock can result from spinal cord injuries.

An 86-year-old man with a history of recurrent urinary tract infection presents with fever, tachypnea, tachycardia, mental obtundation, and reduced blood pressure. Which of the following forms of shock is most likely? (A) Anaphylactic shock (B) Cardiogenic shock (C) Hypovolemic shock (D) Neurogenic shock (e) Septic shock

The answer is E. The tuberculin test is a classic example of delayed hypersensitivity, a form of cell-mediated hypersensitivity involving CD4+T cells and macrophages. Native CD4+T cells are converted to TH1 cells that secrete cytokines, especially interferon-γ, which is a central mediator of delayed hypersensitivity. Among the many actions of interferon-γ, the most important is the activation of macrophages.

An HIV-positive intravenous drug user is suspected of having active tuberculosis, and a tuberculin (Mantoux) intradermal skin test is performed. After 48 hours, 10 cm of induration is observed. Which of the following are involved in this form of hypersensitivity reaction? (A) B cells and antibodies (B) Basophils and IgE (C) Immune complexes and complement (D) Plasma cells and IgM (E) T cells and macrophages

The answer is D. Because vitamin C is a required cofactor for the synthesis of hydroxyproline and hydroxylysine, which are both required for collagen synthesis, vitamin C deficiency can lead to impaired capillary formation, with consequent bleeding, and impaired wound healing. Both vitamin B12 and folic acid are required for DNA synthesis. Pyridoxine is required for GABA synthesis. Vitamin K is required for carboxylation of clotting factors. Patients with vitamin D-resistant rickets have a deficiency of renal 1α-hydroxylase.

An abused child is found living in the basement of his parents' home after they are arrested on drug charges. In addition to being severely malnourished, the child is found to have bleeding gums and easy bruisability, along with numerous poorly healing skin ulcerations. Assuming that these findings resulted from vitamin C deficiency, what is the likely mechanism of these findings? (A) Defective DNA synthesis (B) Defective production of γ-aminobutyric acid (C) Impaired carboxylation of coagulation factors II, VII, IX, and X (D) Impaired hydroxyproline and hydroxylysine production (E) Impaired renal 1α-hydroxylase

The answer is E. The sequence of events in hypoxic cell damage is as follows: Hypoxia results in failure of oxidative phosphorylation, with resultant depletion of ATP and increase in adenosine monophosphate and adenosine diphosphate. Anaerobic glycolysis and glycogenolysis are stimulated (notinhibited) through increased phosphofructokinase and phosphorylase activities, respectively. This results in an accumulation of cell lactate, with a decrease in intracellular pH and depletion of cellular glycogen stores. Decreased availability of ATP also results in failure of the Na+K+-ATPase pump, which then leads to increased cell Na+ and water and decreased cell K+.

An impending myocardial infarction was successfully averted by thrombolytic (clot-dissolving) therapy in a 55-year-old man. Which of the following biochemical events most likely occurred during the period of hypoxia? (A) Decreased hydrogen ion concentration (B) Increase in oxidative phosphorylation (C) Loss of intracellular Na+ and water (D) Stimulation of ATP synthesis (E) Stimulation of anaerobic glycolysis and glycogenolysis

The answer is C. Multifactorial disorders are among the most common familial abnormalities and are much more common than monogenic disorders. They include a number of common entities, such as ischemic heart disease, diabetes mellitus, hypertension, gout, schizophrenia, bipolar disorder, and neural tube defects.

As part of a fourth-year elective, a medical student rotating through a medical genetics service is assigned to counsel a patient who is concerned about a family history of hypertension. To be properly prepared for the counseling session, the student reviews course notes on modes of inheritance of various disorders. Knowledge of which of the following modes of inheritance is most pertinent to the upcoming discussion with the patient? (A) Autosomal dominant (B) Autosomal recessive (C) Multifactorial (D) X-linked dominant (E) X-linked recessive

The answer is A. von Willebrand factor is required for platelet adhesion to the subendothelium of damaged blood vessels. See further discussion of von Willebrand disease in Chapter 13.

During a laboratory exercise on coagulation testing, a 23-year-old medical student is found to have a prolonged bleeding time. She has had a long history of "easy bleeding," with frequent bleeding of the gums, epistaxis, cutaneous bleeding, and menorrhagia. Further testing revealed a deficiency of von Willebrand factor. Which of the following thrombogenic processes involving platelets is most directly impaired? (A) Adhesion (B) Conformational change with activation of phospholipid surface (C) Formation of fibrinogen bridges (D) Release reaction (e) Stabilization of platelet plug

The answer is A. The patient has homogentisic oxidase deficiency, a rare inborn error of metabolism (actually the first such disorder described by Garrod in 1902), clinically manifest by alkaptonuria and ochronosis. The term alkaptonuria refers to urinary excretion of unmetabolized homogentisic acid imparting a dark color to urine on standing. The term ochronosis refers to pigment deposition in multiple tissues, most prominently in cartilage and connective tissue. Most symptoms result from joint involvement, which can lead to disabling arthritis as patients age. Other affected structures include the eyes, larynx and bronchi, heart and vessels, prostate, and sweat glands.

During a routine physical examination, a 41-year-old woman is noted to have blueblack pigmented patches in the sclerae and gray-blue discoloration of the ear cartilages. The extensor tendons of the hands exhibit similar discoloration when she is asked to "make a fist." On questioning, the patient vaguely remembers hearing her mother say that the patient had dark discoloration on her diapers when she was an infant. Her only current complaint is slowly increasing pain and stiffness of the lower back, hips, and knees. A urine sample darkens on standing. These findings are characteristic of a deficiency of which of the following enzymes? (A) Homogentisic oxidase (B) Hypoxanthine-guanine phosphoribosyltransferase (C) L-Iduronosulfate sulfatase (D) Ketoacid decarboxylase (E) Phenylalanine hydroxylase

The answer is B. The patient has cirrhosis of the liver secondary to chronic alcoholism. A prominent manifestation of this disorder is decreased hepatic synthesis of albumin, the most significant contributor to plasma oncotic pressure. In addition, ascites is associated with increased sodium and water retention because of stimulation of the renin-angiotensin system. Also, hydrostatic forces (because of intrahepatic scarring and partial obstruction of the portal venous return) result in fluid transudation and increased secretion of hepatic lymph.

Fluid is aspirated from the grossly distended abdomen of a 47-year-old chronic alcoholic man. The fluid is straw colored and clear and is found to have a protein content (largely albumin) of 2.5 g/dL. Which of the following is a major contributor to the fluid accumulation in this patient? (A) Blockage of lymphatics (B) Decreased oncotic pressure (C) Decreased sodium retention (D) Increased capillary permeability (e) Inflammatory exudation

The answer is A. Cocaine is associated with increased myocardial irritability and sometimes with myocardial infarction, hypertension, and cerebral vascular accident. Both ethyl alcohol and heroin are long-term contributors to heart disease but are unlikely to cause an acute event in a young patient. Methyl alcohol can lead to blindness, as well as kidney damage. Phenacetin, an analgesic related to acetaminophen, can lead to kidney damage and is no longer used in this country.

Following his return home from a party, a 22-year-old man develops crushing chest pain and is brought to the emergency department by ambulance. Questioning reveals no cardiac risk factors. Electrocardiographic and serum enzyme findings are consistent with acute myocardial infarction. It is suspected that the cardiac damage is related to toxicity of a drug of abuse. Which of the following drugs is most likely? (A) Cocaine (B) Ethyl alcohol (C) Heroin (D) Methyl alcohol (E) Phenacetin

The answer is D. PGI2 is a prostaglandin that is synthesized and expressed primarily in endothelial cells. It is a product of the cyclooxygenase pathway of arachidonic acid metabolism, which is inhibited by aspirin. PGI2 is a potent vasodilator and platelet antiaggregant. These properties are often contrasted with those of TxA2, which is primarily synthesized in platelets and is a vasoconstrictor and platelet aggregant. The other compounds are products of the lipoxygenase pathway of arachidonic acid metabolism, which is not inhibited by aspirin.

In a laboratory exercise for medical students, an unknown compound is studied. The students are informed that the compound has been isolated from endothelial cells and that its synthesis can be inhibited by aspirin. In the laboratory, the students demonstrate that the compound is a potent vasodilator and platelet antiaggregant. Given these findings, the substance is most likely which of the following mediators? (A) 5-HPETE (B) LTC4 (c) LXA4 (d) PGI2 (e) TxA2

The answer is C. The clinical description and the figure are both typical of advanced secondary tuberculosis. Although this disorder is now relatively uncommon, its incidence is increasing, especially in association with immunodeficiency. Tuberculosis is a classic cause of granulomatous inflammation, which is characterized by the presence of "granulomas," which by definition consist of clusters of modified macrophages referred to as epithelioid cells. Additional features such as caseous necrosis, giant cell formation, and identifiable etiologic agents may or may not be present and are not invariable features of this form of inflammation. Granulation tissue is a feature of early repair and is totally unrelated to granulomatous inflammation.

The accompanying figure is representative of the findings in a hilar lymph node from a 54-year-old man who sought medical care for low-grade fever, anorexia, fatigue, night sweats, weight loss, and persistent cough with bouts of hemoptysis. A chest x-ray had revealed a right apical infiltrate with beginning cavitation, and examination of the sputum had revealed acid-fast bacilli. This condition is typified by a form of inflammation that invariablyincludes which of the following? (A) A morphologically identifiable etiologic agent (B) Caseous necrosis (c) Clusters of epithelioid cells (d) Multinucleated giant cells (e) Prominent granulation tissue

The answer is A. The figure illustrates fatty change of the liver, which is characterized by the accumulation of intracellular parenchymal triglycerides. It is seen most frequently in the liver, heart, and kidney and is commonly secondary to alcoholism. Fatty change results from an imbalance between the uptake, utilization, and mobilization of fat from liver cells. Alcoholic fatty liver may be reversible with complete abstinence from alcohol.

The illustration is from a liver biopsy of a 34-year-old woman with a long history of alcoholism. Which of the following is the best explanation for the changes shown here? (A) Accumulation of triglycerides within hepatocytes (B) Apoptosis with replacement of damaged cells by lipid-laden macrophages (C) Bilirubin accumulation with mobilization of fat by bile salts (D) Enzymatic fat necrosis with digestion of liver parenchyma by released enzymes (E) Irreversible damage to mitochondria

The answer is D. The illustration shows marked hypertrophy of the left ventricle. Hypertrophy of this extent, often seen in hypertensive heart disease, is caused by increased workload from increased ventricular pressure. This organ enlargement is the result of an increase in size of the individual muscle cells.

The illustration shows a section of the heart from a 45-year-old African-American man with long-standing hypertension who died of a "stroke." Which of the following adaptive changes is exemplified in the illustration? (A) Aplasia (B) Atrophy (C) Hyperplasia (D) Hypertrophy (E) Hypoplasia

The answer is C. Patients with Down syndrome are at increased risk of lymphoblastic leukemia. In addition, there is common occurrence of congenital heart disease, especially defects of the endocardial cushion (atrioventricular valve malformations and atrial and ventricular septal defects), and increased susceptibility to infection. Many patients with Down syndrome who are older than 35 years of age show clinical signs, symptoms, and pathologic findings of Alzheimer-type dementia, with an incidence much higher than in the general population.

The parents of a 17-year-old boy with Down syndrome seek counseling because they are concerned that their son may develop a life-threatening disorder known to be associated with his chromosomal abnormality. The physician should be prepared to discuss which of the following disorders in terms of its association with Down syndrome? (A) Berry aneurysm of the circle of Willis (B) Creutzfeldt-Jakob disease (C) Lymphoblastic leukemia (D) Medullary carcinoma of the thyroid (E) Osteosarcoma

The answer is B. The figure illustrates general preservation of myocardial architecture with some fragmentation, more intense cytoplasmic staining corresponding to increased cellular eosinophilia, and loss of nuclei, all of which are characteristics of coagulative necrosis.

This figure illustrates the microscopic appearance of the heart of a 56-year-old man who died after a 24-hour hospitalization for severe "crushing" chest pain complicated by hypotension and pulmonary edema. The type of necrosis shown is best described as (A) caseous. (B) coagulative. (C) fibrinoid. (D) gangrenous. (E) liquefactive.

This is the answer side of the first card in this set. Make sure you select "definition" in the "start with" box. Otherwise you will just see the answers first, which isn't super productive. (I had to do this because quizlet only allows images on the back of cards. Which is dumb. But we work with what we've got).

This is the question side of the first card in this set. Make sure you select "definition" in the "start with" box. Otherwise you will just see the answers first, which isn't super productive. (I had to do this because quizlet only allows images on the back of cards. Which is dumb. But we work with what we've got). I also recommend "flow" instead of "flip," but that's just a personal preference.

The answer is C. The figure shows a segment of a large atheromatous plaque. The clinical scenario strongly suggests that this affected vessel is a coronary artery. Cleft-like spaces indicating the presence of cholesterol crystals are very prominent and help in the identification of the lesion. The incidence of atherosclerosis is strongly associated with hypercholesterolemia. Renal disease is the most frequent cause of secondary hypertension. Endocrine disorders, such as pheochromocytoma, Conn syndrome, and acromegaly, represent the next most common cause. Coarctation of the aorta is a frequent cause of hypertension limited to the upper extremities.

This segment of the arterial intima of a coronary vessel is observed at autopsy in a 56-year old man who died suddenly on rising in the morning. Which of the following abnormalities is considered a major risk factor for the development of this lesion? (A) Congenital vascular muscle weakness (B) Cystic medial necrosis (C) Hypercholesterolemia (D) Medial calcification (E) Syphilis

The answer is E. The history is that of pulmonary embolism and infarction, a danger of immobilization, and the postoperative state. The infarct consists of an area of coagulative necrosis with superimposed hemorrhage, a combination referred to as a hemorrhagic, or red, infarct. Red infarcts are typical of tissues with a redundant arterial blood supply. Prominent examples are the lung with its double circulation from the pulmonary and bronchial arteries and the gastrointestinal tract with its multiple anastomoses between branches of the mesenteric artery. When a portion of the blood supply is obstructed, other portions remain patent, which can lead to hemorrhage into the infarcted area.

Two days following a cholecystectomy, a 32-year-old hospitalized woman has sudden onset of dyspnea, pleural pain, and cough productive of frothy, blood-tinged sputum. Ventilation-perfusion scintigraphy indicates a perfusion defect. If it were possible to examine a portion of the affected lung, which of the following would most likely have been found? (A) Air embolism (B) Anemic (white or pale) infarct (C) Disseminated intravascular coagulation (DIC) (D) Generalized thrombosis (e) Hemorrhagic (red) infarct

The answer is B. Lead poisoning leads to basophilic stippling of erythrocytes and a hypochromic, microcytic anemia. Megaloblastic anemias are seen in folate and B12 deficiencies. Lead lines are seen in the oral mucosa, not within peripheral blood cells. Microangiopathic changes are characteristic of disseminated intravascular coagulation and other disorders with mechanical disruption of RBCs, while leukocytosis can be seen in a variety of reactive and neoplastic hematologic conditions.

Which of the following peripheral blood findings is associated with lead poisoning? (A) Megaloblastic anemia (B) Basophilic stippling (C) Microangiopathic changes (D) Lead line (E) Leukocytosis

The answer is D. These children suffer from kwashiorkor, a form of protein-calorie malnutrition that is associated with a protein-poor diet. Kwashiorkor should be distinguished from the relative deficiency of all calories known as marasmus. Anorexia and bulimia are psychiatric eating disorders that are significantly more prevalent in developed countries. Beriberi is due to deficiency of thiamine.

While working with an international group of physicians to administer polio vaccines, a medical student sees several children with abdominal distention and pale streaks in the hair and skin. Cursory physical examination reveals significant hepatomegaly. The children likely suffer from (A) anorexia. (B) beriberi. (C) bulimia. (D) kwashiorkor. (E) marasmus.

The answer is D. The clinical description is characteristic of systemic anaphylaxis, an IgE-mediated type I hypersensitivity reaction. In type I hypersensitivity, reaction of antigen with preformed IgE antibodies fixed by Fc receptors to the surface of basophils or tissue mast cells results in cytolysis and degranulation of these cells, with release of histamine and other mediators.

Within minutes of a bee sting, a 23-year-old woman develops generalized pruritus and hyperemia of the skin, followed shortly by swelling of the face and eyelids, dyspnea, and laryngeal edema. This reaction is mediated by (A) antigen-antibody complexes. (B) cytotoxic T cells. (C) IgA antibodies. (D) IgE antibodies. (E) IgG antibodies.


Conjuntos de estudio relacionados

Network+ Chapter 7: Virtualization and Cloud Computing

View Set

Economics Module 3: Applications of Supply and Demand

View Set

Chapter 18: Planning Nursing Care

View Set

Virginia DMV Practice Test, Learners Permit, Questions to Review, 2021

View Set

Chapter 27: Safety, Security, and Emergency Preparedness - PrepU

View Set

CSS145 Midterm - Case Studies in Customer Service

View Set

Credit Reporting Agencies and Financial Regulations

View Set